You are on page 1of 117

Bases Fsicas del Medio Ambiente

Grado en Ciencias Ambientales

Mara Vir. Martnez Gonzlez

Bases Fsicas del Medio Ambiente


Bloque Temtico 1: Mecnica
1.1 Sistemas de Unidades. Dimensiones fsicas. 1.2 Cinemtica. 1.3 Dinmica. 3 4 7 18

Bloque Temtico 2: Vibraciones y Ondas


2.1 Oscilaciones 2.2 Ondas 2.3 Interferencia. Ondas estacionarias

32
33 42 48

Bloque Temtico 3: Campos elctricos y magnticos


3.1 La interaccin elctrica 3.2 Corriente elctrica 3.3 El campo magntico 3.4 Induccin magntica 3.5 Campo electromagntico y radiacin

52
54 67 72 77 81

Bloque temtico 4: Termodinmica


4.1 Propiedades termodinmicas de la materia 4.2 Equilibrio trmico y mecnico 4.3 Equilibrio trmico en presencia de gravedad 4.4 Calor, trabajo y mquinas trmicas 4.5 Cambios de fase

84
86 91 94 98 106

Bloque temtico 5: Fsica de Fluidos


5.1 La dinmica de los fluidos 5.2 Fluidos ideales y fluidos reales 5.3 Termodinmica y dinmica de la atmsfera

111
112 112 116

Mara Vir. Martnez Gonzlez

Bloque Temtico 1: Mecnica


El anlisis dimensional es de gran utilidad en fsica. La correccin dimensional de una ecuacin en fsica es equivalente a la correccin sintctica de una expresin gramatical. Por ello es importante comprobar en cada paso de un razonamiento que no hay errores dimensionales pues, si los hubiera, es un claro indicio de que la deduccin es falsa de entrada. El anlisis dimensional tambin puede servir de ayuda para conjeturar una posible solucin a un problema, puesto que nos permite adivinar qu magnitudes, o qu parmetros, y cmo, pueden entrar en la solucin. Adems, sabiendo cmo dependen estos parmetros de las escalas de longitud o tiempo, podemos establecer leyes de semejanza vlidas en cualquier escala. Recprocamente, la existencia de leyes de escala empricas, que expresan la dependencia de algunas soluciones con respecto a determinadas magnitudes en algunos sistemas especiales, nos puede ayudar a identificar el mecanismo que hay tras dichas leyes. El primer paso en el estudio de la fsica es el estudio del movimiento. Puesto que, en general, el movimiento tiene lugar en varias dimensiones, las magnitudes cinemticas se expresan mediante vectores. Por lo tanto, para el estudio del movimiento es necesario un conocimiento de las ideas bsicas sobre vectores, en especial las ideas de producto escalar y vectorial. La cinemtica describe el movimiento sin atender a las causas que lo producen: sencillamente nos da la posicin de un mvil en funcin del tiempo. No obstante, una descripcin detallada del movimiento permite obtener relaciones entre la aceleracin de un mvil y la posicin que ocupa. Entramos ahora en la dinmica. La segunda ley de Newton nos dice que la aceleracin que adquiere un cuerpo es proporcional a las fuerzas que actan sobre el mismo. Por lo tanto, a partir del movimiento de un cuerpo podemos deducir la forma de las fuerzas que actan sobre l. Recprocamente, conociendo las fuerzas podemos deducir el movimiento. A partir de las leyes de Newton se pueden deducir ciertos principios de conservacin, es decir, la existencia de ciertas magnitudes que se conservan durante el movimiento del cuerpo. Las leyes de conservacin fundamentales son: 1) ley de conservacin de la cantidad de movimiento: la cantidad de movimiento en una direccin se conserva si sobre el cuerpo no actan fuerzas que tengan componentes en dicha direccin; 2) ley de conservacin del momento angular: el momento angular de un sistema se conserva si todas las fuerzas que actan sobre el sistema son fuerzas centrales; 3) ley de conservacin de la energa: el trabajo total realizado por las fuerzas que actan sobre un cuerpo es igual a la variacin de la energa cintica de dicho cuerpo. Existe un tipo especial de fuerzas, llamadas fuerzas conservativas, que se pueden obtener por derivacin a partir de una funcin de las coordenadas. Esta funcin es un campo escalar que se denomina energa potencial, y la fuerza correspondiente es el gradiente de dicho campo. De este modo, dada la energa potencial podemos conocer las caractersticas fundamentales del movimiento en dicho campo: puntos de equilibrio estable e inestable, lmites del movimiento, etc. Adems, el trabajo que realiza la fuerza conservativa es sencillamente igual a la variacin de la energa potencial. No obstante, no todas las fuerzas son conservativas. Por ejemplo, hay fuerzas que dependen de la velocidad o de otros parmetros que no pueden deducirse a partir de una funcin de la posicin. Entre las fuerzas no conservativas las ms importantes son las fuerzas de rozamiento. Estas fuerzas siempre se oponen al movimiento y, por consiguiente, reducen la energa cintica del cuerpo. Las fuerzas de rozamiento a las que est sometido un cuerpo que se mueve en un fluido dependen de la velocidad y dan lugar a movimientos caractersticos que tienden asintticamente hacia unos valores lmite. Asimismo, el movimiento de unas regiones de un medio continuo es obstaculizado por las regiones vecinas del medio, lo que se traduce en un coeficiente de viscosidad. Estas fuerzas sern importantes para el estudio de la fsica de fluidos.
2 Mara Vir. Martnez Gonzlez

CAPTULO 1
Dimensiones fsicas
Las leyes de la fsica se expresan mediante ecuaciones que ligan diferentes magnitudes fsicas. Los argumentos dimensionales son importantes en la bsqueda de leyes o para detectar si una ecuacin es erronea. Para ello hay que tener en cuenta algunos requisitos simples: a) Los dos miembros de una ecuacin deben tener las mismas dimensiones. b) Todas los trminos de una suma deben tener las mismas dimensiones. c) Las constantes numricas (por ejemplo, , e) no tienen dimensiones. d) Los argumentos de funciones trigonomtricas, exponenciales o logartmicas no tienen dimensiones. Ejercicio 1.- Cules son las dimensiones de la constante gravitatoria G en la expresin de la fuerza gravitatoria entre dos masas M y m Mm F =G 2 r Solucin: La fuerza F tiene dimensiones de masa por aceleracin, es decir, [F ] = MLT2 . Entonces, la ecuacin en dimensiones correspondiente a la expresin anterior es M2 MLT2 = [G] 2 L y as [G] = M1 L3 T2 Ejercicio 2.- Tenemos una expresin emprica que da la velocidad de un mvil en funcin de la distancia x a un punto dado p v = C1 C2 x siendo C1 y C2 constantes. Qu dimensiones tienen estas constantes? Solucin: Escribamos v 2 = C1 C2 x. Entonces, teniendo en cuenta el requisito b): v 2 = [C1 ] = [C2 x]; y puesto que [v ] = LT 1 , tenemos [C2 ] = LT2 L2 T2 = [C1 ] = [C2 ] L Ejercicio 3.- Es correcta dimensionalmente la expresin A(t) = A0 sin t exp t2 ?

Solucin: El argumento del seno debe ser adimensional, de modo que [] =T1 . De la misma forma, el exponente tambin debe ser adimensional, de modo que [] =T2 . Esta contradiccin muestra que la expresin es incorrecta.

Ejercicio 4.- Un objeto que se mueve en el seno de un fluido experimenta una fuerza de rozamiento cuyo valor es proporcional a la velocidad del objeto con respecto al fluido, FR = kv. Qu dimensiones tiene k ? Solucin: Escribiendo la correspondiente ecuacin dimensional MLT2 [F ] = = MT1 [k ] = [v ] LT1

Ejercicio 5.- Experimentalmente se ve que, en el caso de un objeto esfrico, la resistencia que ofrece el fluido puede depender del radio R del objeto y de la densidad y la viscosidad cinemtica del fluido. Cmo podemos combinar estas tres magnitudes para obtener una constante con las dimensiones de la k anterior? (La viscosidad cinemtica tiene dimensiones [ ] =L2 T1 ) Solucin: y el radio de la Supondremos que la densidad contribuye de la forma , la viscosidadlo hace de la forma forma R . Por lo tanto, tenemos que encontrar los valores de , , tales que R = [k ] = MT 1 . 3 2 1 R = ML L T L = M L3+2 + T = MT1

Mara Vir. Martnez Gonzlez

Entonces

=1 3 + 2 + = 0 ===1 = 1 y as la constante k es proporcional a R. El argumento dimensional no puede darnos la constante de proporcionalidad, puesto que las constantes numricas no tienen dimensiones. Un tratamiento terico detallado permite demostrar que k = 6 R y as FR = 6 Rv, que se conoce como ley de Stokes. (Se estudiar en el Captulo 18).

Leyes de escala
Si conocemos cmo depende un fenmeno de los parmetros del sistema y, a su vez, conocemos cmo dependen dichos parmetros del tamao del sistema, podemos saber cmo depende el fenmeno del tamao del sistema y deducir leyes de escala. La ecuacin de movimiento para un objeto de masa m sometido a una fuerza de rozamiento del tipo anterior es dv = kv (1) m dt cuya solucin es v (t) = v0 e(k/m)t v0 et/ (2) donde hemos definido m/k que es una constante con dimensiones de tiempo (como debe ser para que el exponente t/ sea adimensional). Asi, para t = , ( ) = 0 /e; para t = 2 , (2 ) = 0 /e2 y as sucesivamente. Por cada tiempo caracterstico que transcurre, la velocidad se reduce en un factor e. Por lo tanto, en todos los sistemas de este tipo (cuerpo en el seno de un fluido) para los que tenga el mismo valor, la velocidad del objeto variar de la misma forma. Ahora bien, la masa del objeto, m, es proporcional a su volumen, que es proporcional a R3 . Y puesto que hemos visto en un ejercicio anterior que k R, tenemos R3 m = R2 = k R Por lo tanto, si se duplica el radio del objeto que se mueve en el fluido, la constante de tiempo se multiplica por 4 y la velocidad viene dada ahora por v (t) = v0 et/4 . Ejercicio 6.- La variacin de temperatura T de un cuerpo que se deja enfriar en un ambiente a temperatura Ta es proporcional a la diferencia de temperatura T Ta (sta es la ley de enfriamiento de Newton que se ver en el Captulo 13). Cunto tarda la diferencia de temperatura en reducirse en un factor e? Solucin: Expresada en trminos matemticos, la ley de enfriamiento de Newton es dT = (T Ta ) dt (3)

que es similar a la ecuacin (1). Su solucin es T (t) Ta = (T0 Ta ) et (siendo T0 la temperatura inicial del cuerpo). As, la diferencia de temperatura T (t) Ta se reduce en un factor e por cada tiempo tc = 1/ transcurrido. V eamos ahora algunas leyes de escala relacionadas con el enfriamiento de esferas. Podramos hacer un anlisis de la constante en (3) similar al que hemos hecho para la constante de friccin en el Ejercicio 4, aunque no hace falta entrar en detalles. Lo importante es que el calor contenido en una esfera es proporcional a su masa; y para esferas de la misma densidad, la masa es proporcional al volumen, y por lo tanto a R3 . Por otra parte, el calor se pierde a travs de su superficie, que es proporcional a R2 . Por lo tanto, las prdidas escalan como R2 /R3 = 1/R. Es decir, una esfera pequea se enfra ms rpidamente que una grande. Que la relacin entre superficie y volumen es inversamente proporcional a las dimensiones lineales es vlido para cualquier cuerpo con superficies suaves. Por eso, los nios se enfran ms fcilmente que los adultos. En general, todos los intercambios de masa y energa con un slido tienen lugar a travs de su superficie. Por ello, el ritmo de dichos intercambios escala de manera inversamente proporcional a su tamao lineal. Esto es cierto, por ejemplo, en el caso de un slido que se disuelve en un lquido, y por ello troceamos el slido en fragmentos ms pequeos para que se disuelva antes.

Mara Vir. Martnez Gonzlez

Ejercicio 7.- Tenemos un bloque cbico de arista L y lo dividimos en 8 cubos de arista L/2. Cmo ha variado tras la divisin la razn de superficie a volumen? Solucin: Inicialmente el volumen es Vini = L3 , y la superficie es 6L2 , de modo que la rzn de superficie a volumen es 6/L. Tras la divisin, el volumen se mantiene, pero la superfice es ahora 8 6(L/2)2 = 12L2 , de modo que la razn de superficie a volumen se ha multiplicado por 2. Si volviramos a dividir cada uno de los bloques, la razn superficie a volumen se volvera a duplicar. En definitiva, tras n divisiones la razn superficie a volumen se multiplicara por 2n , que se hace ilimitadamente grande cuando n crece. V eamos ahora otras leyes de escala.1 Supongamos un peso suspendido de una cuerda. El peso es proporcional al volumen, de modo que vara como L3 . Sin embargo, la resistencia de una cuerda es proporcional a su seccin, es decir, vara como L2 . Lo mismo ocurre con el peso que puede soportar una viga. As pues, la razn entre el peso y la resistencia crece como L. Una maqueta puede aguantarse, pero si aumentamos la escala sin cambiar el material, llegar un momento en que se vendr abajo porque el peso no podr ser aguantado. Un problema tpico relacionado con esto es cmo vara la altura de los saltos que puede dar un animal con su tamao. (Consideramos animales con una constitucin similar, por ejemplo mamferos.) Cuando un animal de masa m salta una altura h su energa potencial gravitatoria cambia en una cantidad mgh (como veremos en el Captulo 3). Esta energa debe ser igual al trabajo hecho por el animal en el salto. Para un animal de tamao lineal L, este trabajo es del orden de FL. Aqu F es la fuerza muscular que vara como la seccin de los musculos, es decir, como L2 . En resumen el esfuerzo FL vara como L3 . Y puesto que la masa vara como L3 , tenemos (para un valor de g dado) mgh FL L3 h L3 Es decir, h no depende de L. Un gerbo salta tanto como un caballo. h L0

V eamos ahora un problema ligeramente diferente. Ahora tenemos un mismo animal (por ejemplo, un mismo hombre) que salta en dos ambientes diferentes (por ejemplo, la Tierra y la Luna). En este caso, la masa y el esfuerzo muscular son los mismos, y lo que vara es g. As, para un mismo esfuerzo FL, la altura h alcanzada en el salto es inversamente proporcional a g. En el problema 4 del Captulo 1 de la Unidad Didctica se muestra que g es aproximadamente proporcional al radio del astro, de modo que h 1/g 1/R y un salto en la Luna alcanzar una altura mayor que un salto en la Tierra. Por la misma razn, la altura mxima de las montaas de un planeta tambin escala con el radio del planeta. En efecto, una estimacin de la altura puede hacerse de la siguiente forma: supongamos, por simplicidad, que tenemos un cilindro de altura h. Para aumentar la altura es necesario elevar masas m hasta una altura h, lo que requiere una energa mgh. Cuando esta energa sea igual a la energa necesaria para fundir una cantidad de masa m, el cilindro no podr aumentar, pues por cada m que subamos fundimos una m equivalente en el suelo de la montaa. La energa necesaria para fundir depende solo del material. Si dos planetas tienen una constitucin similar, esta cantidad es bsicamente la misma, de modo que ghmax ' cte y hmax 1/g 1/R. Por ejemplo, el radio de Marte es aproximadamente la mitad del de la Tierra, de modo que las montaas marcianas podran ser el doble de altas que las de la Tierra. De hecho, el monte Olimpo de Marte tiene una altura de unos 27 Km, tres veces la del Everest. La explicacin de est discrepancia es la misma que la que vimos en el problema 4 del Captulo 1 al hablar de la gravedad lunar. Si las densidades no son iguales hmax 1/g 1/R. La densidad de marte es aproximadamente 0.7 veces la de la Tierra. Entonces RT 1 hmax (M arte) = T = 2.85 = hmax (T ierra) M RM 0, 7 0, 5 Evidentemente todos estos razonamientos son cualitativos y adems se refieren a las alturas mximas posibles y no a las reales. Estas ltimas dependern de la actividad tectnica de cada planeta. No obstante, es un buen terreno para ver cmo razonamientos simples llevan a estimaciones bastante aproximadas o permiten descartar algunos modelos.

1 Estos ejemplos se dan solo a ttulo informativo y el alumno puede pasarlos por alto. No obstante, su lectura es recomendable, pues muestra cmo se pueden explicar cualitativamente muchos fenmenos sin necesidad de conocer las leyes exactas que los gobiernan.

Mara Vir. Martnez Gonzlez

CAPTULO 2
Cinemtica
En la seccin 2.2 de la Unidad Didctica se introducen los conceptos de velocidad y aceleracin instantnea para un cuerpo que se mueve en una sola dimensin. Es inmediato extender estos conceptos al movimiento en 3 dimensiones La trayectoria de un mvil en el espacio tridimensional se especifica mediante una funcin ~ r=~ r(t) que nos da la posicin que ocupa el mvil en cada instante de tiempo. Aqu ~ r = (x, y, z ) es un vector con 3 componentes, de modo que la funcin vectorial ~ r=~ r(t) equivale a tres funciones escalares x = x(t), y = y (t), z = z (t). La velocidad instantnea se define cmo d~ r(t) (1) ~ v (t) = dt que, en general, es tambin una funcin de t. Y la aceleracin instantnea se define como d~ v (t) d2~ r(t) ~ a(t) = = (2) dt dt2 Si podemos eliminar t entre estas ecuaciones, podemos encontrar relaciones que ligan directamente la velocidad o la aceleracin con la posicin que ocupa el mvil. Las ecuaciones (1) y (2) anteriores equivalen a Z ~ r(t) = ~ v (t)dt Dos tipos importantes de movimiento son: a) movimiento uniforme: v (t) = v0 = constante. Entonces (suponiendo v0 a lo largo del eje X) las ecuaciones (2) y (3) se traducen en Z t dv0 =0 x(t) x(t0 ) = v0 d = v0 (t t0 ) (4) a= dt t0 b) movimiento uniformemente acelerado: a(t) = a0 = cte. Entonces Z t v (t) v (t0 ) = a0 d = a0 (t t0 )
t0

~ v (t) =

~ a(t)dt

(3)

(5) (6)

Para t0 = 0, estas ecuaciones se reducen a las de la seccin 2.2 de la Unidad Didctica. En particular, la ecuacin (6) queda 1 x(t) x(t0 ) = at2 + v0 t 2 de modo que para obtener el tiempo t necesario para recorrer una distancia D = x(t) x(t0 ) hay que resolver una ecuacin de segundo grado, y la solucin es p 2 + 2aD v0 v0 (8) t= a Si, adems, v0 = 0, esta expresin se reduce a r 2D (9) t= a Ejercicio 1.- La trayectoria que ocupa un mvil que se mueve en una dimensin en un instante t viene dada por una funcin de la forma x(t) = Ctn , siendo una constante.

1 2 a0 (t t0 ) + v0 (t t0 ) 2 t0 t0 Teniendo en cuenta que de (5) obtenemos t t0 = (v (t) v (t0 ))/a0 , podemos escribir (6) como 1 (v (t) v (t0 ))2 1 (v (t) v (t0 )) v (t)2 v (t0 )2 + v0 = x(t) x0 = a0 2 a2 a 2 a 0 0 0 x(t) x(t0 ) = v ( )d = [a0 ( t0 ) + v (t0 )] d =

(7)

Mara Vir. Martnez Gonzlez

a) Cules son las dimensiones de C ? b) Escribir las expresiones para la velocidad y la aceleracin en funcin del tiempo. c) Escribir expresiones para la velocidad y la aceleracin en funcin de la posicin. d) Particularizar los resultados para n = 0, n = 1 y n = 2. Solucin: a) Evidentemente [C ] = LTn L = [C ] Tn b) dv d(Ctn ) = nCtn1 = n(n 1)Ctn2 a= v= dt dt c) De la ecuacin de la trayectoria tenemos t = (x/C )1/n , de modo que x (n1)/n v = nC = nC 1/n x(n1)/n C x (n2)/n a = n(n 1)C = n(n 1)C 2/n x(n2)/n C d) n=0: x=C v=0 a=0 ausencia de movimiento n=1: x = Ct v = C a=0 movimiento uniforme v = 2 Cx a = 2C movimiento uniformemente acelerado n=2: x = Ct2 Ejercicio 2.- Si los frenos de un automvil producen una deceleracin de a = 4 m/s2 , cunto tardar en llegar al reposo un automvil que circula a 36 Km/h? Que distancia habr recorrido en la frenada? Solucin: Con objeto de obtener los valores numricos que nos piden, debemos poner la velocidad en unidades del S.I. Esto es sencillo en este caso 1000 m Km = 36 = 10 m/s. 36 h 3600 s Si suponemos que la frenada empieza en el instante t = 0, tenemos v0 = 10 m/s y por (5) v (t) v0 = at v (t) = v0 at = (10 4t) m/s que se hace 0 para t1 = 2, 5 s. La distancia recorrida durante este periodo ser, aplicando (6 1 1 = 10 2, 5 4 (2, 5)2 = 12, 5 m x(t1 ) = v0 t1 + at2 2 1 2 Tambin podramos haber aplicado (7). Entonces, la distancia recorrida cuando la velocidad v (t) se ha hecho igual a 0 es 102 v2 = 12, 5 m x(t1 ) = 0 = 2a 8 Ejercicio 3.- Cul sera la distancia de frenada si el automvil fuera a 72 km/h? Solucin: No tenemos que hacer ningn clculo nuevo. Acabamos de ver que, para una aceleracin dada, la velocidad recorrida es proporcional al cuadrado de la velocidad inicial. Por lo tanto, si se duplica esta velocidad, la distancia de frenada se multiplica por 22 = 4. Es decir, la nueva distancia de frenada es 50 m. Ejercicio 4.- Un objeto cae, partiendo del reposo, desde una altura H = 125 m. Qu distancia recorre durante el ltimo segundo de cada? (Para simplificar los clculos puede tomarse |g | = 10 m/s2 ) Solucin: Si en la ecuacin (9) hacemos D = H y a = g = 10 m/s2 , el tiempo total de cada ser s r 2H 2 125 m = tT = =5s g 10 m s2 En los 4 primeros segundos el objeto habr cado una distancia h tal que s 1 1 2h h = g (4 s)2 = 10 m s2 42 s2 = 80 m 4s= g 2 2

Mara Vir. Martnez Gonzlez

de modo que la distancia recorrida en el ltimo segundo es d = H h = 45 m Ejercicio 5.- Desde lo alto de una torre dejamos caer una bola en el instante t0 = 0 . En un instante tc posterior dejamos caer una segunda bola. Cmo vara la separacin entre las dos bolas en funcin del tiempo? (Las dos bolas caen con la aceleracin a = g de la gravedad terrestre. Tomar, por simplicidad g = 10 m/s2 )

Solucin: Si la primera bola se deja caer en el instante t0 = 0, su coordenada vertical y1 en un instante t ser 1 y1 (t) = gt2 2 La segunda bola empieza a caer en el instante tc , de modo que en el instante t solo lleva cayendo un tiempo t tc . Entonces 1 y2 (t) = g (t tc )2 2 y as tc 1 2 1 2 y1 (t) y2 (t) = gt g (t tc ) = gtc t 2 2 2 de modo que el valor absoluto |y1 (t) y2 (t)| aumenta con el tiempo t. Por supuesto, la expresin anterior solo es vlida para t tc ; para t < tc la segunda bola no ha empezado a caer. Para t = tc la expresin da y1 (t) y2 (t) = (1/2)gt2 c , lo que es natural ya que en dicho instante y2 (tc ) = 0 y la primera bola lleva un tiempo tc cayendo, durante el que ha cado una distancia (1/2)gt2 c.

Solucin: Supondremos que el origen de coordenadas est en lo alto del edicicio. Entonces la coordenada y del suelo es ysuelo = h. Si llamamos t1 al tiempo que tarda en llegar al suelo la primera bola tenemos 1 h = gt2 2 1 La segunda bola parte con una velocidad inicial v y llega al suelo en un tiempo t2 = t1 + t, donde t = 4 s. Entonces 1 1 = v (t1 + t) + g (t1 + t)2 h = vt2 + gt2 2 2 2 Igualando los segundos miembros de estas expresiones obtenemos 1 g (t)2 v t 1 =4s t1 = 2 (v + g t) t1 = g (t)2 v t 2 v + g t que es el tiempo que tarda en llegar al suelo la primera bola. La altura del edificio es entonces 1 h = g 42 = 80 m 2 Ntese que la bola lanzada hacia arriba alcanza la velocidad cero para un tiempo de subida ts tal que v + gts = 0, es decir, ts = 3 s. En ese momento est a una altura 1 y (ts ) = vts + gt2 = 45 m 2 s por encima del edificio. As pues, para llegar al suelo tiene que caer ahora una distancia de 80+45=125 metros partiendo de una velocidad cero, lo que le costar un tiempo de bajada tb dado por 1 125 = gt2 tb = 5 s. 2 b de modo que el tiempo total que la segunda piedra ha estado en el aire es t2 = ts + tb = 8 s. que, como dice el enunciado, es 4 segundos ms que t1. Ejercicio 7: Supongamos el mismo edificio del problema anterior con una altura h = 80 m. En el instante t = 0 dejamos caer desde lo alto una piedra. Dos segundos despus, lanzamos desde lo alto una segunda piedra pero

Ejercicio 6.- Desde lo alto de un edificio se deja caer una piedra. Al mismo tiempo, se lanza verticalmente hacia arriba con una velocidad v = 30 m/s una segunda piedra que, tras alcanzar una cierta altura, se frena y empieza a caer hacia abajo. La segunda piedra llega al suelo 4 segundos despus que la primera. Cul es la altura h del edificio? Cunto tiempo tard en en llegar al suelo la primera piedra? (Para simplicidad en los clculos, tomar g = 10 m/s2 .

Mara Vir. Martnez Gonzlez

esa vez con una velocidad inicial v < 0 (es decir, hacia abajo). Cunto debe valer v para que las dos piedras lleguen al suelo al mismo tiempo? Solucin: La distancia recorrida por la primera piedra en un instante t cualquiera ser 1 y1 (t) = gt2 2 de modo que llega al suelo en el instante s 2h =4s tf = g La segunda piedra empieza a caer en el instante tc = 2 s con una velocidad v en dicho instante. Entonces la distancia recorrida en un instante cualquiera t ser 1 z2 (t) = v (t tc ) + g (t tc )2 2 y puesto que tambin llega al suelo en el instante tf tendremos 1 g (tf tc )2 h 1 20 80 = 30 m/s v= 2 = h = v (tf tc ) + g (tf tc )2 2 tf tc 2 Ejercicio 8: En el instante t = 0 se deja caer una piedra desde lo alto de un acantilado. Dos segundos despus se lanza una piedra hacia abajo pero esta vez con una velocidad inicial v = 30 m/s. Las dos piedras llegan al mismo tiempo al suelo. Cul es la altura h del acantilado? Solucin: La distancia recorrida por la primera piedra en un instante t cualquiera ser 1 y1 (t) = gt2 2 La segunda piedra empieza a caer en el instante tc = 2 s con una velocidad v = 30 m/s en dicho instante. Entonces la distancia recorrida en el instante t ser 1 y2 (t) = v (t tc ) + g (t tc )2 2 Puesto que las 2 piedras llegan al suelo en el mismo instante tf , tenemos y1 (tf ) y2 (tf ) = 0; es decir 2 vtc 1 1 60 + 20 2 gtc =4s + gt t t = = 0 = v (tf tc ) + gt2 c f f c 2 v + gtc 30 + 20 Por lo tanto, la altura del acantilado es 1 h = g 42 = 80 m 2 Ntese que este ejercicio es prcticamente el mismo que el anterior. En el ejercicio 7 se daba la altura 80 m. y se peda la velocidad de lanzamiento, que resultaba ser de 30 m/s. En este ejercicio se da esta velocidad y se pide la altura, que lgicamente debe ser de 80 m Aunque los movimientos uniforme y uniformemente acelerado son importantes y obedecen a ecuaciones muy simples, hay otros tipos de movimiento igualmente importantes. Ejercicio 9: La trayectoria de un mvil viene dada por una funcin de la forma x(t) = A exp t. a) Determinar la velocidad y la aceleracin del mvil en cualquier instante. Solucin: dv (t) dx(t) = A exp t a(t) = = 2 A exp t v (t) = dt dt Adems es fcil ver que v (t) = x(t) a(t) = v (t) = 2 x(t) Ejercicio 10: La trayectoria de un mvil viene dada por una funcin de la forma x(t) = A sin ( t + ) donde A, y son constantes. a) Cuales son las dimensiones de A, y ?

Mara Vir. Martnez Gonzlez

b) Determinar la velocidad y la aceleracin del mvil? Solucin: a) Los argumentos de los ngulos no tienen dimensiones de modo que [ t + ] = 1 [ t] = [] = 1 [ ] = T1 Por otra parte, como el seno no tiene dimensiones, tenemos [A] = L b) dv (t) dx(t) = A cos( t + ) a(t) = = 2 A sin( t + ) dt dt De la segunda expresin es inmediato que a(t) = 2 x(t). Podemos comparar esta relacin con la del problema anterior: a(t) = 2 x(t). La diferencia es que ahora el signo est cambiado. Ahora la aceleracin tiene el mismo sentido que el desplazamiento, mientras que en el problema anterior la aceleracin tiene sentido contrario al del desplazamiento. En cuanto a la expresin para la velocidad, es fcil ver que q p cos( t + ) = 1 sin2 ( t + ) = 1 (x/A)2 y as p p v = A 1 (x/A)2 = A2 x2 v (t) = Un movimiento de este tipo se llama movimiento armnico simple, y tendremos ocasin de estudiarlo con ms detalle en el Captulo 4. Por el momento basta con sealar que cuando el mvil pasa por el origen x = 0, su velocidad es mxima vmax = A y su aceleracin es cero. Por el contrario, cuando el mvil llega a la distancia mxima x = A, su velocidad es nula y su aceleracin es mxima.

Ejercicio 11: En el instante inicial t = 0 un mvil parte del origen de coordenadas con una velocidad inicial v (0) = v0 y una aceleracin inicial a(0) = a0 = v0 . A partir de entonces la aceleracin del mvil obedece a la expresin a(t) = a0 exp (t) . Cuales son las velocidad y la posicin del mvil en cualquier instante t? Cules son la velocidad y la posicin en trminos de la distancia del mvil al origen? Solucin: De la ecuacin (3) Z t Z t a( )d = v0 exp ( ) d = v0 exp ( )]t v (t) v0 = 0 = v0 [exp (t) 1]
0 0

y as

v (t) = v0 exp(t) que decae asinttica y exponencialmente hacia cero. La distancia recorrida en un tiempo finito t es Z t Z t v0 [1 exp(t)] v ( )d = v0 exp ( ) d = x(t) = 0 0 Ahora es fcil ver que 1 (/v0 )x(t) = exp(t) y as v (t) = v0 1 x(t) a(t) = a0 1 x(t) v0 v0 Para t 0, la velocidad tiende a cero y la distancia recorrida tiende a un valor finito x() = v0 /.

Movimiento en dos dimensiones


Ejercicio 12: Desde lo alto de un acantilado se lanzan horizontalmente y al mismo tiempo dos piedras. Una de ellas se lanza con una velocidad v1x = 20 m/s y la otra se lanza con una velocidad v2x . La primera piedra llega al suelo a una distancia de 160 m del pie del acantilado y la segunda llega al suelo a una distancia de 200 m del pie del acantilado. Con que velocidad v2x se lanz la segunda piedra? Cul es la altura del acantilado? Cunto tiempo tardan en caer las piedras? (Tomar, por simplicidad, g = 10 m/s2 ) Solucin: En el ejercicio de la pgina 24 de la Unidad Didctica se estudia el caso general del movimiento de un proyectil en el campo gravitatorio terrestre, cuando la velocidad inicial forma un ngulo con la horizontal. En el caso particular

10

Mara Vir. Martnez Gonzlez

= 0,tenemos 1 |g | 2 2x 2 v0 (donde hemos escrito explcitamente |g | para dejar claro que la y es negativa. Tomamos el origen de coordenadas en el borde del acantilado). a) Sea h la altura del acantilado. Cuando la primera piedra llega al suelo, su coordenada horizontal es x1 = 160 m. de modo que |g | h = 2 x2 2v1x 1 De la misma forma se tiene para la segunda piedra |g | h = 2 x2 2v2x 2 De las dos ecuaciones es inmediato x2 200 x2 x2 1 m/s = 25 m/s = 22 v2x = v1x = 20 2 v1x v2x x1 160 b) De la primera ecuacin 10 h= (160)2 m = 5 82 m = 320 m 2 (20)2 c) Ambas piedras tardan en caer el mismo tiempo s r 2h 2 320 tc = = s=8s g 10 y= Otro modo de caracterizar el movimiento en dos dimensiones es mediante la utilizacin de coordenadas polares, cuya relacin con las coordenadas cartesianas se estudia en la seccin 2.2.1 de la Unidad Didctica. Un tipo de movimiento particularmente simple es el movimiento circular, en el que la distancia al origen es constante. En este caso, la posicin del mvil queda definida por el ngulo a lo largo de un crculo de radio = R. La velocidad y la aceleracin angular se definen ahora como d (t) d2 (t) d(t) (t) = = (t) = dt dt dt expresiones anlogas a las (1), (2) para el movimiento en una dimensin. Asimismo, podemos escribir Z Z (t) = (t)dt (t) = (t)dt Ejercicio 13: Calcular explcitamente las componentes cartesianas de la posicin, velocidad y aceleracin para el movimiento circular uniforme de radio R. Demostrar que este movimiento es combinacin de dos movimientos armnicos simples. Solucin: En el caso del movimiento circular uniforme = cte, (t) = t + 0 . Si suponemos que 0 = 0 para t = 0 (es decir, para t = 0 el mvil est situado sobre el eje X), tenemos dx(t) dvx (t) = R sin t ax (t) = = R 2 cos t x(t) = R cos (t) = R cos t vx (t) = dt dt dy (t) dvy (t) = R cos t ay (t) = = R 2 sin t y (t) = R sin (t) = R sin t vy (t) = dt dt Evidentemente x2 + y 2 = R2 (cos2 t + sin2 t) = R2 , que es la ecuacin de una circunferencia de radio R. Ntese que d2 x(t) d2 y (t) 2 ax (t) = = x ( t ) a ( t ) = = 2 y (t) (10) y dt2 dt2 Pero hemos visto antes que estas expresiones corresponden al movimiento armnico simple. V emos entonces que un movimiento circular uniforme de velocidad angular puede descomponerse en dos movimientos armnicos perpendiculares de frecuencia . Los mdulos de la velocidad y la aceleracin son q q v2 2 2 + v2 = R 2 a = a2 v = vx y x + ay = R = R

11

Mara Vir. Martnez Gonzlez

En la seccin 2.2.1 de la Unidad Didctica se ha visto que la velocidad tiene la direccin de ~ e ,es decir, es perpendicular al radio vector. Entonces debe cumplirse ~ r ~ v = 0. En efecto ~ r ~ v = xvx + yvy = R cos t (R sin t) + R sin t (R cos t) = 0 Asimismo, la aceleracin es perpendicular a la velocidad ~ a~ v = ax vx + ay vy = R2 cos t (R sin t) + R 2 sin t (R cos t) = 0 y por lo tanto tiene la direccin del radio vector. De hecho, ya hemos visto en (10) que para el movimiento circular r(t) ~ a(t) = 2~ Ejercicio 14: En el caso del movimiento circular que acabamos de estudiar, los movimientos a lo largo de los ejes X e Y tienen la misma amplitud R. Qu sucede si componemos movimientos armnicos de la misma frecuencia y diferentes amplitudes, Ax , Ay ? Solucin: Ahora x(t) = Ax cos t vx (t) = Ax sin t ax (t) = 2 Ax cos t y (t) = Ay sin t vy (t) = Ay cos t ay (t) = 2 Ay sin t de modo que y2 x2 + =1 (11) A2 A2 x y que es la ecuacin de una elipse de semiejes Ax y Ay . En cuando a los mdulos de la velocidad y la aceleracin, obtenemos q q que dependen del tiempo. Ahora ~ r ~ v
2 + v 2 = A2 sin2 t + A2 cos2 t vx y x y q q 2 2 2 2 2 a = a2 A2 x + ay = x cos t + Ay sin t

= Ax cos t (Ax sin t) + Ay sin t (Ay cos t) = 2 = A2 y Ax sin t cos t ~ r ~ a = Ax cos t Ax 2 cos t + Ay sin t Ay 2 sin t = 2 2 2 = 2 A2 x cos t + Ay sin t de modo que ~ v solo es ortogonal a ~ r cuando sin t = 0 o cos t = 0; es decir, cuando = 0 y = (primer caso) o cuando = /2 y = 3 /2 (segundo caso). En todos estos casos la aceleracin tiene sentido contrario al radio vector; es decir, est dirigida hacia el origen de coordenadas. Tambin es fcil deducir las relaciones generales 2 2 vy a2 a2 vx y x 2 + = + = 4 2 2 2 A2 A A A x y x y

Movimiento relativo
En la seccin 2.3 de la Unidad Didctica se explica la idea de la aceleracin de Coriolis, que es la aceleracin que tiene que introducir un observador situado en un sistema de referencia en rotacin. Para profundizar en esta idea, vamos a comparar la descripcin de un movimiento simple desde dos sistemas de referencia, uno inercial y otro en rotacin. Sea O un observador situado en un sistema inercial en reposo. En este sistema, las coordenadas cartesianas de un mvil son x e y, y sus coordenadas polares son y . Entonces, las ecuaciones para un movimiento uniforme con velocidad ~ v a lo largo del eje X son x(t) = x0 + vt y (t) = x(t) = 0 = cte en coordenadas cartesianas y polares respectivamente. Es decir, el radio vector ~ r(t) est siempre en la direccin del eje X. y (t) = 0

12

Mara Vir. Martnez Gonzlez

V eamos ahora cmo se describe el movimiento en un sistema de referencia en rotacin con velocidad ~ . (El origen de coordenadas es el mismo que en el sistema en reposo.) El vector ~ es perpendicular al plano XY y est dirigido en la direccin que sale del papel; por lo tanto, es perpendicular a ~ v y as |~ ~ v | = v. Si suponemos que en t = 0 los ejes X y X de los dos sistemas coinciden, en un instante t posterior formarn un ngulo t. Las coordenadas cartesianas del mvil en el sistema en rotacin son las proyecciones de ~ r(t) sobre los ejes X e Y y (t) = x(t) cos t = (x0 + vt) sin t x (t) = x(t) cos t = (x0 + vt) cos t Las componentes cartesianas de la velocidad son entonces dx (t) vx (t) = = v cos t (x0 + vt) sin t = v cos t sin t dt dy (t) = v sin t (x0 + vt) cos t = v sin t cos t vy (t) = dt y el mdulo v 2 = v 2 + (x0 + vt)2 2 = v 2 + 2 2 En cuanto a la aceleracin dv (t) = 2v sin t 2 cos t ax (t) = x dt dvy (t) = 2v cos t + 2 sin t ay (t) = dt y 2 2 a 2 = (2v ) + 2

Podemos descomponer as la aceleracin en el sistema en rotacin en una parte cuya amplitud solo depende de v y , y otra parte cuya amplitud depende de y de la distancia al origen . La primera es la aceleracin de Coriolis 2 aCentr p =

y la segunda es la aceleracin centrpeta ~ v| |aCoriolis | = 2v = 2 |~

Para completar este estudio veamos cmo se describe el movimiento en coordenadas polares y en el sistema en rotacin. Evidentemente, puesto que mide la distancia al origen de coordenadas, y este origen es el mismo para ambos sistemas de referencia, debe ser (t) = (t). En efecto q p (t) = x (t)2 + y (t)2 = (x0 + vt)2 cos t + (x0 + vt)2 sin2 t = x0 + vt = (t) Por otra parte = t Las componentes polares de la velocidad son entonces d (t) d (t) =v v = = v = dt dt con lo que de nuevo llegamos a 2 = v 2 + 2 2 v 2 = v2 + v

Gradientes y curvas de nivel


En la seccin 2.4 de la Unidad Didctica se explica la relacin general entre un campo escalar y el campo vectorial que puede derivarse a partir del mismo. Estas ideas son fundamentales en fsica pues los campos de fuerzas son campos vectoriales que se obtienen a partir de campos escalares con dimensiones de energa (energa potencial). Vamos a estudiar los conceptos de gradiente y curvas de nivel con un ejemplo que tiene una interpretacin fsica muy simple y directa. Consideremos una montaa que se levanta sobre un terreno llano que suponemos que es el plano XY , y supongamos un origen de coordenadas en dicho plano. Entonces, el perfil de la montaa queda definido si para cada punto de la ladera de la montaa de coordenadas (x, y ), tenemos una funcin z (x, y ) que nos da la coordenada z (es decir, la altura) de dicho punto. Entonces la altura z = z (x, y ) es un campo escalar. Por simplicidad, supondremos que la cima de la montaa est sobre el origen de coordenadas y que la montaa tiene simetra circular en torno a un eje vertical (el eje Z) que pasa por la cima, es decir, que tiene la misma forma vista desde cualquier direccin. Esto quiere decir que la altura de un punto p de la ladera de la montaa solo depende de la distancia de dicho punto al eje, es decir, que z solo depende de r = x2 + y 2 , pero no de x e y por separado.

13

Mara Vir. Martnez Gonzlez

Empezaremos suponiendo el caso de una montaa cnica. En este caso la montaa est definida por la funcin z (x, y ) = H ar = H a(x2 + y 2 )1/2 r < H/a siendo H la altura de la montaa y a un parmetro que nos da la anchura de la montaa, es decir, el valor r0 = H/a para el que z = 0. La curva de nivel correspondiente a una altura h es el lugar geomtrico de los puntos de la ladera de la montaa que estn a una altura h. Esto se obtiene haciendo z (x, y ) = h en la ecuacin anterior, de modo que 2 H h 2 2 1/2 2 2 x +y = h = H a(x + y ) a que es una circunferencia de radio rh = (H h)/a. Para h = H (altura mxima) rH = 0 y la curva de nivel se reduce a un punto (el vrtice de la montaa cnica). Para h = 0, la curva de nivel tiene un radio r0 = H/a, como ya hemos dicho. Para la curva de nivel correspondiente a una altura h + h tendremos un radio rh+h = (H h h)/a. Entonces, rh rh+ = h/a. Es decir, la diferencia de los radios de las curvas de nivel solo depende de la diferencia de alturas. Las curvas de nivel correspondientes a diferencias de altura iguales estn igualmente espaciadas. Ahora el vector gradiente del campo z (x, y ) es1 ax ay z (x, y ) z (x, y ) ~ , = 2 = , z = x y (x + y 2 )1/2 (x2 + y 2 )1/2 a2 x a2 y a2 = , = (x, y ) H h H h H h Puesto que H > h, el gradiente es siempre negativo, es decir, a medida que descendemos por la ladera de la montaa las coordenadas x e y crecen. Adems, el mdulo del gradiente vale 2 a2 2 ~ ~ (x2 + y 2 ) = a2 z = a z = hH Es decir, el mdulo del gradiente es el mismo, independientemente del punto (x, y ) y de la curva de nivel en la que nos movamos. En otras palabras, la pendiente de la ladera es la misma en cualquier punto. Por otra parte, puesto que las componentes X e Y del vector gradiente son proporcionales, respectivamente, ~ = (x, y ); es decir, el vector gradiente en un a las coordenadas x e y, el vector gradiente tiene la direccin del vector R punto est dirigido a lo largo del radio de la curva de nivel que pasa por dicho punto; o, lo que es lo mismo, el vector gradiente es perpendicular a las curvas de nivel. V eamos ahora un caso un poco ms complicado. La montaa sigue teniendo simetra en torno a su eje pero ahora el perfil es gaussiano z (x, y ) = H exp r2 /a2 = H exp (x2 + y 2 )/a2 La curva de nivel correspondiente a una altura h es ahora H x2 + y 2 = a2 ln h = H exp (x2 + y 2 )/a2 h p que es una circunferencia de radio rh = a ln(H/h). Para h = H tenemos rH = 0, que de nuevo se reduce a un punto. Sin embargo, para h = 0 tenemos r0 = . p La curva correspondiente a una altura h + h tiene un radio rh+h = a ln(H/(h + )) y as h + h h 2 2 2 2 = a rh = a ln ln 1 + rh + h h de modo que ahora la diferencia entre los radios de las curvas no depende solo de la diferencia de alturas h sino de la diferencia relativa h/h. El gradiente de alturas en la ladera es ahora
1 Recurdese la idea de derivada parcial que ya se ha explicado en la seccin 2.4 de la Unidad Didctica. Si nos movemos en un plano y =Y=cte, podemos escribir

y derivando con respecto a x, resulta

z (x, Y) = H a(x2 + Y2 )1/2 1 z (x, Y) = a 2x(x2 + Y2 )1/2 x 2

14

Mara Vir. Martnez Gonzlez

Resumamos lo visto hasta ahora. Para una montaa cnica tenemos: a) curvas de nivel circulares; b) el espaciado entre curvas de nivel solo depende de la diferencia de alturas; c) el gradiente es perpendicular a las curvas de nivel; d) el gradiente es el mismo en todos los puntos. Para una montaa gaussiana tenemos: a) curvas de nivel circulares; b) el espaciado entre las curvas depende de la altura; c) el gradiente es perpendicular a las curvas de nivel; d) el gradiente es constante a lo largo de una curva de nivel pero vara de una curva a otra; el gradiente es cero en h = 0 y h = H, y su valor absoluto tiene un valor mximo a una altura intermedia. Los dos casos que hemos tratado hasta ahora tenan simetra cilndrica, lo que garantizaba de entrada que las curvas de nivel fueran circulares y que el gradiente tuviese direccin radial. (Vase el problema 6 del Captulo 2 de la Unidad Didctica.) V eamos ahora una pequea variante del problema. Ahora la montaa ya no tiene una simetra de revolucin perfecta en torno a un eje sino que esta ligeramente achatada vista desde arriba. El perfil es ahora y2 x2 z (x, y ) = H exp 2 2 ax ay y la curva de nivel correspondiente a una altura h es x2 y2 H 2 2 H exp x2 /a2 + 2 = ln x y /ay = h 2 ax ay h p p que es una elipse de semiejes ax / ln(H/h), ay / ln(H/h) . Si ax > ay la elipse est alargada en la direccin del eje X y achatada en la direccin del eje Y . El vector gradiente es 2xz (x, y ) 2yz (x, y ) z (x, y ) z (x, y ) ~ , = , z = x y a2 a2 x y cuyo mdulo a lo largo de una curva de nivel correspondiente a una altura h es 2 2 x y2 ~ ~ z = 2x h, 2y h h2 = 4 + z a2 a2 a4 a4 x y x y Por lo tanto, el mdulo del gradiente ya no es constante a lo largo de una curva de nivel, sino que depende del punto de la curva en que nos encontremos. Por otra parte, no es difcil demostrar (aunque no vamos a hacerlo ahora) que este vector gradiente es perpendicular a la curva de nivel en cada punto.

2xz (x, y ) 2yz (x, y )) z (x, y ) z (x, y ) , = , x y a2 a2 y para una altura h el gradiente vale 2h 2xh 2yh ~ z = 2 , 2 = 2 (x, y ) a a a que de nuevo est dirigido en la direccin radial. El gradiente es de nuevo perpendicular a las curvas de nivel. El gradiente es negativo porque la altura disminuye cuando aumentan x e y. El mdulo del gradiente vale ahora 2h r H 2 4(x2 + y 2 )h2 4h2 ln H ~ ~ h = ln z = z = a4 a2 a h As, el mdulo del gradiente es el mismo en todos los puntos de una misma curva de nivel, pero vara de una ~ z = 0, y lo mismo pasa para h = 0. ste es curva de nivel a otra. Es inmediato ver que para h = H tenemos evidentemente el valor mnimo del mdulo del gradiente. Para cualquier otra altura hay un gradiente mayor (en valor absoluto) y, en particular, habr una altura crtica hc para la que el gradiente es mximo (en valor absoluto). Calcular cul es esta altura es un simple problema de mximos y mnimos. Para ello basta derivar la expresin del gradiente con respecto a h e igualar a cero. Procediendo as obtenemos p 1 H ln(H/hc ) p =1 hc = He1/2 ' 0, 61H = 0 2 ln hc 2 ln(H/hc ) ~z =

Ejercicio 15.- Tenemos un campo escalar con simetra circular dado por z (x, y ) = ar2 + br4 = a(x2 + y 2 ) + b(x2 + y 2 )2

15

Mara Vir. Martnez Gonzlez

a) Para que curva de nivel el gradiente vale 0? b) Cunto vale z en dicha curva de nivel? Solucin: a) El gradiente es ~ z = z (x, y ) , z (x, y ) = 2ax + 4bx(x2 + y 2 ), 2ay + 4by (x2 + y 2 x y y se hace nulo para x = y = 0,y para 2br2 = a r2 = a/2b b) Para esta curva de nivel, el valor de z (x, y ) es a2 a2 a a2 a2 = h = a + b 2 = + 2b 4b 2b 4b 4b

16

Mara Vir. Martnez Gonzlez

CAPTULO 3 Dinmica
Leyes de Newton
Recordemos algunas expresiones de las secciones 3.1 y 3.2 del libro de texto La expresin de la segunda ley de Newton es ~ = m~ F a (1) o alternativamente, considerando la masa m constante, d(m~ v) d~ p v ~ = m d~ = (2) F dt dt dt ~ es la fuerza ejercida sobre la partcula, Estas expresiones son vlidas para una partcula puntual, y la fuerza F fuerza que, evidentemente, tiene un origen externo. ~i,int internas En el caso de un sistema de partculas, sobre una partcula i del sistema pueden actuar fuerzas F ~ al sistema (fuerzas que ejercen las otras partculas) y fuerzas Fi,ext externas al sistema. As, la expresin de la segunda ley para cada partcula individual i es vi ~i,ext = mi d~ ~i,int + F F dt ~ij sobre una Por otra parte, la tercera ley de Newton afirma que si una partcula i ejerce una fuerza sobre F ~ partcula j , entonces est partcula j ejerce una fuerza Fji igual y de sentido contrario sobre la partcula i. Entonces, sumando las fuerzas que actan sobre todas las partculas de un sistema y utilizando la tercera ley de Newton se llega a N ! N N X X X ~CM d~ v dV i ~i,ext = = (3) mi mi F dt dt i=1 i=1 i=1 siendo P ~ CM vi i mi~ dR ri i mi~ ~ ~ = P (4) RCM = VCM = i mi dt i mi el radio vector y la velodidad del centro de masas del sistema. El centro de masas es un punto ideal y no tiene por qu coincidir con una partcula del sistema. De cuerdo con (3) N X ~CM = constante. ~i,ext = 0 V (5) F
i=1

En definitiva: a) Las fuerzas internas en un sistema no afectan al movimiento del centro de masas. b) Si la suma de las fuerzas externas que actan sobre un sistema de partculas es cero, el centro de masas del sistema permanece en reposo o se mueve con movimiento uniforme. Ejercicio 1.- Una persona de masa m est de pie sobre el suelo de un ascensor que se mueve en direccin vertical Qu fuerzas actan sobre la persona y qu fuerzas ejerce la persona? Qu relacin existen entre los valores de dichas fuerzas en funcin del movimiento del ascensor?

Solucin: ~ = m~ Sobre la persona acta la fuerza de atraccin gravitatoria de la Tierra, que da lugar al peso P g . (Esta ~ ez un vector unitario fuerza est dirigida hacia abajo, de modo que podemos escribirla como P = mg~ ez , siendo ~ ~ dirigido hacia arriba.) Por el principio de accin y reaccin (tercera ley de Newton), la persona ejerce una fuerza R igual y opuesta sobre la Tierra: ~ = P ~ R ~ , tambin Por otra parte, la persona que est sobre el suelo del ascensor ejerce una fuerza sobre este suelo, F dirigida haca abajo. Asimismo, por la tercera ley de Newton, el suelo ejerce sobre la persona una fuerza de reaccin ~ igual y opuesta N

17

Mara Vir. Martnez Gonzlez

~ = F ~ N Estas dos relaciones son simples expresiones de la tercera ley, y son independientes de que el ascensor est o no en movimiento. ~ (dirigida hacia abajo) y la que ejerce el V emos as que las fuerzas que actan sobre la persona son el peso P ~ (dirigida hacia arriba). Entonces, la segunda ley de Newton afirma que suelo N ~ +N ~ = m~ P a siendo ~ a la aceleracin con que se mueve la persona, que ser la misma que la del ascensor que la lleva (mientras la persona est en contacto con el suelo). Si el ascensor est en reposo o se mueve con movimiento uniforme, entonces ~ a = 0, y as ~ = N ~ P que junto con las ecuaciones que vienen de la tercera ley, dan ~ = N ~ =F ~ = R ~ P

Es decir, en este caso la fuerza que ejerce la persona sobre el suelo tiene el mismo valor y sentido que el peso de la persona. Sin embargo, si el ascensor ( y la persona dentro del mismo) se mueve con aceleracin ~ a, obtenemos ~ +N ~ =P ~ F ~ = m~ ~ =P ~ m~ a P a F a = mg~ ez m~ ~ As, si ~ a es negativa, ~ a = a~ ez , tenemos F = m(g a)~ ez y el valor de la fuerza que ejerce la persona sobre el suelo es menor que el peso de la persona. En otras palabrs, el peso efectivo que medira una balanza de resorte colocada en el ascensor sera m(g a). Si el ascensor estuviera en cada libre, entonces a = g, y el peso efectivo sera nulo. ~ = m(g + a)~ ez y la fuerza que ejerce la Por el contrario, si la aceleracin es positiva, ~ a = a~ ez ; entonces F persona sobre el suelo es mayor que el peso. Ejercicio 2.- Sobre una masa m acta una fuerza constante F = F0 durante un tiempo tc . Cunto habr aumentado su momento lineal durante ese tiempo? Solucin: Una respuesta por una va indirecta sera la siguiente: la fuerza constante hace que el cuerpo adquiera una aceleracin constante a = F0 /m. Aplicando entonces la regla del movimiento acelerado tendramos que la velocidad al cabo de un tiempo t es F0 tc v = v0 + atc = v0 + m y as p = mv mv0 = F0 tc Sin embargo, podramos haber llegado directamente a esta expresin, integrando la frmula (2) dp = F dt y p = Z
tc

F dt = F0 tc

Ejercicio 3.- Resolver el mismo problema anterior pero ahora con una fuerza variable F (t) = F0 sin t. Solucin: En este caso la aceleracin ya no es uniforme. Sin embargo, la expresin dp = F dt sigue siendo vlida, y as Z tc F0 F0 t cos t]0c = (cos tc 1) kg m s1 F0 sin( t)dt = p = 0 Ejercicio 4.- Dos patinadores, A y B, se encuentran en reposo sobre una pista de hielo sin rozamiento a una distancia de 60 m. uno de otro. La masa del primero es mA = 60 kg y la del segundo es mB = 90 kg. Ambos patinadores estn unidos por una cuerda. En un momento determinado los patinadores empiezan a tirar de la cuerda para aproximarse uno al otro. Cuando lleguen a juntarse, qu distancia habr recorrido cada uno? Solucin:

18

Mara Vir. Martnez Gonzlez

Puesto que la pista de hielo no tiene rozamiento, no actan fuerzas horizontales externas sobre el sistema constituido por ambos patinadores. (Por supuesto, sobre los patinadores actan fuerzas verticales externas: sus pesos y las fuerzas que ejerce la pista sobre ellos. Estas fuerzas verticales se compensan.) Las fuerzas que cada uno de ellos ejerce sobre el otro a travs de la cuerda, cuando tiran de ella, son fuerzas internas al sistema y no afectan al movimiento del centro de masas. Por lo tanto, el centro de masas permanece en reposo mientras los patinadores se acercan. Si consideramos que el centro de masas coincide con el origen de coordenadas, tenemos XCM = 0. Entonces, si xA y xB son las coordenadas iniciales de A y B mB mA xA + mB xB =0 xA = xB mA + mB mA Por otra parte, la distancia inicial entre los patinadores es xB xA = 60 m., que junto con la ecuacin anterior

90 xA = 36 m xA = (60 + xA ) 60 Por lo tanto, cuando ambos patinadores se encuentren en el centro de masas, el patinador A habra recorrido 36 m y el patinador B habr recorrido 24 m. Ejercicio 5.- Es correcto decir que la Tierra describe una rbita alrededor del Sol y la Luna describe una rbita alrededor del centro de la Tierra? Solucin: La Tierra y la Luna constituyen un sistema de dos cuerpos, y el radio vector de su centro de masas es ~ ~ ~ CM = MT RT + ML RL R MT + ML ~ T la masa y el radio vector de la Tierra, respectivamente (y lo mismo para la Luna). Es este centro de siendo MT y R masas el que describe una rbita alrededor del Sol, como lo hara un cuerpo de masa MT + ML sometido a la atraccin gravitatoria del Sol. Por su parte, la Tierra y la Luna describen rbitas en torno a este centro de masas comn. Lo que ocurre es que, debido a que la masa de la Tierra es mucho mayor que la de la Luna, el centro de masas del sistema Tierra-Luna est muy cerca del centro de la Tierra. En efecto, consideremos un sistema de coordenadas ~T y R ~ L son simplemente las distancias ~ CM = 0. Entonces, R cuyo origen coincide con el centro de masas, es decir, R ~ ~ ~ de la Tierra y la Luna, respectivamente, al centro de masas, y RT + RL = d es la distancia entre la Tierra y la Luna. As ~ R ~ T = ML R ~T ~ T + ML R ~L = 0 R ~ L = ML d ~ CM = 0 MT R R MT MT y ~ = 0, 012 384000 km = 4660 km ~ T = (ML /MT ) d R 1 (ML /MT ) 0, 988 que, teniendo en cuenta que (ML /MT ) ' 0, 012 y d = 384.000 km, da RT ' 4.660 km. Puesto que el radio de la Tierra es de unos 6.400 km, vemos que el centro de masas del sistema Tierra-Luna se encuentra dentro de la esfera terrestre. Ejercicio 6.- Una pelota de 60 g se lanza horizontalmente contra la pared de un frontn a 10 m/s y rebota con una velocidad de 8 m/s. (a) Qu impulso se ha transmitido a la pared? (b) Si el contacto dura 0,003 s cul es la fuerza media ejercida por la pared sobre la pelota durante el choque? Solucin: El momento inicial es pi = mvi = 0, 06 kg 10 m/s = 0, 6 kgm/s, y el momento final es pf = mvf = 0, 48 kgm/s, de modo que p = pf pi = 0, 12 kg m/s As, la fuerza media ejercida por la pared sobre la pelota durante el intervalo t es 0, 12 p = N = 40 N F = t 0, 003

da

Momento angular. Fuerzas centrales


De la ecuacin (2) para una partcula puntual se puede deducir la ecuacin

19

Mara Vir. Martnez Gonzlez

siendo i,ext el momentode las fuerzas externas que actan sobre la partcula i. Una vez ms, las fuerzas internas al sistema no afectan al momento angular total del mismo. Una fuerza central es una fuerza que est siempre dirigida hacia un punto dado, que es el centro de fuerzas. ~ =0yL ~ = cte. Asi, si tomamos dicho punto como origen de coordenadas ~ =~ rF Una fuerza que solo depende de la distancia r a un punto necesariamente esta dirigida hacia dicho punto y, por lo tanto, es una fuerza central. V er el razonamiento (y no limitarse a aprenderlo de memoria) en el problema 2.6 del libro de texto. La trayectoria de una partcula sometida a una fuerza central satisface la llamada ley de las reas. V er el problema 3.2 del libro de texto. Ejercicio 7.- En el ejercicio de la pgina 47 del libro de texto se demuestra que un objeto que cae desde una torre de altura h0 situada en el ecuador llega al suelo por delante del pie de la torre. Supongamos ahora una situacin diferente: el objeto se lanza desde el suelo verticalmente hacia arriba con una velocidad tal que alcanza una altura h0 y vuelve a caer. Dnde caer? Solucin: Como en el ejercicio citado, el objeto est sometido a la fuerza gravitatoria dirigida hacia el centro de la Tierra y, por lo tanto, se conserva el momento angular. Pero ahora la distancia inicial al centro es 0 = RT , y la conservacin del momento da 2 2 RT h(t) 2 T = (RT + h(t))2 (t) (t) = T = T 1 RT RT + h(t) RT + h(t) As pues, mientras el objeto est en el aire h(t) > 0 y (t) < T . Es decir, la velocidad angular del objeto en el aire es siempre menor que la velocidad angular de la Tierra. Por lo tanto, el objeto se rezaga continuamente respecto a la Tierra y caer al suelo por detrs del punto desde el que fue lanzado. Tambin ahora podemos hacer una estimacin aproximada siguiendo pasos similares a los del ejercicio del libro. Para empezar, puesto que h(t) RT podemos aproximar 2 2 h(t) h(t) h(t) 1 ' 1 '12 RT + h(t) RT RT Por otra parte, para que el objeto llegue a una altura h0 la velocidad con la que se lanza debe ser v = 2gh0 , y as h(t) = 2gh0 t (1/2)gt2 . En definitiva h(t) 2 2gh0 g 2 = T 1 t+ t (t) = T 1 2 RT RT RT p El objeto volver al suelo al cabo de un tiempo tf = 2 2h0 /g y el ngulo que habr recorrido en ese tiempo ser ! Z tf g t3 2 2gh0 g 2 2 2gh0 t2 f f + = T 1 t+ t dt = T tf RT RT RT 2 RT 3 0 Restndole el ngulo T tf recorrido por un punto de la superfie de la Tierra y multiplicando el ngulo resultante por RT obtenemos ! 1/3 p t2 t3 8 2 h0 f f d = RT ( T ) = T 2 2gh0 + g = 2 3 3 g

~ dL dt ~ el momento de la fuerza respecto al origen de coordenadas y L ~ =~ siendo ~ =~ rF rp ~ el momento angular de la partcula. Asimismo, para un sistema de partculas P ~ d X i Li i,ext = dt i ~ =

Dinmica de rotacin de un slido rgido


Existe un paralelismo entre la dinmica de traslacin de una partcula y la dinmica de rotacin de un slido

20

Mara Vir. Martnez Gonzlez

rgido. En general, podemos pasar de las ecuaciones de la dinmica de traslacin a las de la dinmica de rotacin sin ms que sustituir la velocidad de traslacin por la de rotacin (v ), la fuerza por el momento de la fuerza (F ), el momento lineal por el momento angular (p L) y la masa por el momento de inercia (m I ) Traslacin Rotacin p = mv L = I dv d dp dL =m ext = =I Fext = dt dt dt dt 1 1 mv 2 EkR = I 2 EkT = 2 2 El momento de inercia juega as en la dinmica de rotacin un papel semejante a la masa en la dinmica de traslacin. Cuanto mayor sea el momento de inercia, ms difcil es poner un cuerpo en rotacin. Pero hay que tener en cuenta que el momento de inercia depende no slo de la masa del cuerpo que gira sino de cmo est distribuida dicha masa con respecto al eje de giro. En la ecuacin (3.19) del libro de texto vemos que el momento de inercia de una distribucin discreta de masas respecto a un eje es X mi 2 I= i
i

Las dimensiones del momento de inercia son entonces [I ] = ML2 . As, para un slido regular de masa total M y una dimensin lineal caracterstica L, el momento de inercia ser de la forma cM L2 . Por ejemplo, en el caso de una varilla, L ser la longitud; en el caso de un disco o de una esfera, L ser el radio. El factor c es un factor puramente numrico (sin dimensiones) que depende de cmo est distribuida la masa respecto al eje considerado. Por ejemplo, en el caso de una varilla, si el eje considerado es un eje perpendicular a la varilla y que pasa por un extremo de la misma, la mitad de la masa estar a una distancia menor o igual que L/2, y la otra mitad estar a una distancia mayor que L/2. Sin embargo, si el eje considerado es un eje que pasa por el centro de la varilla, toda la masa estar a una distancia del eje menor o igual que L/2. Por lo tanto, el momento de inercia respecto al eje que pasa por el centro es menor que el momento respecto al eje que pasa por un extremo. No es difcil, en este caso, hacer el clculo detallado. Consideremos, en primer lugar, el momento de inercia la varilla respecto a un eje que pasa por uno de sus extremos. Si la varilla es uniforme, un elemento de la varilla de longitud d tendr una fraccin d/L de la masa total, es decir, dm = M d/L. Entonces Z 1 M L 2 M L3 = M L2 dm = d = (6) Ie = L L 3 3 0 0 Consideremos ahora el momento de inercia respecto a un eje que pasa por el centro de la varilla. Ahora Z L/2 Z L/2 1 M (L/2)3 = M L2 2 dm = 2 2 dm = 2 (7) Ic = L 3 12 L/2 0 Z
L 2

siendo i la distancia de la masa mi al eje. En el caso de una distribucin continua la suma se convierte en una integral y as Z I= 2 dm

De hecho, podramos haber obtenido directamente el resultado (??) a partir de (6) sin necesidad de ms integrales. En efecto, podemos empalmar dos varillas de masa M/2 y longitud L/2 para construir una varilla de masa M y longitud L. Ahora, el punto medio de esta varilla larga coincide con uno de los extremos de cada varilla corta. Entonces el momento de inercia de la varilla de masa M y longitud L respecto a un eje que pasa por el centro ser la suma de los momentos de inercia de dos varillas de masa M/2 y longitud L/2 respecto a un eje que pasa por un extremo. Es decir 2 1 M 1 L M L2 = Ic = 2 3 2 2 12 Es fcil ver que 2 L Ie = Ic + M 2 Este es un caso especial de un resultado ms general conocido como teorema de Steiner, ste afirma que si IG es el momento de inercia con respecto a un eje que pasa por el centro de masas, el momento I con respecto a un

21

Mara Vir. Martnez Gonzlez

segundo eje paralelo al anterior y a una distancia d del mismo es I = IG + M d2 (8) Si aplicamos una fuerza F al extremo de una varilla que gira en torno a un extremo, el momento de la fuerza (suponiendo que la fuerza es perpendicular a la varilla) ser e = F L. Si aplicamos la misma fuerza al extremo de la varilla que gira en torno al centro el momento ser c = F (L/2). La aceleracin angular = d /dt de la varilla, en cada caso, ser e FL 3F c F (L/2) 6F c = = = = = e = Ie (1/3)M L2 ML Ic (1/12)M L2 ML Ejercicio 8.- Sabiendo que el momento de inercia de un disco circular macizo de masa M y radio R respecto a su eje de simetra es I = M R2 /2, calcular el momento de inercia de una corona circular del mismo material. Solucin: Un disco de radio R2 puede considerarse idealmente dividido en un disco de radio R1 y una corona circular de radio interior R1 y radio exterior R2 . El momento de inercia del disco completo puede escribirse Z R2 Z R1 Z R2 2 2 dm = dm + 2 dm = I1 + Icorona I2 =
0 0 R1

es decir, el momento de inercia del disco de radio R2 es igual al momento de inercia de un disco de radio R1 ms el momento de inercia de una corona de radios interior y exterior R1 y R2 respectivamente. Por otra parte, si llamamos M2 a la masa total del disco de radio R2 , la masa del disco de radio R1 ser M1 = M2 (R1 /R2 )2 ; y la masa de la corona ser 2 R1 M2 2 2 R1 Mcorona = M2 M1 = M2 1 2 = 2 R2 R2 R2 Entonces, los momentos de inercia I1 e I2 son, de acuerdo con el enunciado del problema 1 1 1 R4 2 2 I2 = M2 R2 I1 = M1 R1 = M2 1 2 2 2 2 R2 y as 1 1 1 R4 1 4 2 4 = Icorona = I2 I1 = M2 R2 M2 1 = M2 2 R2 R1 2 2 2 R2 2 R2 1 2 1 Mcorona 4 4 2 = Mcorona R1 R2 R1 + R2 = 2 2 2 (R2 R1 ) 2

Ejercicio 9.- Un disco de radio R gira en torno a su eje con velocidad angular . Queremos sustituir el disco por una corona circular (una arandela) que pueda girar a velocidad angular doble pero manteniendo el mismo momento angular. Qu radio debera tener el orificio de la arandela?

Solucin: De la ecuacin L = I se deduce que para que la velocidad angular se duplique, manteniendo el mismo momento angular, el momento de inercia debe dividirse por 2. Sea M la masa del disco y sea r el radio del orificio? Entonces, utilizando la expresin obtenida en el ejercicio anterior 1 1 1 1 R M 2 R4 r 4 = M R2 r = 1/4 Icorona = Idisco 2 2 R 4 2

Fuerzas conservativas y energa potencial. Puntos de equilibrio.


~ (~ El trabajo que realiza una fuerza F r) que acta sobre un cuerpo cuando ste se desplaza entre los puntos A y B es W = Z
rB

Este trabajo depende en general de la trayectoria que ha seguido el cuerpo para ir de A a B . Sin embargo, hay fuerzas para las que el trabajo realizado es independiente de la trayectoria; es decir, cualquiera que sea la trayectoria r), que sigamos, el trabajo es el mismo y solo depende de los puntos inicial y final. Entonces existe una funcin Ep (~ llamada energa potencial, tal que

rA

~ (~ F r) d~ r=

rB

(Fx dx + Fy dy + Fz dz )

rA

22

Mara Vir. Martnez Gonzlez

Fx = o en forma vectorial

Ep , x

Fy =

Ep , y

Fz =

Ep , z

~ = ~ Ep F Es decir, una fuerza conservativa puede escribirse como el gradiente de un campo escalar, que se denomina energa potencial. Entonces W = Ep = Ep (B ) Ep (A) que solo depende del valor de Ep en los puntos A y B. Si nos restringimos a movimientos en una dimensin, por ejemplo a lo largo del eje X, cualquier fuerza F (x) que dependa de la variable x es una fuerza conservativa, pues siempre se puede encontrar una funcin Ep (x) que satisfaga Z dEp (x) Ep (x) = F (x)dx F (x) = dx La energa potencial est definido salvo una constante de integracin. Esto no presenta ningn problema, ya que para calcular el trabajo realizado solo necesitamos diferencias de energa potencial, y en estas diferencias la constante desaparece. En el captulo 2 se ha visto que el gradiente de un campo escalar es cero en los puntos en donde el campo tiene un mximo o un mnimo. Por lo tanto, una fuerza conservativa es cero en los puntos donde la energa potencial tiene un mximo o un mnimo, es decir, en los puntos tales que dEp (x)/dx = 0. Estos son puntos de equilibrio. Sabemos tambin que en las proximidades de un mnimo de un campo escalar, el gradiente es positivo. Por lo tanto, en las proximidades de un mnimo de la energa potencial la fuerza es negativa: apunta hacia el mnimo. As, si el cuerpo se desplaza ligeramente del punto de equilibrio, experimenta una fuerza que tender a devolverlo a l: el punto es un punto de equilibrio estable. Por el contrario, en las proximidades de un mximo de la energa potencial la fuerza es positiva: apunta alejndose del mximo. Si el cuerpo se desplaza ligeramente del punto de equilibrio, la fuerza tender a alejarlo todava ms: el punto es un punto de equilibrio inestable. Todo lo que se ha dicho para movimientos en una dimensin es vlido para movimientos en ms dimensiones si la fuerza es central, es decir, si solo depende de la distancia radial r a un punto.

Conservacin de la energa
Si la nica fuerza que acta sobre un cuerpo es una fuerza conservativa, se debe cumplir Ek = W = Ep o, lo que es lo mismo [Ek (x) + Ep (x)] = 0 Ek (x) + Ep (x) = ET otal = constante Puesto que la energa cintica es necesariamente positiva (o nula), debe satisfacerse siempre ET otal > Ep (x) La regin que comprende los valores de x que satisfacen esta condicin es la regin permitida para el movimiento del cuerpo. Por el contrario, el cuerpo nunca puede estar en una posicin que viole la condicin anterior. El punto para el que ET otal = Ep (x) separa las regiones permitida o prohibida y se llama punto de retorno, pues en estos puntos la energa cintica (y con ello la velocidad) es cero. El cuerpo que se acerca al punto de retorno desde la zona permitida se frena hasta detenerse en el punto de retorno e invierte su movimiento. Por otra parte, es evidente que la energa cintica tendr un mximo cuando la energa potencial tenga un mnimo, es decir, en los puntos de equilibrio estable. Recprocamente, la energa cintica tendr un mnimo cuando la energa potencial tenga un mxima, es decir , en los puntos de equilibrio inestable (siempre que estos puntos se encuentren en la regin permitida). Ejercicio 10.- Un cuerpo que se mueve a lo largo del eje X est sometido a un potencial Ep (x) = ax2 bx4 , siendo a y b constantes positivas a) Qu dimensiones tienen las constantes a y b? b) Qu fuerza acta sobre el cuerpo en cada punto?

23

Mara Vir. Martnez Gonzlez

c) Cules son los puntos de equilibrio estable e inestable? d) Que trabajo se realiza cuando el cuerpo se desplaza entre los dos puntos inestables. Solucin: a) La energa potencial tiene dimensiones de energa, ML2 T2 . Entonces ML2 T2 = [a] L2 = [b] L4 de modo que [b] = ML2 T2 [a] = MT2 b) La fuerza es la derivada (con signo negativo) del potencial. En este caso d(ax2 bx4 ) dEp (x) = = 2ax + 4bx3 F (x) = dx dx c) Los puntos de equilibrio son aquellos para los que se anula la fuerza, es decir 0p 3 2ax + 4bx = 0 xeq = a/2b Para ver cules son estables y cules inestables, calculamos la derivada segunda del potencial d2 Ep (x) = 2a 12bx2 dx2 2 2 Para x = 0, tenemos p d Ep (x)/dx = 2a > 0, y por lo tanto este punto es un punto de equilibrio estable. Por el contrario, para x = a/2b se tiene 2 a d Ep (x) = 2a 12b = 4a < 0 dx2 2 b x= a/2b p y as los puntos simtricos x = a/2b son inestables. d) La energa potencial es una funcin par. Por lo tanto, tiene el mismo valor en los dos puntos inestables. As pues, el trabajo total realizado cuando el cuerpo se mueve de un punto inestable a otro es cero. Ejercicio 11.- Sobre un cuerpo que se mueve a lo largo del eje X acta una fuerza F (x) = a bx2 , siendo a y b constantes positivas a) Qu dimensiones tienen las constantes a y b? b) De qu potencial deriva esta fuerza? c) Cules son los puntos de equilibrio estable e inestable? d) Qu trabajo realiza la fuerza cuando el cuerpo se mueve de un punto de equilibrio a otro? Solucin a) La dimensin de la fuerza es [F ] =MLT2 y as MLT2 = [a] = [b] L2 b) El potencial es Ep (x) =

[b] = ML1 T2

c) Los puntos de equilibrio son p a bx2 = 0 xeq = a/b Adems p d2 Ep (x) +2ab > 0 si x = +pa/b = 2bx = 2ab < 0 dx2 si x = a/b p p Por lo tanto, el punto xeq = + a/b es estable y el punto xeq = a/b es inestable. d) El trabajo realizado cuando el cuerpo se mueve del punto estable al inestable es 1/2 4 b b 1/2 3/2 1/2 3/2 = a3 /b W = a(a/b) + (a/b) a(a/b) (a/b) 3 3 3 Ntese que esta combinacin de las constantes a y b tiene, como debe ser, dimensiones de energa 3/2 h i MLT2 3/2 1/2 a /b = ML2 T2 = 1/2 (ML1 T2 )

b (a bx2 )dx = ax + x3 + C 3

24

Mara Vir. Martnez Gonzlez

Ejercicio 12.- Un potencial muy utilizado en el estudio de interacciones entre tomos y superficies es el potencial de Lennard-Jones a b Ep (x) = 12 6 x x Supongamos que desde un punto muy alejado de la superficie (prcticamente a una distancia infinita) se lanza un tomo de masa m con una velocidad inicial vi . a) Cul ser la mxima velocidad que puede alcanzar el tomo en su camino hacia la superficie? A qu distancia de la superficie alcanzar dicha velocidad mxima? b) Cul ser la distancia ms prxima a la superficie a la que puede llegar el tomo? Solucin: Por la conservacin de la energa 1 a b mv 2 (x) + 12 6 = cte 2 x x Muy lejos de la superficie (x = ) la energa potencial es cero, de modo que la energa total vale 1 2 ET otal = mvi 2 y puesto que la energa total se conserva, obtenemos que para una distancia x cualquiera a b 1 1 2 mv 2 (x) + 12 6 = mvi 2 x x 2 o 1 1 a b 2 mv 2 (x) = mvi 12 + 6 2 2 x x Evidentemente, la energa cintica ser mxima cuando la energa potencial sea mnima. Sabemos que esto corresponde a un punto de equilibrio xeq . La energa potencial tiene un mnimo en 1/6 12a 6b 2a dEp = 13 + 7 = 0 xeq = dx x x b y para este valor de x el valor de Ep es 2 b2 b b = b Ep (xeq ) = a 2a 2a 4a Entonces el valor mximo de la energa cintica ser 1 b2 2 Ek,max = mvi + 2 4a y as 1 b2 b2 1 2 2 2 2 mvmax vmax = mvi + = vi + 2 2 4a 2ma La distancia ms prxima ser aquella para la que la velocidad se haga cero. Es decir a b 1 mv 2 = 12 6 2 i x x Llamando x6 = y, esta ecuacin se convierte en una ecuacin de segundo grado 1 mv 2 y 2 + by a = 0 2 i con solucin p 2 b b2 + 2amvi y= 2 mvi Es decir, el tomo que se acerca a la superficie va ganando velocidad hasta quellega al punto xeq , luego empieza a frenarse y finalmente retrocede en el punto x = y 1/6 . Ek + Ep = Ejercicio 13.- Un pndulo de longitud l = 3 m tiene una lenteja de masa m = 2 kg. Cuando el pndulo est en reposo en la posicin vertical se golpea horizontalmente la lenteja para que alcance una velocidad de 5,5 m/s. (a) Qu ngulo mximo respecto a la vertical alcanzar el pndulo? (b) Calcular la velocidad de la lenteja y su energa potencial cuando forma un ngulo de 30o con la vertical. Solucin: a) Tomaremos el origen de alturas en el punto de equilibrio de la lenteja. Entonces, cuando el pndulo este inclinado un ngulo , la lenteja estar a una altura h = l(1 cos ); en este situacin la energa potencial ser

25

Mara Vir. Martnez Gonzlez

Ep = mgh = mgl(1 cos ), y la energa total del pndulo ser 1 ET otal = mv 2 () + mgl(1 cos ) 2 Inicialmente i = 0 y vi = 5,5 m/s, de modo que la conservacin de la energa da 1 1 2 mv 2 () + mgl(1 cos ) = mvi 2 2 y as q 2 2gl(1 cos ) v () = vi Cuando el pndulo alcance su ngulo mximo, la velocidad ser cero, y as v2 2 vi 2gl(1 cos max ) = 0 cos max = 1 i 2gl Con los valores dados obtenemos 1 5, 52 m2 /s2 ' max = 60o cos max = 1 23m 2 10 m/s 2 b) Para = 30o , cos 30o = 3/2. Entonces Ep = 2 kg 10 m/s2 3 m 1 3/2 = 8, 04 J r v = 5, 52 2 10 3 1 3/2 m/s = 4, 68 m/s

Energa de rotacin y conservacin de la energa


Consideremos un slido rgido que rota con velocidad angular alrededor de un eje. (Por ejemplo, un disco que gira alrededor de su eje central). Si, adems, el slido se esta desplazando de modo que su eje lleva una velocidad ~ v, la velocidad de cada partcula del slido se puede descomponer como la suma (vectorial) de la velocidad de traslacin v del eje ms una velocidad de rotacin en torno al mismo. La energa cintica total es entonces 1 1 (9) Ek = M v 2 + I 2 2 2 Si el slido se mueve bajo la accin de una fuerza conservativa, que deriva de un potencial, la variacin de la energa potencial ser igual (y de sentido contrario) a la variacin de la energa cintica total, es decir, a la suma de las variaciones de las energas cinticas de traslacin y de rotacin. Consideremos, por ejemplo, un cilindro de radio R que cae por un plano inclinado desde una altura h. Si no hay rozamiento entre el cilindro y el plano, el cilindro deslizar libremente sin rotar; es decir, el punto de contacto entre el cilindro y el plano inclinado es siempre el mismo. Entonces slo hay energa cintica de traslacin y la velocidad vc del centro de masas del cilindro al llegar al final del plano inclinado vendr dada por p 1 2 Ep = Ek M gh = M vc vc = 2gh 2 Sin embargo, si hay rozamiento entre el cilindro y el plano, el cilindro no puede deslizar libremente y empezar a rotar. Si el rozamiento es pequeo, el movimiento ms general es una combinacin de rotacin y deslizamiento. Cuando el rozamiento es lo suficientemente grande para impedir el deslizamiento, se produce un movimiento de rodadura sin deslizamiento. En este caso hay una relacin simple entre la velocidad de rotacin y la velocidad de traslacin del eje del cilindro. En efecto, cuando el cilindro haya rotado un ngulo , el eje se habr desplazado una distancia s = R. Entonces ds/dt = R(d/dt); es decir v = R . Por otra parte, al no haber deslizamiento la fuerza de rozamiento no realiza trabajo pues solo acta instantneamente sobre el punto de contacto, que se renueva continuamente. Por ello, sigue siendo aplicable el principio de conservacin de la energa (incluyendo ahora las energas cinticas de traslacin y rotacin). Ahora la velocidad vc cuando el cilindro llega abajo satisface 1 1 1 1 1 v2 2 2 2 M gh = M vc + I 2 = M vc + M R2 c = M (1 + )vc 2 2 2 2 2 R 2 y as s 2gh vc = 1+ As, la velocidad de caida de un cilindro que rueda es siempre menor que la de un cilindro que desliza.

26

Mara Vir. Martnez Gonzlez

En el caso concreto de un cilindro hueco de masa M y radio R es claro que el momento de inercia es I = M R2 puesto que toda la masa est a una distancia R del eje. En este caso = 1 y vc = gh. Para un cilindro macizo de la misma masa < 1 (el valor concreto depender de cmo vare la densidad en su interior) y la velocidad de cada ser mayor.

Choques: una aplicacin sencilla de las leyes de conservacin


Un ejemplo importante de las leyes de conservacin se da en los choques entre partculas. Consideremos el u1 y ~ u2 a las velocidades antes choque entre dos partculas de masas m1 y m2 . Para simplificar la notacin llamremos ~ v2 a las velocidades despus del choque. del choque, y ~ v1 y ~ Si sobre las partculas no actan fuerzas externas, el momento lineal total debe conservarse, es decir, u1 + m2 ~ u2 = m1~ v1 + m2~ v2 (10) m1 ~ Si adems, las partculas son rgidas y la nica interaccin entre ellas se produce enel instante del choque, no se realizar ningn trabajo y, por lo tanto, se conservar tambin la energa total, que ahora es solo energa cintica. Se dice entonces que el choque es elstico 1 1 1 1 2 2 m1 ~ u2 u2 v1 + m2~ v2 (11) 1 + m2 ~ 2 = m1~ 2 2 2 2 Estas dos ecuaciones se pueden reordenar de la forma m1 (~ u1 ~ v1 ) = m2 (~ u2 ~ v2 ) 2 2 2 2 u1 ~ = m2 ~ u2 ~ m1 ~ v1 v2 y dividiendo la segunda por la primera se obtiene v1 = ~ u2 + ~ v2 o ~ v1 ~ v2 = (~ u1 ~ u2 ) ~ u1 + ~ que nos da un resultado importante: en un choque elstico la velocidad relativa entre las partculas cambia de signo, pero su mdulo se mantiene. Ahora es fcil escribir las velocidades finales en funcin de las velocidades iniciales. Por ejemplo, de la ltima v1 + ~ u1 ~ u2 , que introducido en (10) da expresin tenemos ~ v2 = ~ m1 m2 2m2 ~ v1 = ~ u1 + ~ u2 m1 + m2 m1 + m2 y analogamente m2 m1 2m1 ~ v2 = ~ u2 + ~ u1 m1 + m2 m1 + m2 Podemos estudiar ahora algunos casos particulares. Por ejemplo, si las masas de las partculas son iguales, m1 = m2 , las ecuaciones se reducen a u2 y ~ v2 = ~ u1 ~ v1 = ~ es decir, las partculas simplemente intercambian sus velocidades. Otro caso particular es cuando una de las partculas tiene una masa mucho mayor que otra. Por ejemplo, si m1 /m2 ' 0 u1 + 2~ u2 ~ v1 = ~ es decir, la partcula de masa muy grande conserva su velocidad. ~ v2 = ~ u2

Ejercicio 14.- Dos cuerpos (a los que designaremos mediante los ndices 2 y 3) de masa m = 1 kg se encuentran inicialmente en reposo en las posiciones x2 (0) = 0 y x3 (0) = 1 m, respectivamente. En el intante t = 0, otro cuerpo (que designaremos con el subndice 1) de masa M = 3m, y que se mueve en la direccin del eje X con velocidad v = 1 m/s, colisiona elsticamente con el cuerpo situado en el origen. Cules sern las velocidades de los 3 cuerpos para t > 2 s? Solucin: En el instante = 0 colisionan 1 y 2 en el origen de coordenadas, de modo que x1 (0) = x2 (0) = 0. Teniendo en cuenta que 2 estaba en reposo antes de la colisin, sus velocidades inmediatamente despus de la colisin sern, de acuerdo con (??) y (??) v 3 3m m 6m v = = 0, 5 m/s v2 (0) = v = v = 1, 5 m/s v1 (0) = 3m + m 2 4m 2

27

Mara Vir. Martnez Gonzlez

Con esta velocidad v2 (0) = 3/2 m/s el cuerpo 2 tardar 2/3 = 0, 66 segundos en recorrer la distancia que le separa del cuerpo 3, que sigue en reposo en x3 = 1 m. Ahora solo hay que tener en cuenta que cuando colisionan dos cuerpos de la misma masa, intercambian sus velocidades, es decir, el cuerpo 2 queda en reposo y el cuerpo 3 sale con la velocidad que llevaba el cuerpo 2. As, inmediatamente despus de la colisin en el instante t = 2/3, tenemos v2 (t = 2/3) = 0 v3 (t = 2/3) = 1, 5 m/s x2 (t = 2/3) = x3 (t = 2/3) = 1 m Ahora, el cuerpo 1 que llevaba una velocidad de 0,5 m/s tras la primera colisin, tarda 2 segundos en llegar al punto x = 1 m donde ha quedado en reposo el cuerpo 2. Tras est nueva colisin entre 1 y 2, las nuevas velocidades sern 0, 5 3 = 0,25 m/s v2 (t = 2) = 0, 5 = 0, 75 m/s v1 (t = 2) = 2 2 A partir de este momento ya no hay ms colisiones, de modo que las velocidades finales son v1 = 0, 25 m/s, v2 = 0, 75 m/s y v3 = 1, 5 m/s Podemos comprobar que en el proceso total se han conservado el momento y la energa. En efecto, el momento inicial era Pini = M v = 3 kgm/s. y el momento final es Pf in = 3mv1 + mv2 + mv3 = 3 0, 25 + 1 0, 75 + 1 1, 5 = 3 kg m/s Asimismo, la energa inicial es Ei = (1/2)M v 2 = 3/2 julios, y la energa final 1 3 0, 252 + 1 0, 752 + 1 1, 52 = 3/2 julios Ef in = 2

Rozamiento
Ejercicio 14: Si los frenos de un automvil producen una deceleracin de a = 4 m/s2 , cunto tardar en llegar al reposo un automvil que circula a 36 Km/h? Que distancia habr recorrido en la frenada? Solucin: Este problema es el mismo que el problema 2 del material complementario del captulo 2. All se resolva aplicando ecuaciones puramente cinemticas. Ahora vamos a resolverlo a partir de ecuaciones dinmicas. Para que el automvil frene con una aceleracin a = 4 m/s, sobre el mismo debe actuar una fuerza de rozamiento F = ma. Si en la frenada se recorre una distancia s, la fuerza habr realizado un trabajo W = F s = mas. Este trabajo es igual a la variacin de energa cintica. Le energa cintica inicial es Ei = (1/2)mv 2 y la energa cintica final es cero. Entonces 1 mv 2 = mas 2 Asimismo, de la segunda ley de Newton Ecin = W p = F t s= 102 m2 /s2 v2 = = 12, 5 m 2a 2 4 m/s2 10 m/s v = = 2, 5 s a 4 m/s2

mv = mat

t =

Ejercicio 15.- El coeficiente de rozamiento entre el suelo de un camin y una caja es = 0,30. El camin circula a 72 km/h por una carretera horizontal. Cul es la mnima distancia de frenada del camin para que la caja no deslice? Solucin: Si la caja no desliza, su aceleracin ser la misma que la del camin, a, y por lo tanto sobre la caja debe actuar una fuerza F = M a (siendo M la masa de la caja). La nica fuerza horizontal que acta sobre la caja es el rozamiento con el suelo del camin, que puede alcanzar un valor mximo Fr = M g. Por ello, la caja no deslizar mientras la aceleracin del camin sea menor que ac = Fr /M = g. Para esta deceleracin crtica, la distancia recorrida sera 202 m2 /s2 v2 = ' 68 m s= 2g 0, 6 10 m/s2 Ejercicio 16.- Una conocida atraccin de feria consiste en una habitacin circular que gira en torno a un eje vertical y donde los participantes se mantienen pegados a la pared, sin caer al suelo, por efecto del rozamiento. (a) Si el coeficiente de rozamiento con la pared tiene el valor = 0,4 y el radio de la habitacin es R = 2 m, con qu velocidad angular mnima min tendra que girar la habitacin para que los participantes no caigan? (b) Supongamos ahora que el motor de la mquina se estropea y slo puede proporcionar una velocidad = min /2. Cunto tardara un participante que inicialmente estaba a 4 m de altura en llegar al suelo? Solucin:

28

Mara Vir. Martnez Gonzlez

Un participante de masa m que gira, pegado a la pared, con velocidad angular tiene una aceleracin centrpeta ac = 2 R. Para ello, la pared debe ejercer sobre el participante una fuerza normal FN = mac = m 2 R dirigida hacia el eje de la habitacin (que es el eje de giro). Puesto que la pared tiene rozamiento, ejercer tambin sobre el participante una fuerza de rozamiento vertical FR = FN = 2 R. Para que el participante no caiga, esta fuerza de rozamiento debe ser igual al peso del mismo, es decir, la min debe satisfacer 9, 8 m s2 g g 2 = = 12, 25 s2 FR = m 2 2 min R = mg min R = min = R 0, 4 2 m Ntese la similitud entre este problema y el problema 6 del captulo 3 del libro de texto. La diferencia es que ahora la aceleracin centrpeta 2 R ocupa el lugar de la aceleracin lineal. Supongamos ahora que la habitacin solo gira con min /2. Entonces la fuerza de rozamiento es FR = / m( 2 min 4)R = m(g/4R)R = mg/4, y la fuerza vertical total que acta sobre el participante es mg (mg/4) = mg (1 1/4), de modo que el participante caer con una aceleracin 1 a=g 1 4 y caer una altura h = 4 m en un tiempo v v r u 2h u 8m u 2h u = 1, 48 s t= =t =t 1 a g 1 4 9, 8 m/s2 1 11 ,6

Friccin en fluidos
Ejercicio 17.- Desde el suelo lanzamos verticalmente hacia arriba una bola con velocidad inicial vi . Sobre ~r = k~ v . La bola alcanza una la bola acta la gravedad y la resistencia del aire, que es proporcional a la velocidad F altura hmax y cae de nuevo al suelo, al que llega con una velocidad final vf . a) Cul es mayor, la velocidad final vf o la velocidad inicial vi ? b) Cul es mayor, el tiempo de subida o el de bajada? Solucin: Aunque no es estrictamente necesario para responder a las cuestiones planteadas, vamos a explicar en detalle cul es el movimiento exacto de la bola. La ecuacin de movimiento, de acuerdo con la segunda ley de Newton, es dv = mg kv m dt con la condicin v (t = 0) = vi . (Nos ahorramos la notacin vectorial porque el movimiento es unidimensional.) En el material del captulo 1 vimos una ecuacin similar, aunque all no haba trmino constante mg . Vimos entonces que m/k tiene dimensiones de tiempo, y podemos definir una constante de tiempo m/k. Es fcil comprobar que la solucin es o v = (m/k )g [1 exp ((k/m)t)] + vi exp ((k/m)t) = g [1 exp (t/ )] + vi exp (t/ )

v = g + (vi g ) exp (t/ ) As, para t = 0, tenemos v (0) = vi , la velocidad inicial. Y para t tenemos v g = mg/k. sta es la velocidad lmite. Para esta velocidad el segundo miembro es cero y, por lo tanto, tambin lo es la aceleracin. Podemos ver tambin que para t = tenemos 1 v = g + (vi g ) e As pues, durante el movimiento de subida la velocidad es positiva y va frenndose hasta hacerse cero. Esto ocurre para un tiempo t0 tal que vi g vi = ln 1 g + (vi g ) exp (t0 / ) = 0 t0 = ln g g A partir de ese instante la bola empieza a caer. Como hemos visto, si la bola continuara cayendo durante mucho tiempo su velocidad se aproximara a la velocidad lmite vlim = g. Sin embargo, la bola llegar al suelo antes de haber alcanzado dicha velocidad lmite. a) De hecho, no hace ninguna falta conocer el movimiento exacto de la bola para responder a la pregunta. Basta con considerar el trabajo total realizado por las fuerzas que actan sobre la bola. La fuerza gravitatoria es una

29

Mara Vir. Martnez Gonzlez

fuerza conservativa y, por lo tanto, el trabajo total que ha realizado cuando la bola vuelve al punto de partida es cero. Dicho de otra forma, el trabajo mghmax (negativo) que ha realizado durante el trayecto de subida se cancela con el trabajo mghmax (positivo) realizado durante la bajada. Es decir, Wgravedad = 0. Por su parte, el trabajo realizado por la fuerza de resistencia del aire es siempre negativo, ya que la fuerza tiene siempre sentido contrario al movimiento. En definitiva, tenemos Ek (f inal) Ek (inicial) = Wgravedad + Wrozamiento < 0 y por lo tanto vf < vi . b) El mismo razonamiento del apartado a) muestra que la velocidad de la bola cuando pasa por una altura h en su movimiento ascendente es mayor que la velocidad de la bola cuando pasa por la misma altura en su movimiento descendente. Es decir, la bola al bajar pasa por los mismos puntos por los que pas al subir pero con menor velocidad. Por lo tanto, el tiempo total de bajada debe ser mayor que el de subida. Ejercicio 18.- La resistencia que impone un fluido al movimiento de un objeto es F = kv, siendo k una constante. En el seno de dicho fuido se deja caer, partiendo del reposo, una bola de masa m = 10 g. Al cabo de cierto tiempo tc se observa que la bola ha recorrido 1 m en 1 segundo. Cunto vale la constante k? Cunto debe valer tc para que podamos decir que la velocidad es aproximadamente constante? (Tomar, por simplicidad, |g | = 10 m/s2 ) Solucin: Como hemos visto en el Problema 13, la ecuacin de movimiento para la bola es dv = mg kv m dt y la solucin es mg mg (1 exp (tk/m)) (1 exp (t/ )) v= k k Para un tiempo tc / , el trmino exp(t/ ) tiende a cero y se alcanza la velocidad lmite vlim = mg/k. As, si suponemos que ya se ha alcanzado este rgimen de cada al medir la velocidad de la bola, tenemos mg 0, 01 kg 10 m/s2 mg = 1 m/s k = = = 0, 1 N/m vlim = k 1 m/s 1 m/s Sin embargo, para poder aceptar este valor de k tendramos que estar seguros de que ha pasado un tiempo tc suficiente para poder considerar que estamos muy cerca de la velocidad lmite; por ejemplo, que v > 0, 95vlim . Para ello bastara que tc = 3 , pues exp(3 / ) = e3 ' 1/20 y 1 19 mg 1 = vlim = 0, 95vlim v (t = 3 ) = k 20 20 Por lo tanto basta con esperar un tiempo 0, 03 kg m = 0, 3 s tc = 3 = k 0, 1 N/m Ejercicio 19.- Supongamos que en el mismo fluido dejamos caer una bola del mismo material que la anterior pero de un radio doble. Cul sera ahora la velocidad lmite? Cunto tiempo habra que esperar ahora para alcanzar una velocidad mayor que un 95% de dicha velocidad? Solucin: Este problema es un simple recordatorio de una ley de escala que vimos en el material complementario del captulo 1. All se deca que, para cuerpos esfricos, la constante k es proporcional al radio R. Y puesto que m es proporcional al volumen, y por lo tanto a R3 , tenemos m R2 k Por lo tanto, la velocidad lmite de un bola de radio doble ser 22 = 4 veces mayor que la de la primera bola, y tambin tc ser 4 veces mayor.

30

Mara Vir. Martnez Gonzlez

Bloque Temtico 2: Vibraciones y Ondas


Las oscilaciones son un fenmeno ubicuo en la naturaleza. Todo sistema en equilibro se encuentra en un mnimo de energa potencial. Si el sistema se aparta ligeramente de dicho punto, realizar oscilaciones en torno al mismo. Adems, si el sistema se encuentra suficientemente prximo al punto de equilibrio, las oscilaciones sern las correspondientes a un movimiento armnico simple; en efecto, cerca de un punto de equilibrio la curva de energa potencial puede aproximarse por una parbola vertical, y esta es precisamente la forma del potencial armnico. En resumen, todo sistema alejado ligeramente de su posicin de equilibrio realiza un movimiento armnico con una frecuencia natural que es propia del sistema. No obstante, normalmente el sistema est sometido a un rozamiento que amortigua las oscilaciones y llevan al sistema al reposo. Sin embargo, el sistema puede estar sometido tambin a fuerzas externas que le aportan energa. Especialmente importantes son las fuerzas externas peridicas, pues en este caso, si se dan las condiciones apropiadas, el sistema puede absorber mucha energa del agente que produce la fuerza externa. Este es el fenmeno de resonancia, por el que el sistema puede alcanzar amplitudes de oscilacin enormes aunque la fuerza externa que lo perturba sea muy pequea. Cuando una oscilacin se produce en un medio continuo, el movimiento en una regin afecta a las regiones vecinas y la perturbacin que ello produce se propaga en el espacio. Las regiones contiguas oscilan con la misma frecuencia, pero las oscilaciones en una regin estn desfasadas con respecto a las oscilaciones en las regiones contiguas. Obtenemos as una onda. La velocidad de propagacin de la onda depende de las propiedades del medio en que se propaga. Para cada onda existe una relacin concreta entre la frecuencia de las oscilaciones, la longitud de onda y la velocidad de propagacin. Cada punto del espacio realiza una oscilacin alrededor del equilibrio. Si las oscilaciones en cada punto son perpendiculares a la direccin de propagacin de la onda, tenemos una onda transversal, como son las ondas en una cuerda vibrante. Si las oscilaciones en cada punto tienen lugar a lo largo de la direccin de movimiento de la onda, tenemos una onda longitudinal, como son las ondas sonoras. El medio que oscila no se desplaza en promedio; por ejemplo, cuando pasa una ola el agua realiza un movimiento peridico ascendente y descendente pero no hay un desplazamiento global del agua en una direccin. Sin embargo, la onda si que viaja y puede transmitir momento lineal y energa. Por otra parte, la intensidad de las ondas depende de la dimensionalidad del espacio en que se mueven. Una onda que se propaga por una cuerda en una nica direccin solo transporta energa a lo largo de dicha direccin y la onda se propaga sin que disminuya su intensidad. Sin embargo, la energa de una onda que se propaga en un medio bidimensional (por ejemplo, una membrana) o tridimensional (un slido) se reparte entre las 2 o 3 dimensiones y su intensidad decrece a medida que la onda se aleja del origen. Cuando varias ondas de la misma frecuencia se propagan en la misma regin del espacio pueden interferir y dar lugar a una onda suma. La onda suma resultante depender de la fase relativa de las ondas que interfieren y puede dar lugar a interferencia constructiva (una suma total de las amplitudes), destructiva (una cancelacin total) o cualquier caso intermedio. Un caso especial de interferencia es la de dos ondas de la misma frecuencia que viajan en direcciones opuestas. Esto sucede, por ejemplo, cuando una onda viajera se propaga en una regin limitada del espacio: la onda rebota en un extremo y vuelve en la direccin contraria, con lo que puede interferir con la onda viajera que sigue llegando desde la direccin inicial. La suma de estas ondas viajeras da lugar a una onda estacionaria. Ahora, todas las regiones del medio oscilan con la misma fase, aunque la amplitud vara de una regin a otra, e incluso puede hacerse nula en algunos puntos. Un tipo de onda especialmente importante son las ondas armnicas. Su importancia reside, aparte de su sencillez, en que cualquier onda puede describirse o construirse como una superposicin de ondas armnicas. Las ondas armnicas caractersticas de un sistema son los modos normales de dicho sistema. El anlisis de una onda en modos normales es fundamental para estudiar dicho sistema.
31 Mara Vir. Martnez Gonzlez

x(t)

x(t) = x(t + T ).


x(t) = A cos(t + ),

x(t) A (t + ) t = 0

v=

dx = A sen(t + ) dt dv = 2 A cos(t + ) dt


a=

2 rad = 360 s1 rad/s s = 1/T Hz

32

Mara Vir. Martnez Gonzlez


sen2 (t + ) + cos2 (t + ) = 1

a = 2 A cos(t + ) = 2 x

sen(t + ) =

1 cos2 (t + )

v = A sen(t + ) = A2 x2



A cos(t + ) = A cos( (t + T ) + )

x(t) = x(t + T )


t + = (t + T ) + + 2n,

n = 0, 1, 2, .. n = 0 T = 0 n = 1 T = 2

n n


F = ma = m 2 x = kx

k = m 2 k k k = mg/l

33

Mara Vir. Martnez Gonzlez

m = 0,1 kg T = 1 s
t=0

x = 0,4 m v = 20 m/s

= 2 = 2 s1 T


x(t = 0) = A cos = 0, 4 v (t = 0) = A sen = 20 1 20 = 7,96 2 0, 4


tan =

= arc tg(7,96) = 1,4 rad


A=

x0 0, 4 = = 3, 2 m cos cos 1, 4

k = m 2 = 3, 95 N/m

m l

m P T FT = mg sen R O
m d2 s = mg sen , dt2

s = l
m d2 mg = mg = s. 2 dt l

k = mg/l

34

Mara Vir. Martnez Gonzlez

s
g d2 = . dt2 l

(t) = 0 sen t 2 = g/l


T = 2 l . g

T s FT mg N

m l

m h l


E = mL g = V g = l2 hg.

P = mg

35

Mara Vir. Martnez Gonzlez

E = P
mg = l2 h0 g

h0 =

mg l2 g

x = h h0
E P = l2 (h0 + x)g mg = gl2 x.


m d2 x = gl2 x dt2


T = 2 m . gl2


d2 x + Cx = 0 dt2

C
X = A sen t
T =

2 = 2

1 C

1 A2 m 2 A2 K 1 Ek = mv 2 = m[A sen(t + )]2 = sen2 (t + ) = sen2 (t + ) 2 2 2 2

x
x x

EP =
0

F dx =
0

1 1 kx dx = kx2 = kA2 cos2 (t + ) 2 2

36

Mara Vir. Martnez Gonzlez


1 E = Ek + Ep = kA2 2

sen2 (t + ) + cos2 (t + ) = 1.


1 kg

k = 100 N/m x(t) x



=


x(t) = A cos(t + )
k = 10 s1 m


sen = 0


= =0

v (0) = A sen( 0 + ) = 0

x(0) = A cos 0 = A = 0, 02 m.

Ek

Ep

1 1 1 1 E = EP + Ek = kx2 + mv 2 = kA2 (cos2 t + sen2 t) = kA2 2 2 2 2

E = Ep + Ek = 2Ek = 2Ep
1 2 1 2 kx = kA , 2 4

v = dx/dt = A sen t

x = A/ 2 = 0, 014

m = 2 km

4 k = 10 N/m 3m

37

Mara Vir. Martnez Gonzlez


x = 3 x
W =
0

x = 0

1 kxdx = kx2 = 4, 5 104 . 2


E = Ep + Ek = 4, 5 104

x=1

1 1 E = kx2 + mv 2 = 4, 5 104 . 2 2


v= 9 104 104 = 200 m/s 2

d2 x dx m 2 = kx b dt dt


= b/m

x = Ae/t cos(t + )


k 2 = m 4
2 0

2 . 4

2 0 >

2 4
1 E = m 2 A2 = 2 1 t m 2 A2 = E0 et 0 e 2


Q= 0

E 2 = E Q

38

Mara Vir. Martnez Gonzlez

T = 2 s 2 m Q


t1 = 10T = 10(2 )/ t = 20 2
2 2 = 0

A(t) = A0 et/2 ,

A0 = 2 A(t1 ) = 2e10/ = 1


10 = . ln(1/2)


Q= 0 = 2 1 + = 14, 4 2 4


E A E = m 2 A2 /2 = A2 /2 = m 2
2 K = m0

A A = A A

1 E = (A A)2 = [A2 2AA + (A)2 ] [A2 2AA] = E AA, 2 2 2

(A)2 (A)2 << A E


E = E E

39

Mara Vir. Martnez Gonzlez


E = AA

E A =2 = 0, 04. E A

Q = 200

E 2 = = = 3, 14 % E Q 100

40

Mara Vir. Martnez Gonzlez


y (x, t) = A cos k (x vt)

k = 2/
0, 001 m

1 m1 3 m/s

y (x, t) = A cos k (x + vt) = 0, 001 cos(x + 3t)


v= = = /2 T

= 2/k = 6, 3 m

= 2 =


3 s1

= v/ = 0, 48 Hz

x t y (x, t)
v (x, t) = dy (x, t) = Akv sen k (x + vt) dt

vM = Akv = 0, 003 m/s


y (x, t) = A cos k (x vt)

41 Mara Vir. Martnez Gonzlez

v = /T = /k t) = A cos(kx t) k

y (x, t) = A cos k (x vt) = A cos k (x

= 2/T k = 2/
t x y (x, t) = A cos k (x vt) = A cos 2 ( ) T

T
v= T ,

v=

Y
Y ,

M
v= RT , M

50 Hz 1 cm 10 m 1 kg 1 s

= 50 v = 10 = v/ = 0, 2
y (x, t) = A sen(kx t) = A sen 2 x t = 102 sen 2 (5x 50t)

42

Mara Vir. Martnez Gonzlez


v=


T .

= 1/10 kg/m
T = v 2 = 10 N.

= 40 5 cm

Hz

/6 0, 005 s 0, 01 kg/m

x t

x1 x2
k x = k=

sen (kx1 t) = sen (kx2 t +)

1 = x 30

= 2/k = 60 cm x
sen (kx t1 ) = sen (kx t2 + )

t1 t2 = t1 + t
= t = 2 t = 0, 4

2 = = = 60 cm 40 T k
1

= 24 m/s


T = 2 = 0, 01

T /

576 2 2 = 5, 76

x1 = 0 y1 = 0, 2 sen 3t x2 = 1 y2 = 0, 2 sen (3t + /8)

y (x, t) = A sen

2 2 (x vt) = A sen ( x t).

43

Mara Vir. Martnez Gonzlez

x = 0
y (0, t) = A sen 2 vt = 0, 2 sen 3t L

A = 0, 2 m v = 3/2 m/s = /2 = 3/2 Hz x = 1


y (1, t) = A sen 2 (1 vt) = 0, 2 sen (3t + /8). L

2/ = /8 v = = 24 m/s

= 16 m


2 (x + vt).

y (x, t) = A sen

k = 2/ = 2, 2 1 = 2 v = /T = = 1 2v/ =
y (x, t) = A sen(kx t) = 0, 003 sen ( 2 x t) 1, 43

v (t) = 0, 003 = 0, 0094 1


dy = A cos(kx t) dt

vm = A =

I Pm = dEm /dt

44

Mara Vir. Martnez Gonzlez

r
I=

Pm Pm S

S = 4r2 r
L = 10 log I I0

I0 = 1012 W/m2

Pm

r
I= Pm Pm = . S 4r 2


I I0

60 = 10 log

I = I0 1060/10 = 106 2


Pm = IS = I 4r 2 = 4 106 = 12, 6 106

5m

102 W/m2

r I r 2
I=

Pm 10 = = 3, 18 105 W/m2 , 2 4r 4 52 I = 10 log 3, 18 107 = 75 I0

L = 10 log

I = 2I
L = 10 log I 2I = 10 log 2 + 10 log = 78 I0 I0

45

Mara Vir. Martnez Gonzlez


I0 = 1012 W/m2

200 W

R I R
I= Pm . 4R2


I = 130 I0

L = 10 log

I = I0 1013 = 10 W/m2


R= W = 4I 5 = 1, 2 m.

46

Mara Vir. Martnez Gonzlez

A y1 (x, t) = A sen (kx t) y2 (x, t) = A sen (kx t /3) y3 (x, t) = A sen (kx t + /3).


sen( ) = sen cos cos sen
y = = + = A sen (kx t /3) + A sen (kx t) + A sen (kx t + /3) = A sen (kx t) cos /3 A cos(kx t) sen /3 + A sen (kx t) A sen (kx t) cos /3 + A cos(kx t) sen /3 A [2 cos(/3) + 1] sen (kx t) = 2A sen (kx t)

cos /3 = cos 60 = 1/2

y (x, t) = 0, 05 sen 2, 52x cos 500t

A sen (kx + t) + A sen (kx t) = 2A sen kx cos t

k
A = 0, 025 = 6, 28 2 = 2, 5 k 2, 52


= = = 200 k

47 Mara Vir. Martnez Gonzlez

y = 0,
sen 2, 52x = 0 x= n 2, 52

l = x.

x = /2, 52 1, 25

A = 10 = 50 /4

sen(kx t + ) 2 2

y = A sen(kx t) + A sen(kx t + ) = 2A cos


2A cos

= 20 cos = 20 cos 22, 5 = 18, 4 2 8


y = A cos kx cos t

A = A cos kx x

4 103

48

Mara Vir. Martnez Gonzlez


l=n 2


n = = n = n1 2l


n=5

375 = n1 450 = (n + 1)1

1 = 75

n (n 1) x = 0
x= l 2l (n 1)l , ,..., n n n


l=

= T /.
T 1 = 150 360 =2 4 103

1 = 21 21

A = 10 = 50 /4

sen(kx t + ) 2 2

y = A sen(kx t) + A sen(kx t + ) = 2A cos


2A cos

= 20 cos = 20 cos 22, 5 = 18, 4 2 8

y (x, t) = 4, 2 sen 0, 2x cos 300t

49

Mara Vir. Martnez Gonzlez



sen 0, 2x = 0

0, 2x = n

x=n

15, 71 n 0, 2

d = /0, 2 = 2/0, 2 = 10 = /2 = 300/2


= = 15

n n 1
L = nd = n = 20 = 0, 63 m 2

n = 5 sen 0, 2L/2 = 1
v= y (L/2, t) = 4, 2 300 cos 300t t

vmax = 12, 6

50

Mara Vir. Martnez Gonzlez

Bloque Temtico 3: Campos elctricos y magnticos


Algunas de las partculas constituyentes de la materia tienen carga elctrica que es origen de las fuerzas electrostticas. La fuerza electrosttica entre dos cargas es inversamente proporcional al cuadrado de la distancia que las separa. Por lo tanto: a) es una fuerza central; b) puede obtenerse por derivacin a partir de un potencial. Tenemos as un campo vectorial (un campo de fuerzas) y un campo escalar (el potencial electrosttico). Entre ambos existen las relaciones que ya se vieron en el bloque temtico 1. Como all se dijo, los campos escalares se suman escalarmente y los campos vectoriales se suman vectorialmente. As los campos escalares y vectoriales debidos a una distribucin de cargas son la suma de los campos escalar y vectorial producidos por cada carga. Sin embargo, en muchas ocasiones hay una forma ms sencilla de obtener el campo creado por una distribucin de cargas. En efecto, una consecuencia del hecho de que la fuerza entre dos cargas puntuales sea inversamente proporcional a la distancia es el teorema de Gauss, que liga el flujo del campo a travs de una superficie cerrada imaginaria con la carga total encerrada por dicha superficie. En situaciones muy simples podemos elegir superficies tales que el campo vectorial tenga la misma intensidad en todos sus puntos, y el flujo del campo se reduce al producto de dicha intensidad por el rea de la superficie. La distribucin y el tipo de cargas elctricas (libres o ligadas) en un cuerpo determinan sus propiedades elctricas. De acuerdo con esto los medios materiales se dividen en dielctricos (donde las cargas estn ligadas) y conductores (cuyas cargas pueden moverse libremente). La respuesta o el comportamiento de un cuerpo en el seno de un campo elctrico externo dependen de este carcter. As los campos elctricos no pueden penetrar en un medio conductor en equilibrio. Por el contrario, en un medio dielctrico las cargas ligadas se polarizan y dan lugar a un campo interno de sentido contrario al campo externo. Esta propiedad de los dielctricos hace posible almacenar grandes cantidades de carga elctrica en placas conductoras aun manteniendo bajo el campo elctrico entre las mismas gracias a la presencia de un dielctrico; est combinacin de un material dielctrico situado entre dos conductores constituye un condensador. Las cargas elctricas que se mueven libremente dentro de un conductor dan lugar a una corriente elctrica. Lo que mueve a estas cargas es un campo elctrico externo, que realiza un trabajo sobre las mismas. Por lo tanto, las cargas en movimiento tienden a ganar energa cintica. Sin embargo, el movimiento de las cargas se ve dificultado por las caractersticas del medio, lo que da lugar a una fuerza de rozamiento, similar a las fuerzas viscosas. Por consiguiente, esta fuerza de rozamiento tiende a reducir la energa cintica de las cargas. Se llega a s a una velocidad lmite (como se estudio en el Bloque Temtico 1) que determina la intensidad de corriente en el medio. Esta intensidad es proporcional a la diferencia de potencial aplicada entre los extremos, lo que constituye la ley de Ohm. Asimismo, la resistencia del medio da lugar a una disipacin de la energa en forma de calor, el efecto Joule. Las cargas elctricas libres se conservan, es decir, su nmero no vara. Por lo tanto, cuando un conductor por el que circulan cargas se bifurca, una parte de las cargas va por una rama de la bifurcacin y la parte restante va por la otra rama. Lo mismo sucede cuando un conductor se divide en varias ramas. La intensidad de corriente que circula por cada una se obtiene a partir de unas ecuaciones que constituyen las leyes de Kirchoof, que no son otra cosa que la combinacin de la ley de Ohm en cada rama y la conservacin de la carga elctrica total. Si las cargas elctricas que circulan por un conductor se mueven siempre en el mismo sentido, la corriente se llama continua. Las cargas elctricas se mueven por la existencia de un campo elctrico externo, pero para que el flujo de carga se mantenga en un circuito hace falta un generador de cargas en un extremo del mismo. El mecanismo de generacin puede ser de carcter qumico y entonces tenemos una pila voltaica. Una corriente elctrica produce un campo magntico. ste se manifiesta porque ejerce una fuerza sobre una carga elctrica en movimiento. Esta fuerza es: a) perpendicular a la direccin de movimiento de la carga; b) proporcional a la velocidad de la misma. Como consecuencia de a) la fuerza magntica no realiza
51 Mara Vir. Martnez Gonzlez

trabajo sobre la carga y no cambia la energa cintica de sta. Como consecuencia de b) entre dos conductores por los que circulan corrientes elctricas aparecer una fuerza. Una vez ms, el campo producido por varias corrientes es la suma vectorial de los campos producidos por cada una por separado. Pero una vez ms, hay un mtodo que en muchas ocasiones permite acortar los clculos, y es el teorema de Ampere. Este afirma que la circulacin del campo magntico a lo largo de una curva cerrada es proporcional a la intensidad de corriente total que atraviesa una superficie limitada por dicha curva. En situaciones sencillas podemos encontrar una curva a lo largo de la cual el campo tenga la misma intensidad y as la circulacin total es simplemente la intensidad del campo multiplicada por la longitud de la curva. Dentro de los medios materiales hay minsculas espiras de corriente, que dan lugar a campos magnticos internos. A su vez, el comportamiento de estas minsculas espiras en un campo externo da lugar a las propiedades magnticas del medio. As, en funcin de su constitucin interna los medios se dividen en paramagnticos, diamagnticos y ferromagnticos. El ferromagnetismo es especialmente importante: en un material ferromagntico todas las espiras internas estn alineadas y producen un campo magntico intenso. Puesto que un campo magntico produce una fuerza sobre una carga en movimiento, producir una fuerza sobre un conductor en movimiento en el que existen cargas libres. En un conductor cerrado puede darse el caso de que las fuerzas sobre las cargas se sumen dando lugar a una fuerza electromotriz que mueve las cargas a lo largo del circuito. Puede demostrarse entonces que el valor de dicha fuerza electromotriz es igual a la variacin temporal del flujo del campo magntico a travs de una superficie limitada por el circuito: sta es la ley de Lenz. La variacin del flujo puede ser debida a una variacin del campo, a una variacin del rea o a ambas cosas. Podemos aprovechar este efecto para generar una fuerza electromotriz variable y, con ello, una corriente alterna. Asimismo, variando la corriente que circula por un circuito, variamos el campo magntico creado por ste y, con ello, podemos inducir una fuerza electromotriz en un segundo circuito, lo que constituye el fenmeno de induccin mutua. Podemos crear as una corriente en un circuito sin tener contacto fsico con l. Un fenmeno importante es el de autoinduccin. Ahora la variacin de corriente en un circuito altera el flujo del campo magntico que atraviesa el propio circuito y crea una fuerza electromotriz autoinducida que se opone a la variacin inicial de la intensidad. La autoinduccin en un circuito tiene as un sentido similar al de la inercia en mecnica: la autoinduccin se opone a las variaciones de la intensidad. Podemos establecer entonces una analoga entre un oscilador armnico amortiguado y un circuito de corriente en el que haya condensadores, resistencias y autoinducciones. La autoinduccin juega el papel de la masa, la resistencia juega el papel de amortiguador y el condensador juega el papel de la constante k que determina el periodo natural de oscilacin. La ecuacin para el valor de la intensidad en un circuito RLC es idntica a la ecuacin para el movimiento de un oscilador armnico forzado. De la misma forma que un campo magntico variable da lugar a una fuerza electromotriz inducida, un campo elctrico variable da lugar a un campo magntico. Los campos elctrico y magntico variables estn as inextricablemente unidos y en conjunto constituyen un campo electromagntico. El campo electromagntico se propaga en forma de ondas, cuya velocidad est relacionada con las propiedades elctricas y magnticas del medio en que se propagan. En particular, las ondas electromagnticas se propagan en el vaco a una velocidad constante que no depende de la frecuencia. Estas ondas son transversales y se pueden polarizar. Asimismo, transmiten momento y energa. Aunque todas las ondas electromagnticas son idnticas en su naturaleza, es normal clasificarlas de acuerdo con sus frecuencias, puesto que dan lugar a efectos diferentes. Especialmente relevante es la luz visible, que consiste en ondas electromagnticas de tal frecuencia que son capaces de excitar las clulas de la retina. Todos los cuerpos tienen cargas elctricas y todos emiten radiacin electromagntica. La distribucin de la radiacin emitida en las diferentes frecuencias es una propiedad del cuerpo. Particularmente importante es la radiacin de cuerpo negro, que sigue una ley de distribucin universal que sirve de referencia para todas las dems. 52 Mara Vir. Martnez Gonzlez

CAPTULO 7 Interaccin elctrica


Ley de Coulomb
La fuerza entre dos cargas puntuales Q y q separadas a una distancia d tiene un valor 1 Qq Qq F = Ke 2 (1) 2 40 d d y acta en la direccin de la recta que une ambas cargas. (Aqu Ke 1/(40 ) ' 9 109 N.m2 /C2 ) ~ (fuerza por unidad de carga) en un punto ~ El campo elctrico E r creado por una carga puntual Q situada en el origen es Q ~ (~ er (2) E r) = Ke 2 ~ r siendo ~ er un vector unitario en direccin radial a partir del origen. Recordemos del Captulo 3 que un campo vectorial que solo depende de la distancia r a un punto puede obtenerse como el gradiente de un campo escalar. As, el campo elctrico de una carga puntual puede obtenerse a partir de un campo escalar V (r) Z Z Q dr dV (r) V (r) = E (r)dr = Ke Q (3) = Ke + C E (r) = 2 dr r r La funcin V (r) tiene dimensiones de energa por unidad de carga y se denomina potencial elctrico. La constante de integracin C puede fijarse arbitrariamente. Si suponemos que V (r) = 0 en el infinito, entonces C = 0. Evidentemente, si la carga Q est situada en un punto de coordenadas ~ r0 la expresin (2) se transforma en Q ~ (~ er0 E r) = Ke 2~ |~ r ~ r0 | donde ahora ~ er0 es un vector unitario en la direccin de la recta que une el punto ~ r con el punto ~ r0 . El campo elctrico creado por varias cargas es la suma vectorial de los campos creados por cada carga. As, r1 , ~ r2 , ..., ~ ri , ..., el campo creado en ~ r por un conjunto de cargas Q1 , Q2 , ...Qi , ... situadas en puntos de coordenadas ~ respectivamente, es Q1 Q2 Qi ~ (~ er1 + Ke er2 + ... + Ke ~ eri + ... = E r) = Ke 2~ 2~ |~ r ~ r1 | |~ r ~ r2 | |~ r ~ ri |2 X Qi = Ke ~ eri (4) r ~ ri |2 i |~ y el potencial elctrico creado por esta distribuacin de cargas es la suma escalar de los potenciales creados por cada carga X Qi V (r) = Ke |~ r ~ ri | i Ejercicio 1.- Se suspende una partcula de masa m = 105 kg y carga q = 2 104 C, de un hilo inextensible y sin peso, y se introduce en un campo elctrico horizontal de mdulo E = 1 N/C. Se pide (a) Calcular el ngulo de inclinacin del hilo respecto a la vertical, una vez el sistema en equilibrio. (b) Calcular la tensin que soporta el hilo en esa posicin de equilibrio. Tomar, por simplicidad, |g | = 10 m/s2 Solucin: a) Las fuerzas que actan sobre la partcula con el peso P = mg, dirigida verticalmente hacia abajo, la fuerza Fe = qE, dirigida en la direccin del campo elctrico, y la tensin T del hilo. Entonces, llamando al ngulo de inclinacin del hilo, el equilibrio de fuerzas horizontales y verticales da 2 C 104 N/C qE T sin = qE = 5 =2 tan = T cos = mg mg 10 kg 10 m/s2 b) T = 5 104 N T 2 = (mg )2 + (qE )2 = 5 108 N2

53

Mara Vir. Martnez Gonzlez

Ejercicio 2.- Dos cargas puntuales, 3q y q , estn situadas a una distancia d. Determinar: (a) el potencial en un punto P situado en la recta que pasa por las dos cargas y a una distancia x < d de la carga 3q ; (b) el valor de x para que el potencial en dicho punto sea cero; (c) el valor del campo elctrico en dicho punto. Solucin: Supondremos que la carga 3q est en el origen y la carga q est a una distancia d (en el sentido positivo del eje X). Entonces un punto de coordenada x est a una distancia |x| de la carga 3q y a una distancia |x d| de la carga q . Entonces, para 0 < x < d se tiene |x d| = d x y el potencial es q 3q q (3d 4x) + = Ke V (x) = Ke x dx x(d x) b) El potencial ser nulo en un punto x0 tal que 3d 4x0 = 0 x0 = 3d/4 c) El campo elctrico creado por la carga 3q apunta alejndose de la carga mientras que el campo creado por la carga q apunta hacia la carga. Por lo tanto, para x < d los dos campos apuntn en la direccin positiva del eje X. As pues, el campo elctrico total es 3q q 3q q = Ke + E (x) = Ke 2 + Ke x (d x)2 x2 (d x)2 de modo que para x0 = 3d/4 resulta 3q 64 q q Ke 2 = E (x0 ) = Ke + (3d/4)2 (d/4)2 3 d Ejercicio 3.- Una carga puntual 2q est situada en el origen de coordenadas y una segunda carga q est situada en el eje X a la derecha de la primera y a una distancia d. Determinar: a) el potencial elctrico en cualquier punto del eje X, tanto a la izquierda como a la derecha de la carga 2q ; (b) determinar en qu puntos del eje X es nulo el campo elctrico creado por ambas cargas. Solucin: a) Para 0 < x < d, la distancia a la primera carga es x y la distancia a la segunda es d x. Entonces q 2q q (2d 3x) + = Ke V (x) = Ke x dx x(d x) Para x > d, la distancia a la primera carga es x y la distancia a la segunda es x d. Entonces q 2q q (x 2d) V (x) = Ke + = Ke x xd x(x d) b) Para 0 < x < d el campo elctrico q 3x2 + 2d2 4xd 2q q E (x) = Ke = Ke + x2 (d x)2 x2 (d x)2 que se anula en un punto de coordenada x0 tal que 4d 16d2 24d2 2 2 x0 = 3x0 4dx0 + 2d = 0 6 pero este valor es imaginario; es decir, no hay ningn punto entre 0 y d en el que se anula el campo elctrico. Esto era de esperar sin necesidad de clculos. En efecto, entre 0 y d, tanto el campo de la carga 2q como el de la carga q apuntan en el sentido positivo del eje X, luego nunca pueden cancelarse. Para x > d ! q q (x2 + 2d2 4xd) 2q = K E (x) = Ke e x2 (x d)2 x2 (x d)2 que se anula para 2 x2 x0 = 2 + 2 d 0 4dx0 + 2d = 0 Ejercicio 4.- Un anillo de radio R que se encuentra en el plano XY posee una carga Q uniformemente distribuida en toda su longitud. En el centro del anillo se encuentra una partcula de masa m que posee una carga elctrica negativa q . (a) Qu fuerza actuar sobre la carga q se se desplaza una pequea distancia z R a lo largo del eje Z? (b) Calcular el periodo del movimiento de dicha carga q a lo largo del eje Z?

54

Mara Vir. Martnez Gonzlez

Solucin: La longitud total del anillo es L = 2 R. Dividamos el anillo en elementos infinitesimales de longitud l y carga Q = (Q/L)l. Cuando la carga q est desplazada una z a lo largo del eje Z la distancia entre la carga q y cualquier elemento del anillo es p d = z 2 + R2 y la fuerza con la que el elemento i atrae a la carga q es ~ = Ke Q q ~ F ei d2 siendo ~ ei un vector unitario en la direccin de la recta que une la carga q con el elemento i del anillo, que forma un ngulo con el plano XY dado por tan = z/R. Esta fuerza tiene una componente paralela y otra perpendicular al plano XY (en la direccin Z). Es fcil ver que la suma de las componentes paralelas debidas a todos los elementos del anillo es cero: en efecto, por cada elemento habr otro elemento diametralmente opuesto que ejercer sobre la carga q una fuerza de componente paralela igual y opuesta. Por el contrario, todos los elementos ejercern una fuerza en el mismo sentido en la direccin Z, cuyo valor es Q q Q q z Fz = Ke 2 sin = Ke 2 d d d y la suma total para todo el anillo es Ke Q q z Fz = z = Ke Q q 3/2 2 d3 (z + R2 ) Esta fuerza es, en general, una funcin complicada de z. Sin embargo, para z R, podemos hacer la aproximacin d = R, y as Ke Q q z Fz = R3 que es proporcional a z. El movimiento de la carga q a lo largo del eje Z (para pequeos valores de z ) obedece as a la ecuacin Ke Q q d2 z z m 2 = dt R3 que es un movimiento armnico de frecuencia r Ke Q q = mR3 y periodo s mR3 2 = 2 T = Ke Q q

Teorema de Gauss
El flujo E de campo elctrico a travs de una superficie cerrada es I ~ dS ~ = QT = 4 Ke QT E E 0 siendo QT la carga total encerrada por la superficie. El flujo es fcil de calcular para distribuciones de carga con simetras claras. As 1) Campo elctrico creado por un hilo rectilneo infinito con densidad lineal de carga E (r ) = siendo r la distancia al hilo. 1 = 2Ke 20 r r (5)

2) Campo elctrico creado por una lmina plana infinita de densidad superficial de carga . 20 El campo es perpendicular a la lmina e independiente de la distancia a la misma. E= (6)

3) Campo elctrico creado por una superficie esfrica de radio R con carga total Q uniformemente distribuida

55

Mara Vir. Martnez Gonzlez

r r

< R > R

E (r ) = 0 E (r ) =

Q 1 1 = Ke Q 2 (7) 2 40 r r siendo r la distancia al centro de la superficie esfrica. O en trminos de la densidad superficial de carga = Q/(4 R2 ) r < R E (r ) = 0 R2 1 1 = 4 Ke R2 2 r > R E (r ) = 0 r2 r 4) Campo elctrico creado por una esfera maciza de radio R con carga total Q uniformemente distribuida Q r r < R E (r) = 40 R3 Q 1 r > R E (r) = (8) 40 r2 3 siendo r la distancia al centro de la esfera. O en trminos de la densidad volmica de carga = Q/( 4 3 R ) a 4 Ke r r < R E (r ) = r= 30 3 4 3 1 R3 1 R Ke 2 = r > R E (r ) = 30 r2 3 r

Ejercicio 5.- Calcular el potencial elctrico debido a las distribuciones de carga anteriores. Solucin: 1) Hilo rectilneo infinito con densidad lineal de carga . Sustituyendo (5) en (3) Z Z dr = ln r + C V (r) = E (r)dr = 20 r 20 Tal como est escrita, esta expresin no tiene sentido fsico; en efecto, r tiene dimensiones de longitud y el argumento de un logaritmo no debe tener dimensiones. Sin embargo, si se escribe la constante de integracin como C = (/20 ) ln r0 , siendo r0 una distancia de referencia arbitraria, la expresin para el potencial queda Z Z r dr = V (r) = E (r)dr = ln 20 r 20 r0 2) Lmina plana infinita de densidad superficial de carga . Sustituyendo (6) en (3) Z V (x) = dx = x 20 20 siendo x la distancia a la lmina. (Se ha escogido la constante de integracin de modo que el potencial en la lmina sea cero.) 3) Campo elctrico creado por una superficie esfrica de radio R con carga total Q uniformemente distribuida. Sustituyendo (7) en (3) Z Q Q 1 1 r > R V (r ) = dr = 2 40 r 40 r Q 1 r < R V (r) = Cte = 40 R donde el valor de la constante se ha elegido para que el potencial sea continuo en r = R. 4) Campo elctrico creado por una esfera maciza de radio R con carga total Q uniformemente distribuida Z Q Q 1 1 r > R V (r ) = dr = 40 r2 40 r Z Q 1 2 Q r2 Q 3R r2 +C = rdr = r < R V (r ) = 3 3 3 40 R 40 R 2 40 2R

56

Mara Vir. Martnez Gonzlez

donde se ha elegido la constante C= para que el potencial sea continuo en r = R 3 Q 2R 40

Ejercicio 6.- Una partcula con carga Q = 72 109 C est situada en el centro de un cubo. (a) Calcular el flujo total del campo elctrico a travs de la superficie cbica. (b) Cunto vale el flujo a travs de cada cara? (c) Si la carga no estuviera en el centro, cmo cambiaran las respuestas a los apartados anteriores? Solucin: a) Por el teorema de Gauss Q = 4 Ke Q = 4 9 109 N.m2 /C2 72 109 C = 8143 N.m2 /C = 0 b) Puesto que la carga est en el centro del cubo, los flujos a travs de cada una de las 6 caras sern iguales T = = 1357, 17 N.m2 /C 6 El signo negativo indica que el campo elctrico en la superficie del cubo apunta siempre hacia dentro; es decir, las lneas de fuerza entran en el cubo. c) Mientras la carga est dentro del cubo, el flujo total sigue siendo el mismo. Sin embargo, los flujos parciales a travs de cada cara ya no son iguales: los flujos sern mayores (en valor absoluto) en las caras ms prximas a la carga y menores en las ms alejadas. Si la carga se desplaza hasta salir del cubo, ya no habr carga encerrada dentro de la superficie del cubo y el flujo total a travs de las caras del cubo sera cero. Las lneas de fuerza que entran en el cubo por unas caras(las ms prximas a la carga) saldrn por otras. Ejercicio 7.- Tenemos un pndulo de masa m y longitud l sometido a la accin de la gravedad. Adems, la masa del pndulo tiene una carga elctrica q y por debajo del pndulo hay una superficie horizontal uniformemente cargada con densidad superficial de carga . Cul ser el periodo de las oscilaciones del pndulo? Solucin: Sabemos que el periodo de un pndulo simple sometido nicamente a la accin de la gravedad, es decir, al ~ = mg~ peso P ez , es s l T0 = 2 g Si ahora aadimos la fuerza debida al campo elctrico, y tenemos en cuenta que el campo elctrico creado por una superficie plana infinita es perpendicular a la misma y vale E = /20 , la fuerza que acta sobre el pndulo es q q ~ ~ ~ ~ ez ~ ez = m g F = P + q E = mg + 20 20 m El problema es similar al caso del pndulo simple pero ahora a la aceleracin de la gravedad g hay que restarle una aceleracin tambin constante q /2m0 debida al campo elctrico. Entonces, en la expresin que da el periodo basta sustituir g por g (1 q /20 m) y as s T0 l =p T = 2 g (1 q / (20 m)) 1 q / (20 m) Hay que advertir que en esta expresin q y llevan implcito su signo. Es decir, q es positivo si y Q tienen el mismo signo y es negativo si y q tienen signo contrario. En consecuencia T > T0 si y q tienen el mismo signo (lo que da lugar a una fuerza repulsiva), y T < T0 si tienen sentido contrario (fuerza atractiva). Ejercicio 8.- Consideremos dos lminas planas infinitas perpendiculares al eje X que cortan a ste en los puntos x = d y x = d. Estas lminas tienen una misma densidad superficial de carga . Entre ambas (en x = 0) y paralela a ellas hay una tercera lmina infinita con densidad superficial de carga 2 . Calcular el campo y el potencial elctrico a lo largo del eje X. Solucin:

57

Mara Vir. Martnez Gonzlez

El campo elctrico creado por una lmita infinita uniformemente cargado es independiente de la distancia a la lmina. Por lo tanto, para |x| > d el campo total creado por las 3 lminas es 2 + =0 si |x| > d ET = 20 20 20 es decir, el campo creado por la lmina de densidad 2 neutraliza a los campos creados por las lminas de densidad . En la regin d < x < d, los campos creados por las lminas de densidad se cancelan y solo queda el campo creado por la lmina de densidad 2 . As 2 si |x| < d ET = 20 y esta dirigido haca la lmina central. Con esto, el potencial electrosttico a lo largo del eje ser Z Cte = ( /0 )d si |x| > d V (x) = Edx = si |x| < d ( /0 ) |x| donde la constante de ha elegido de modo que el potencial sea continuo en |x| = d. As pues, el potencial es simtrico respecto al origen: es cero en x = 0 y aumenta linealmente con la distancia a la lmina hasta una distancia d donde alcanza el valor ( /0 )d. A partir de entonces permanece constante. Ejercicio 9.- Sean 3 superficies esfricas concntricas de radios r1 = 5 cm, r2 = 10 cm y r3 =15 cm. Las tres superficies tienen una misma carga Q = 8 C uniformemente distribuida. Calcular el campo elctrico en un punto situado a una distancia d = 12 cm del centro de las superficies. (Tomar, por simplicidad, K = 1/(40 ) = 9 109 N.m2 /C2 ) Solucin: El punto en cuestin est situado en el interior de la superficie mayor y fuera de las otras dos superficies. Por lo tanto, el campo creado en este punto por la superficie mayor es nulo, y el campo creado por cada una de las otras dos superficies es el que creara toda la carga de estas situada en el origen. Es decir Q 1 Q 2Q 2Ke Q ~ ~ ~ ~ + +0 ~ er = ~ er = ~ er = ET = E1 + E2 + E3 = 40 d2 d2 40 d2 d2 18 109 N.m2 /C2 8 106 C = = 107 N/C (0, 12)2 m2 Ejercicio 10.- Tenemos una corteza esfrica de radio interior Ri = R y radio exterior Re = 3R con densidad de carga elctrica uniforme . Calcular (en funcin de y R) el campo elctrico creado por la corteza a distancias R, 2R, 3R, 4R y 5R del centro de la corteza. Solucin: Puesto que el problema tiene simetra esfrica, el teorema de Gauss se reduce a Q(r) Q(r) 1 o E (r ) = E (r)4r2 = 0 40 r2 siendo E (r) el campo elctrico a una distancia r del centro de la corteza y Q(r) la carga encerrada por una superficie esfrica de radio r. Por otra parte, el volumen de una corteza esfrica de radio interior R y radio exterior r es V ol(r) = (4 /3)(r3 3 R ). Para r = R tenemos V (R) = 0. La superficie no encierra ninguna carga. Por lo tanto Q(R) = 0 y E (R) = 0. Una superficie esfrica ideal de radio R coincide con la superficie interior de la corteza, que rodea a un vaco. Por lo tanto, no contiene en su interior ninguna carga y as E (R) = 0 Para r = 2R tenemos V ol(2R) = (4/3)(8R3 R3 ); y la carga total encerrada es Q(r) = V ol(2R). Entonces 7R 4(8R3 R3 ) 1 = E (2R) = 2 3 40 (2R) 120

58

Mara Vir. Martnez Gonzlez

Anlogamente, para r = 3R 4(27R3 R3 ) 1 26R = 3 40 (3R)2 270 A partir de esta distancia la carga encerrada permanece constante pues la corteza se ha acabado. Entonces 4(27R3 R3 ) 1 13R E (4R) = = 3 40 (4R)2 240 E (3R) = E (5R) = 4(27R3 R3 ) 1 26R = 2 3 40 (5R) 750

Ejercicio 11.- Sea una superficie esfrica (esfera hueca) S1 de radio R1 y densidad superficial de carga 1 centrada en el origen. En su interior tenemos otra superficia esfrica de radio R2 = R1 /4, densidad superficial de carga 2 y centrada en el punto de coordenadas (R1 /2, 0, 0). Calcular el campo elctrico creado por ambas esferas en un punto P de coordenadas (R1 /2, 0, 0). Solucin: El punto P est dentro de la superficie S1 , por lo que el campo en P debido a S1 es nulo. Adems P es exterior a S2 y est situado a una distancia R1 a la izquierda de su centro. Por lo tanto, el campo creado por S2 en P est dirigido a lo largo del eje X (y en sentido negativo) y su valor es 2 2 Q2 4R2 2 R2 = 4 K E2 = Ke 2 = Ke e 2 2 R1 R1 R1 En resumen 2 2 2 R2 ~1 + E ~ 2 = 4 Ke 2 R2 ~T = E ~ ex = ~ ex E R1 0 R1 Ejercicio 12.- Dentro de una esfera de radio R y uniformemente cargada con densidad volmica de carga existe una cavidad esfrica neutra de radio R1 con su centro a una distancia a del centro de la esfera, tal que (R1 + a) < R. a) Encontrar el campo elctrico en el centro de la cavidad. b) Demostrar que el campo elctrico es el mismo en todos los puntos de la cavidad. Solucin a) La esfera con una cavidad neutra equivale a la combinacin de una esfera totalmente maciza de radio R y carga ms una esfera tambin maciza de radio R1 y carga . De esta forma, la densidad de carga total dentro de la esfera de radio R1 es = 0, y recuperamos el sistema original. Ahora el campo creado por las dos esferas es la suma vectorial de los campos creados por cada una de ellas. El campo creado por la esfera pequea (de radio R1 ) en su centro es nulo, mientras que el campo creado por la esfera grande a una distancia a de su centro tiene direccin radial y vale ~ = a ~ er E 30 b) Vamos a calcular el campo en el interior de la cavidad de forma ms general. Consideremos el centro de la esfera como origen de coordenadas. Sea ~ a el radio vector del centro de la cavidad y sea el ~ r el radio vector de un punto r ~ a es el vector que va desde el centro de la cavidad al punto P. El punto P es un P de la cavidad. Entonces ~ ra = ~ punto interior tanto de la esfera de radio R como de la de radio R1 . As el campo elctrico creado en P por la primera ~ 2 = ~ ~ 1 = ~ r/30 , mientras que el campo elctrico creado por la segunda es E ra /30 = (~ r ~ a)/30 . esfera es E Por lo tanto, el campo total es ~2 = ~ ~ =E ~1 + E r (~ r ~ a) = ~ a E 30 30 30 que es igual, en valor y direccin, en todos los puntos de la cavidad. Ejercicio 13.- Sean dos superficies conductoras, esfricas y concntricas de radios r1 = 3 cm y r2 = 6 cm. Ambas superficies tienen una misma cantidad de carga Q = 5 108 C uniformemente distribuida. (a) Calcular el campo elctrico entre las superficies. (b) Calcular la diferencia de potencial entre las dos superficies. Solucin:

59

Mara Vir. Martnez Gonzlez

(a) Cualquier punto situado entre las superficies est dentro de la suferficie de radio mayor y fuera de la de radio menor. Por lo tanto, el campo creado por la superficie mayor es nulo y el campo creado por la superficie menor es igual al que creara una carga puntual Q situada en el centro, es decir ~ = Ke Q ~ er r1 < r < r2 E r2 siendo ~ er un vector unitario en la direccin radial. (b) La diferencia de potencial entre las superficies es entonces Z r2 Z r2 1 r1 r2 dr 1 ~ d~ = Ke Q = K Q = V2 V1 = E r = Ke Q e 2 r r r r1 r2 2 1 r1 r1 (0, 03) m = 9 109 N.m2 /C2 5 109 C = 7, 5 V 0, 18 m2 El potencial es menor en la superficie de radio mayor porque la carga Q de la superficie menor es positiva. Ntese que este resultado es independiente de la carga que pueda haber en la superficie de radio mayor.

Ejercicio 14.- Sean dos superficies esfricas concntricas de radios r1 = 0, 1 m y r2 = 0, 2 m. con densidades superficiales de carga 1 y 2 , respectivamente. El potencial elctrico en la primera superficie vale V1 = 100 V y el potencial en la segunda superficie vale V2 = 200 Voltios. Calcular la carga elctrica total de la primera superficie. Solucin: De acuerdo con el problema anterior Z r2 1 1 r1 r2 ~ = Ke Q V2 V1 = E d~ r = Ke Q r2 r1 r1 r2 r1 de modo que 100 V 0, 02 m2 2 (V2 V1 ) r1 r2 = = 108 C Q= 9 2 2 Ke (r1 r2 ) 9 10 N m C (0, 1) m 9 Ejercicio 15.- Una superficie esfrica centrada en el origen tiene una densidad superficial de carga = 25 nC/m2 . A una cierta distancia r1 del centro el potencial es V = 500 V y el mdulo del campo elctrico es E = 250 V/m. Determinar el radio R de la superficie esfrica y la carga total de la misma. (Nota: la escala del potencial se ha tomado de modo que el potencial es cero a distancia infinita.) Solucin: Evidentemente R < r1 . De lo contrario el campo elctrico a dicha distancia sera nulo. Entonces, de las expresiones para el campo elctrico y el potencial para distancias r > R obtenemos Ke Q E (r1 ) = 2 = 250 V/m r1 Ke Q = 500 V/m V (r1 ) = r1 y dividiendo la segunda por la primera 500 m=2m r1 = 250 Sustituyendo este valor en la segunda ecuacin se obtiene r1 V (r1 ) 2 m 500 V Q= = = 1, 1 107 C Ke 9 109 N.m2 /C2 y r r Q 1, 1 107 C 2 = R= = 0, 59 m Q = 4R 4 4 25 109 C/m2 Ejercicio 16.- El potencial en la superficie de una esfera uniformemente cargada es 450 V . A una distancia radial de 20 cm de la superficie, el potencial es 150 V . Cul es el radio de la esfera y cul es su carga? Solucin:

60

Mara Vir. Martnez Gonzlez

Sea R el radio de la esfera. Entonces un punto a una distancia de r = 20 cm de la superficie est a una distancia R + r del centro de la esfera. Entonces Ke Q = 450 V V (R ) = R Ke Q V (R + r ) = = 150 V R+r y dividiendo la primera ecuacin por la segunda r R+r =3 R = = 0, 1 m R 2 y as RV (R) 0, 1 m 450 V Q= = = 5 109 C Ke 9 109 N.m2 /C2 Ejercicio 17.- Consideremos un cuerpo metlico esfrico de radio R con una carga elctrica total Q. Calcular el campo elctrico y el potencial a una distancia r del centro de la esfera. Cul sera la respuesta si el cuerpo no es esfrico? Solucin: Un cuerpo metlico es conductor. Toda la carga de un conductor en equilibrio se encuentra en la superficie. Entonces este caso se reduce al caso (3) antes mencionado al tratar el teorema de Gauss. En la superficie, r = R, el campo es normal a la misma y vale E (R) = /0 , con = Q/(4 R2 ). Dentro de la esfera, r < R, el campo es nulo y el potencial es constante e igual al de la superficie. Fuera de la superfie del cuerpo, r > R, las superficies equipotenciales son superficies esfricas. Si el cuerpo no es esfrico, la superficie del cuerpo sigue siendo una superficie equipotencial y el potencial en el interior es el mismo que en la superficie. En la superficie el campo elctrico es normal a la miema pero ya no es igual en todos los puntos de la misma, sino que es mayor en los puntos de mayor curvatura donde la densidad superficial de carga es mayor (ver seccin 7.7.1 de la Unidad Didctica). Fuera del cuerpo las superficies equipotenciales estn ms prximas en las zonas de mayor curvatura y ms separadas en las de menor curvatura. , respectivamente. Ejercicio 18.- Dos esferas metlicas de 5 cm y 10 cm de radio se cargan a 103 V y 103 V Una vez cargadas se alejan hasta una distancia de 10 m, que se puede considerar muy grande comparada con los radios. a) Estas esferas, se atraen o se repelen? Con qu fuerza? b) Las dos esferas se ponen en contacto mediante un hilo metlico. Al cabo de un rato se corta el hilo. En esta nueva situacin, con qu fuerza se atraen o se repelen? (Dato: Ke = 9 109 Nm2 /C2 . Solucin: . El potencial elctrico a) Las esferas estn incicialmente a potenciales V1 =103 V y V2 = V1 = 103 V en la superficie de una esfera de carga Q y radio R es V (R) = Ke Q/R. Por lo tanto, las cargas iniciales de las dos esferas son V V R2 R1 y Q2 = R2 = Q1 = 2Q1 Q1 = Ke Ke R1 (con V = 103 V). Entonces la fuerza entre ambas ser 2 2V 2 R1 Q1 Q2 2Q2 F = Ke 2 = Ke 21 = = d d Ke d2 2 2 106 V2 5 102 m2 5 108 N ' 0, 55 107 N = = 9 2 2 2 2 9 10 N.m /C 10 m 9 y ser atractiva (lo que era inmediato a primera vista ya que al tener los potenciales distinto signo, tambin deberan tenerlo las cargas de cada esfera). b) Al poner en contacto las esferas mediante un hilo conductor, habr una transferencia de carga hasta que los potenciales se igualen. Entonces, las nuevas cargas Q1 y Q2 de las esferas satisfarn Q Q R2 Ke 1 = Ke 2 Q2 = Q1 = 2Q1 R1 R2 R1 que junto con la condicin de conservacin de la carga total V Q1 + Q2 = Q1 + Q2 = (R1 R2 ) Ke

61

Mara Vir. Martnez Gonzlez

V 2V (R1 R2 ) Q2 = (R1 R2 ) 3Ke 3Ke y la fuerza entre ambas esferas ser ahora 2V 2 (R1 R2 )2 Q Q 2Q 2 F = Ke 1 2 2 = Ke 21 = d d 9Ke d2 2 que (multiplicando y dividiendo por R1 ), se puede escribir como 2 2V 2 R1 (R1 R2 )2 F (R1 R2 )2 = F = = 0, 06 107 N F = 2 2 2 Ke d 9R1 9R1 9 y ser repulsiva (lo que una vez ms era inmediato ya que al igualarse los potenciales, debe igualarse el signo de las cargas). Q1 =

da

Trabajo y energa
Puesto que el campo elctrico puede obtenerse a partir de un potencial, el trabajo necesario para llevar una carga q de un punto A a un punto B es independiente del camino seguido y vale WAB = q (VB VA ) La energa total de una partcula de masa m y carga q es la suma de la energa cintica y la energa potencial elctrica, y es constante 1 ET = mv 2 + qV = cte 2 Ejercicio 19.- Una carga elctrica de valor Q = 8C se encuentra fija en el origen de coordenadas. Calcular el trabajo necesario para llevar una carga q = 1 C desde un punto de coordenadas cartesianas (0, 0, 2) a otro de coordenadas (3, 4, 0). (Las coordenadas estn dadas en cm.) (Tomar Ke = 1/(40 ) = 9 109 N.m2 /C2 ) Solucin: La distancia del punto (0, 0, 2) al origen es r1 = 2 cm. La distancia del punto (3, 4, 0) al origen es r2 = 32 + 42 = 5 cm. As, la diferencia de potencial electrosttico creado por la carga Q entre ambos puntos es 1 1 r1 r2 = Ke Q = V2 V1 = Ke Q r2 r1 r1 r2 9 109 N.m2 /C2 (8 106 ) C (0, 03) m = = 2, 16 106 V 0, 02 0, 05 m2 El trabajo necesario para llevar la carga q de un punto a otro ser entonces W = q (V2 V1 ) = 106 C 2, 16 106 V = 2, 16 J Ejercicio 20.- Un conductor rectilneo infinito de densidad lineal de carga = 5 104 C/m corre a lo largo del eje X. Calcular el trabajo necesario para llevar una carga puntual q = 3 105 C desde un r2 = (10, 12, 16). (Las coordenadas se expresan en punto de coordenadas ~ r1 = (4, 8, 6) a otro de coordenadas ~ centmetros.) (Datos: Ke = 9 109 Nm2 /C2 ) Sol:ucin De acuerdo con el Ejercicio 5 anterior, el potencial elctrico debido al conductor rectilneo a una distancia r es Z Z r r dr = ln = 2Ke ln V (r) = E (r)dr = 20 r 20 r0 r0 de modo que el trabajo necesario para llevar la carga q del punto 1 al punto 2 es r1 r2 r1 = 2q Ke ln W12 = q [V (r1 ) V (r2 )] = 2q Ke ln ln r0 r0 r2

62

Mara Vir. Martnez Gonzlez

expresin que, como es lgico, ya no depende del parmetro arbitrario r0 que sale de la constante de integracin; solo depende del cociente de las distancias de los puntos al hilo. Puesto que el hilo corre a lo largo del eje X, estas distancias solo dependen de las coordenadas y y z de los puntos q p p 2 + z2 = 82 + 62 cm = 10 cm r2 = 122 + 162 cm = 20 cm = 2r1 r1 = y2 2 y as W12 = 2q Ke ln 1 = 6 105 C 5 104 C/m 9 109 N m2 /C2 ln(1/2) = 270 ln(1/2) J 2

Ejercicio 21.- Dos superficies planas paralelas distan 5 cm y poseen densidades superficiales de carga uniformes y . Un electrn las atraviesa perpendicularmente. Cuando el electrn atraviesa la superficie cargada negativamente su velocidad es v0 = 1, 0 105 m/s, mientras que cuando atraviesa la superficie con carga positiva su velocidad es v1 = 1, 0 106 m/s. Calcular el valor de . (Datos: Ke = 1/(40 ) = 9 109 Nm2 /C2 ; carga especfica del electrn e/m = 1, 76 1011 C/kg ) Solucin: El campo elctrico creado por una lmina infinita es perpendicular a la misma y tiene un valor constante /20 . Por lo tanto, el campo elctrico entre las dos lminas vale E = /0 y est dirigido desde la lmina positiva a la negativa. Puesto que el campo es constante, la diferencia de potencial elctrico entre las lminas es entonces V = V+ V = Ed = d/0 , donde hemos llamado V+ al potencial de la lmina positiva y V al de la negativa. Puesto que el campo elctrico es conservativo 1 1 e d 1 2 2 2 2 mv1 m(v0 + eV+ = mv0 + eV v1 ) = e(V+ V. ) = 2 2 2 0 y as 2 2 2 (v 2 v1 0 (v0 v1 )m )m = 0 = = 2d e 8 Ke d e 10 10 1012 m2 /s2 = ' 5 1010 C/m2 (8 9 109 ) N.m2 /C2 (5 102 ) m (1, 76 1011 ) C/kg

Ejercicio 22.- Un dipolo elctrico p ~ = q~ a orientado a un ngulo con respecto al eje X se encuentra inmerso ~ = E~ en un campo elctrico uniforme E ex . (a) Qu fuerza neta acta sobre el dipolo? (b) Cul es el momento de las fuerzas que actan sobre el dipolo? (c) Que trabajo hay que realizar para rotar el dipolo un ngulo ? (d) Cul es la posicin estable del dipolo? Solucin: Si llamamos x() , y () = (x, 0) a las coordenadas de la carga negativa del dipolo, las coordenadas de la carga positiva son x(+) , y (+) = (x + a cos , a sin ). ~+ sobre la carga positiva y la fuerza F ~ sobre la (a) La fuerza neta sobre el dipolo es la suma de la fuerza F carga negativa. Puesto que el campo elctrico es uniforme la fuerza sobre las cargas tiene el mismo valor pero sentido contrario ~+ + F ~ = qE~ ~T ot = F ex qE~ ex = 0 F ~ es nulo y el momento de la fuerza (b) Tomando el punto (x, 0) como referencia, el momento de la fuerza F ~ F+ es perpendicular al plano XY y su mdulo es ~ = (qE ) a sin = p ~E

Por lo tanto, para 6= 0, hay un par de fuerzas que tiende a rotar el dipolo. (c) Puesto que el campo elctrico es uniforme, el potencial elctrico vale V (x) = Ex. Entonces, la energa potencial del dipolo a un ngulo es V = qV+ qV = qEx(+) + qEx() = qE (x + a cos ) + qEx = ~ = qEa cos = pE cos = p ~E

63

Mara Vir. Martnez Gonzlez

Si se rota el dipolo un ngulo , quedar formando un ngulo + con el eje X, y la nueva energa potencial ser V+ = pE cos ( + ) de modo que el trabajo realizado para rotarlo es W = V+ V = pE [cos ( + ) cos ()] (c) La posicin estable es aquella para la que la energa potencial es mnima, es decir, cos = 1 y = 0 : el ~ son paralelos y el par que acta dipolo est entonces orientado en la direccin del campo elctrico. En este caso p ~yE sobre el dipolo es nulo. Ejercicio 23.- Un dipolo elctrico p ~ = q~ a orientado a un ngulo respecto al eje X est sometido a un campo ~ (x) = kx~ elctrico no uniforme E ex . Calcular la fuerza total y el momento de la fuerza que acta sobre el dipolo. Solucin: ~ a las fuerzas que actan sobre las cargas positiva y negativa, respectivamente ~+ y F Llamando F ~+ + F ~ = q E ~ (x+ ) q E ~ (x ) = qk (x + a cos )~ ~T ot = F ex qkx~ ex = qka cos ~ ex = k (p ~~ ex ) ~ ex F

~ es nulo y el momento de la fuerza F ~+ Tomando el punto (x, 0) como referencia, el momento de la fuerza F es perpendicular al plano XY y su mdulo es = qk(x + a cos )a sin = qkxa sin + qka2 sin cos = 1 1 ~~ ex | + kpa sin 2 = kpx sin + kpa sin 2 = kx |p 2 2 Para 6= 0, hay un par de fuerzas que tiende a rotar al dipolo. Cuando = 0 el par se anula, pero queda ~T ot = k~ p que arrastra al dipolo en la direccin del eje X. una fuerza neta F El problema anterior se puede generalizar al caso de un dipolo sometido a un campo elctrico con una dependencia cualquiera de x. No hay ms que tener en cuenta que a es muy pequeo y por lo tanto se puede utilizar la aproximacin dE dE E (x+ ) E (x ) + E (x ) = E (x ) + a cos ' x+ x dx dx de modo que dE dE ~ a cos ~ ex = (p ~~ ex ) ~ ex FT ot = q dx dx dE 1 dE a cos a sin = E (x) |p ~~ ex | + pa sin 2 = q E (x) + dx 2 dx ~ = cte, (dE/dx) = 0 y Para E ~ ~T ot = 0 = E |p ~~ ex | = p ~E F ~ = kx~ Para E ex , (dE/dx) = k y ~T ot = k (p ~~ ex ) ~ ex F 1 = k |p ~~ ex | + kpa sin 2 2 como ya hemos visto.

Ejercicio 24.- Un condensador plano-paralelo de capacidad inicial Ci = 109 F tiene una carga Q = 106 C en cada placa. (a) Cul es la diferencia de potencial entre las placas? (b) Cul sera la diferencia de potencial si, manteniendo la carga Q constante, se duplica la separacin entre las placas? (c) Que trabajo hay que realizar para duplicar la separacin? Solucin: (a) De la definicin de capacidad Vi = Q 106 C = 9 = 103 V Ci 10 F

64

Mara Vir. Martnez Gonzlez

(b) Para un condensador plano-paralelo C = 0 S/d, de modo que al duplicar la separacin entre las placas, la capacidad se divide por dos, es decir, Cf = Ci /2. As pues, la nueva diferencia de potencial ser Q 2Q Vf = = = 2 103 V Cf Ci (c) La energa electrosttica de un condensador de capacidad C con carga Q es (ver ecuacin (7.30) de la Unidad Didctica) 1 Q2 Energ a = 2 C de modo que el trabajo necesario para pasar de la situacin inicial a la final es Q2 Q2 1 1012 C2 1 W = E = = 0, 5 103 J = = 2 Cf Ci 2Ci 2 109 F Podramos haber llegado al mismo resultado de otra manera. En efecto, una de las placas del condensador crea un campo elctrico uniforme de valor E = /(20 ) = Q/(20 S ). As pues, la fuerza total que acta sobre la otra placa es Q2 F = QE = 20 S y el trabajo necesario para llevar esta segunda placa desde una distancia inicial d a una distancia final 2d es Q2 d Q2 Q2 (2d d) = = W = 20 S 20 S 2Ci

65

Mara Vir. Martnez Gonzlez

CAPTULO 8
Ley de Ohm
La intensidad de corriente elctrica a travs de una superficie es la carga total que atraviesa dicha superficie por unidad de tiempo. Se llama densidad de corriente a la magnitud vectorial ~ j (~ r) = qn(~ r)~ v (~ r), siendo q la carga elctrica de las partculas, n(~ r) el nmero de partculas por unidad de volumen y ~ v (~ r) su velocidad. La densidad de corriente elctrica en un punto de un conductor es proporcional al campo elctrico en dicho punto ~ (~ ~ j (~ r) = E r) (1) La constante de proporcionalidad es la conductividad elctrica y es caracterstica de cada material q2 n = k Para un conductor rectilneo de longitud l y seccin S , la ecuacin (1) se traduce en V (2) I= R siendo V la diferencia de potencial entre los extremos del conductor, y R = l/ S l/S es la resistencia del conductor.

Efecto Joule
La potencia disipada en un conductor de resistencia R por el que circula una corriente de intensidad I es P = I 2R (3) que, de acuerdo con (2) puede escribirse V2 P = R Ejercicio 1.- En un tomo de hidrgeno en su estado fundamental, un electrn describe una rbita circular de radio r = 0, 5 en torno a un protn. Cul es la intensidad de la corriente elctrica a lo largo de dicha rbita? Solucin: Si el electrn se mueve a velocidad v en una rbita de radio r, el periodo de la rbita ser T = 2r/v. Por lo tanto, por un punto de la rbita pasa una carga e cada tiempo T. La intensidad de la corriente orbital es ev e = I= T 2r Por otra parte, la aceleracin normal del electrn es aN = v 2 /r. Esta aceleracin la proporciona la fuerza elctrosttica de atraccin entre protn y electrn 1/2 v2 e2 Ke e2 v= Ke 2 = m Fe = maN r r mr Entonces s 1/2 Ke 1, 62 1038 C2 9 109 N m2 /C2 e2 = 1, 16 103 A = I= 9, 11 1031 kg 2 r3/2 m 2 0, 35 1015 m3/2 Ejercicio 2.- Por un hilo conductor de 1 m de longitud y 0,2 mm de dimetro circula una corriente de intensidad I = 1 A. La conductividad del material del hilo es = 6107 1 m1 , y la densidad de electrones libres es n = 91028 electrones/m3 . (La carga de un electrn es e = 1, 6 1019 C) Calcular: a) la densidad de corriente; b) la velocidad de los electrones; c) la resistencia total del hilo; d) la diferencia de potencial entre los extremos del hilo; e) el campo elctrico que acta sobre las cargas; f) la constante k de la fuerza de rozamiento F = kv que acta sobre las cargas; g) la potencia disipada en el hilo. Solucin: a) La densidad de corriente es I 1A 108 I A m2 = j= = 2 = S r 108 m2

66

Mara Vir. Martnez Gonzlez

b) La velocidad de los electrones se obtiene de 108 A m2 j = j = env v= = 2, 2 103 m s1 en 1, 6 1019 C 9 1028 m3 c) La resistencia total es 1m l = R= = 5, 3 101 7 1 S 6 10 m1 108 m2 d) La diferencia de potancial entre los extremos del hilo V = IR = 5, 3 101 V e) El campo elctrico puede obtenerse de 108 A m2 j j = E E= = = 5, 3 101 V m1 6 107 1 m1 o alternativamente V = 5, 3 101 V m1 E= l f) La constante de fuerza se obtiene de e2 n 1, 62 1038 C2 9 1028 m3 e2 n k= = = = 3, 84 1017 N m1 k 6 107 1 m1 o alternativamente eE 1, 6 1019 C 5, 3 101 V m1 eE = kv k= = = 3, 84 1017 N m1 v 2, 2 103 m s1 g) La potencia disipada es P = I 2 R = 1 A2 5, 3 101 = 5, 3 101 J s1 Ejercicio 3.- En la seccin 8.6 de la Unidad Didctica se expone que entre la superficie de la Tierra y la capa superior de de la atmsfera hay una gran diferencia de potencial que dara lugar a una intensidad de corriente hacia el suelo. Consideremos dos superficies esfricas concentricas de radios Ra y Rb . Los potenciales elctricos en dichas superficies son Va y Vb . Ahora se llena el espacio entre las dos superficies de un material conductor de resistividad . Cul es la intensidad de la corriente que se establece entre las dos superficies? Cul es la resistencia total de la corteza conductora? Solucin: Si llamamos Q a la carga elctrica de la superficie interna de radio Ra , el campo elctrico entre las superficies es Ke Q Ra < r < Rb E (r ) = 2 r En el conductor aparecer una corriente dirigida radialmente cuya densidad de corriente es 1 Ke Q 1 j (r ) = E (r ) = r2 La corriente total que atraviesa una superficie esfrica ideal de radio r es entonces 1 Ke Q 4Ke Q I = j (r )S = 4r2 = 2 r es decir, la intensidad es la misma a travs de cualquier superficie. Supongamos dos superficies ideales de radios r y r + dr. La resistencia del material conductor comprendido entre estas dos superficies es 1 V (r + dr) V (r) = E (r)dr = dr dR(r) = I 4Ke Q 4 r2 y por lo tanto, la resistencia total entre Ra y Rb es Z Rb 1 1 1 dr = RT otal = 4 Ra r2 4 Rb Ra Ntese que en este problema hemos supuesto que es la misma en cualquier punto del material. En el caso de la atmsfera terrestre la resistividad vara con la altura como se explica en la Unidad Didctica, y el clculo de la resistencia total se hace ms complejo.

67

Mara Vir. Martnez Gonzlez

Combinacin de resistencias
Resistencias en serie
Rserie = R1 + R2 + ... + Rn

Resistencias en paralelo
1 1 1 1 = + + ... + Rpara R1 R2 Rn Ejercicio 4.- Calcular la resistencia equivalente a 3 resistencias R1 , R2 y R3 en paralelo. Solucin: 1 1 1 R2 R3 + R1 R3 + R1 R2 1 = + + = Rpara R1 R2 R3 R1 R2 R3 R1 R2 R3 R2 R3 + R1 R3 + R1 R2 Ntese que el numerador es el producto de las tres resistencias y el denominador es una suma de todos los productos posibles de dos resistencias. Rpara = Ejercicio 5.- Demostrar que la resistencia equivalente de n resistencias en paralelo es menor que cualquiera de las resistencias individuales. Solucin: Procediendo como en el ejercicio anterior 1 1 1 1 1 = + + + ... + = Rpara R1 R2 R3 Rn R2 R3 ...Rn + R1 R3 ...Rn + R1 R2 ...Rn + ... + R1 R2 R3 ... = R1 R2 R3 ...Rn es decir R1 R2 R3 ...Rn Rpara = R2 R3 ...Rn + R1 R3 ...Rn + R1 R2 ...Rn + ... + R1 R2 R3 ... Dividiendo numerador y denominador por R2 R3 ...Rn resulta R1 < R1 Rpara = 1 + (R1 /R2 ) + (R1 /R3 ) + ... + (R1 /Rn ) y un razonamiento anlogo es vlido para cualquiera de las resistencias. Ejercicio 6.- Sean tres resistencias iguales de valor R. Cuando las resistencias estn conectadas en paralelo en un circuito alimentado por un generador que proporciona una diferencia de potencial V, la potencia disipada es P = 45 Js1 . Cul ser la potencia disipada si las resistencias se conectan en serie? Solucin: La resistencia equivalente a tres resistencias iguales en paralelo es Rpar = R3 /(3R2 ) = R/3. As pues V2 3V 2 = 45 J s1 = Ppara = Rpar R Cuando las resistencias estn en serie Rserie = 3R y entonces Ppara V2 V2 = = 5 J s1 Pserie = = Rserie 3R 9 Ejercicio 7.- Tenemos 3 resistencias R1 , R2 = 2R1 y R3 = 4R1 . Cuando las 3 resistencias se conectan en paralelo de modo que entre sus extremos hay una diferencia de potencial V = 20 V , la potencia total disipada es P = 100 Js1 . Calcular los valores de las resistencias y la intensidad de corriente que atraviesa cada una de ellas. Solucin: De acuerdo con el problema anterior, la resistencia equivalente es 3 R1 R2 R3 8R1 4R1 = Rpara = 2 2 + 8R2 = 7 R1 R2 + R1 R3 + R2 R3 2R1 + 4R1 1 La potencia total disipada ser entonces es decir

68

Mara Vir. Martnez Gonzlez

P = de modo que

V2 7V 2 = Rpara 4R1

7 400 V2 7V 2 = =7 4P 4 100 J s1 La intensidad I1 que atraviesa la resistencia R1 ser V 20 A I1 = = R1 7 Asimismo 10 I1 R2 = 2R1 = 14 = A y I2 = 2 7 R1 = R3 = 4R1 = 28 y I3 = 5 I1 = A 4 7

Ejercicio 8.- Tenemos un circuito elctrico consistente en dos resistencias R1 y R2 en paralelo seguidas de una resistencia R3 en serie. Si entre los extremos del circuito hay una diferencia de potencial V, qu intensidad pasar por la resistencia R1 ? Solucin: La resistencia equivalente a las resistencias R1 y R2 en paralelo es R12 = R1 R2 /(R1 + R2 ). Entonces, la resistencia total del circuito es R1 R2 R1 R2 + R1 R3 + R2 R3 + R3 = RT = R1 + R2 R1 + R2 de modo que la intensidad total que circula es V V (R1 + R2 ) IT = = RT R1 R2 + R1 R3 + R2 R3 Esta intensidad total atraviesa la combinacin de R1 y R2 en paralelo. Al hacerlo se divide en dos intensidades I1 e I2 que atraviesan cada una de las resistencias R1 y R2 , de modo que la diferencia de potencial entre los extremos de ambas es la misma. As IT R2 V R2 IT = I1 + I2 I1 = = I1 R1 = I2 R2 R1 + R2 R1 R2 + R1 R3 + R2 R3

Carga y descarga de un condensador


Ejercicio 9.- Entre las placas de un condensador plano-paralelo de capacidad C hay una diferencia de potencial V0 . En el instante t = 0 se conectan las placas delcondensadormediante un hilo conductor de resistencia R, con lo que se forma un circuito cerrado RC. Qu intensidad de corriente circular por el circuito? Cmo disminuye la diferencia de potencial entre las placas del condensador y la carga elctrica acumulada en las mismas? Solucin: Si inicialmente hay una diferencia de potencial V0 entre las placas del condensador, la carga inicial en las placas es Q0 = CV0 (positiva en una y negativa en la otra). Al conectar el condensador al circuito, pasarn cargas de una placa a otra y por el hilo conductor circular una corriente. En un instante t posterior la carga en las placas sera Q(t) y la diferencia de potencial entre las mismas ser V (t) = Q(t)/C. Y de acuerdo con la ley de Ohm, la intensidad de la corriente que circula por el hilo en dicho instante es V (t) I (t) = R La intensidad es la carga que atraviesa una seccin del conductor por unidad de tiempo. Pero si una carga Q atraviesa el conductor, la carga en la placa positiva habr cambiado en una cantidad Q. Por lo tanto podemos escribir I (t) = dQ(t)/dt, siendo Q(t) la carga en la placa. Entonces la ecuacin anterior queda Q(t) dQ(t) 1 dQ(t) = o = Q(t) dt RC dt RC Y a hemos visto antes ecuaciones de esta misma forma, en que la variacin temporal de una magnitud es proporcional al valor de dicha magnitud. (Por ejemplo, en el material complementario del Captulo 1 acerca del movimiento

69

Mara Vir. Martnez Gonzlez

en un medio viscoso.) La solucin es Q(t) = Q(0) exp (t/RC ) = Q0 exp (t/RC ) o V (t) = V0 exp (t/RC ) y as = RC es la constante de tiempo asociada al condensador. Es decir, al cabo de un tiempo tc = , la carga en las placas y la diferencia de potencial se ha reducido en un factor 1/e. Ejercicio 10.- Se conectan los extremos de un condensador cargado de capacidad C = 1000 F a una resistencia R = 5 k. Cuando se inicia el proceso de descarga circula una corriente I = 8 mA. Cul es la constante de tiempo del circuito? Cul ser la intensidad de la corriente tras un intervalo de 3 constantes de tiempo? (Un dato til es e3 ' 20) Solucin: Del ejercicio anterior se deduce I (t) = I (0)et/RC La constante de tiempo es as = RC = 5 103 (103 106 ) F = 5 s. Para t = 5 resulta 1 A = 4 104 A I (5 ) = I0 e3 = 8 103 20 Ejercicio 11.- Considermos un circuito RC como el del ejercicio 9 con el condensador inicialmente descargado. En el instante t = 0 se conecta en el circuito un generador de corriente de fuerza electromotriz . Cmo vara la carga en el condensador? Cunto vale la carga al cabo de una constante de tiempo = RC ? Solucin: En el instante inicial, el condensador est descargado y la diferencia de potencial entre sus extremos es cero. La intensidad que circula por el hilo conductor es entonces I (0) = /R. A partir de entonces el condensador comienza a cargarse: en un instante t tendr una carga Q(t) y entre sus extremos habr una diferencia de potencial V (t) = CQ(t). La diferencia de potencial entre los extremos del hilo conductor es entonces V (t) , y la intensidadque circula es V (t) I (t) = R y procediendo de la misma forma que en el ejercicio 9 Q(t) C = A exp(t/RC ) siendo A una constante de integracin. Para calcularla no hay ms que tener en cuenta que para t = 0 tenemos Q(0) = 0, de modo que C = A. Entonces

Q(t) = C [1 exp(t/RC )] Por lo tanto, la carga en el condensador tiende asintticamente al valor C cuando el tiempo tiende a infinito. Al cabo de una constante de tiempo, la carga vale 1 e1 = C Q( ) = C 1 e e

70

Mara Vir. Martnez Gonzlez

CAPTULO 9
Campo magntico
~ ejerce sobre una carga q que se mueve con velocidad ~ Un campo magntico B v una fuerza dada por ~ ~ F = q~ vB (1) donde denota el producto vectorial. Por lo tanto, la fuerza es perpendicular a la trayectoria de la carga y no realiza trabajo sobre la misma. Si las cargas que se mueven pertenecen a un conductor, la fuerza del campo magntico sobre las cargas se traduce en una fuerza neta sobre el conductor. Como se ha estudiado en el captulo 7, si por un conductor rectilneo de longitud l y seccin S circulan cargas q a una velocidad v, la intensidad de la corriente es I = qnvS, siendo n el nmero de cargas por unidad de volumen. As, el nmero total de cargas en el conductor es nSl y la fuerza total sobre ellas ser ~ = (nSl) q~ ~ = I (~ ~) F vB lB donde ~ l es un vector de longitud l dirigido a lo largo del conductor. Ejercicio 1.- Una varilla metlica de longitud l = 1 m se mueve en un plano YZ a una velocidad ~ v = v~ ey ~ de mdulo v = 10 m/s. En la regin en que se mueve la varilla existe un campo magntico B = B~ ex , de mdulo B = 8 103 T. Calcular la diferencia de potencial entre los extremos de la varilla en los casos: a) la varilla forma un ngulo = 0 con el eje Z; b) la varilla forma un ngulo = 60o con el eje Z. Solucin: ~ = qvB~ ~m = q~ vB ez , con una componente Sobre cada carga q de la varilla acta una fuerza magntica F qvB cos a lo largo de la varilla. Si entre los extremos de la varilla hay una diferencia de potencial V, habr en cada punto de la varilla un campo elctrico E = V /l dirigido a lo largo de la varilla. En la situacin de equilibrio estas fuerzas se cancelan y V = qvB cos V = vBl cos = 8 102 cos V q l de modo que V = 0, 08 V para = 0 y V = 0, 04 V para = 60o . Ejercicio 2.- Una corriente de 2,0 A circula por una espira cuadrada de lado l = 20 cm. La espira est colgada en un campo magntico horizontal de mdulo B = 0, 1 T. a) Encontrar la fuerza que acta sobre los 4 lados de la espira cundo la direccin normal al plano de la misma forma un ngulo de 45 con la direccin del campo. b) Calcular el momento M de las fuerzas respecto a un eje vertical que pasa por el centro de la espira. Solucin: La situacin que describe el enunciado se muestra en la Figura 9.6 de la Unidad Didctica, y una explicacin ~ ~ ~ se da en las pginas 208-9. La fuerza magntica sobre un conductor es F = l I B . Las fuerzas sobre los lados horizontales de la espira son verticales, del mismo valor pero de sentido opuesto en cada lado. Por lo tanto, estas fuerzas non da lugar a ningn momento. Las fuerzas sobre los lados verticales tienen un valor Fvert = lIB y sentidos opuestos, pero sus lneas de aplicacin estn separadas una distancia d = l cos 45 = l 2/2. Por lo tanto, el momento de las fuerzas es 2 2 = 2 A 0, 1 T 4 102 m2 = 4 2 103 N.m = Fvert d = IBl2 2 2

Teorema de Ampre
La circulacin del campo magntico a lo largo de una curva cerrada es proporcional a la intensiad de la corriente que atraviesa una superficie limitada por la curva. I ~ d~ (2) B l = 0 I

A partir de este teorema es fcil obtener la expresin para el campo magntico en situaciones con simetras sencillas.

71

Mara Vir. Martnez Gonzlez

Ejercicio 3.- Calcular el campo magntico creado por un conductor rectilneo infinito de radio R por el que circula una corriente elctrica de intensidad I y densidad uniforme en la seccin del conductor. Solucin: Dada la simetria cilndrica del problema, las lneas de campo elctrico son circunferencias perpendiculares al conductor y con centro en el eje del mismo. Para una circunferencia de radio r, la circulacin del campo es B (r) 2 r. Poe otra parte, el mdulo de la densidad de corriente es j = I/( R2 ), y la intensidad de corriente que atraviesa el rea limitada por la circunferencia de radio r es I (r/R)2 si r < R I (r) = j r2 = I si r > R Por lo tanto i) r < R r 2 I B (r ) = 0 2 r B (r) 2r = 0 I (r) = 0 I R 2 R ii) r > R I (3) B (r) = 0 B (r) 2 r = 0 I 2r Ejercicio 4.- Una corteza cilndrica hueca infinitamente larga de radio interior a y radio exterior b transporta una corriente de intendidad I uniformemente distribuida en toda la seccin transversal de la corteza. Determinar el campo magntico a distancias r < a, a < r < b y r > b. Solucin: La seccin transversal de la corteza cilndrica es una corona circular de radio interno a y radio externo b, de modo que la intensidad I que la atraviesa est uniformemente distribuida sobre una superficie S = (b2 a2 ). Distinguimos ahora 3 casos i) r < a. En este caso, la circunferencia de radio r est en el hueco central de la corteza esfrica por el que no pasa corriente. Entonces I (r) = 0 y, por lo tanto, B = 0. ii) a < r < b. Ahora el rea encerrada por la circunferencia por la que pasa intensidad es (r2 a2 ), y la intensidad encerrada ser entonces Ir = I (r2 a2 )/(b2 a2 ). As r2 a2 0 I r2 a2 B ( r ) = B (r)2 r = 0 I 2 b a2 2r b2 a2 iii) r > b. Ahora la circunferencia encierra a toda la intensidad, y por lo tanto I B (r)2 r = 0 I B (r) = 0 2r Es fcil ver que los casos i) y iii) son casos partculares de ii) cuando r = a y r = b, respectivamente. El campo creado por un solenoide infinito de radio R y n espiras por unidad de longitud por el que circula una intensidad I es paralelo al eje del solenoide y su valor es (ver la demostracin en la Unidad Didctica) 0 nI r<R B= 0 r>R Ejericio 5.- Tenemos 3 solenoides infinitos con un mismo eje, radios R1 , R2 = 2R1 y R3 = 3R1 , respectivamente, y n1 , n2 = n1 /2 y n3 = n1 /3 espiras por unidad de longitud, respectivamente. Por los 3 solenoides circula una corriente de la misma intensidad I pero la que circula por el solenoide 2 lo hace en sentido contrario que la que circula por los solenoides 1 y 3. Calcular el campo magntico total en un punto situado a una distancia r del eje de los solenoides en los casos a) r < R1 ; b) R1 < r < R2 , c) R2 < r < R3 , y d) r > R3 . Solucin: a) si r < R1 , el punto est dentro de los 3 solenoides, de modo que el campo total es la suma de los campos debidos a los tres solenoides, es decir B = 0 n1 I + 0 n2 (I ) + 0 n3 I = 0 I (n1 n2 + n3 ) = 5 = 0 In3 (3 3/2 + 1) = 0 n3 I 2 y estar dirigido a lo largo de la direccin del eje del solenoide. b) si R1 < r < R2 , el punto est fuera del solenoide 1, de modo que el campo debido a este solenoide es nulo. Entonces tenemos simplemente

72

Mara Vir. Martnez Gonzlez

= 0 n2 (I ) + 0 n3 I = 0 I (n3 n2 ) = 1 = 0 In3 (1 3/2) = 0 n3 I 2 c) si R2 < r < R3 , el punto est fuera de los solenoides 1 y 2 y el campo total es B = 0 In3 = 0 In3 d) Finalmente, para r > R3 , el punto est fuera de los tres solenoides y el campo total es nulo. B Ejercicio 6.- Se quiere construir un solenoide da longitud L = 40 cm y seccin circular de dimetro 2R = 40 mm utilizando un hilo conductor cuya seccin tiene un dimetro 2r = 0, 5 mm. Qu diferencia de potencial hay que aplicar entre los extremos del solenoide para que el campo magntico en el interior sea B = 2, 0 102 T? (La resistividad del hilo es = 2, 0 108 m.)

Solucin: El campo magntico en el interior del solenoide es B = 0 nI , siendo n el nmero de espiras por unidad de longitud e I la intensidad que circula por el mismo. Para que circule una intensidad I habr que aplicar una diferencia de potencial V = I R, siendo R la resistencia total del hilo. Entonces BR V V = B = 0 nI = 0 n R 0 n Calculemos ahora la resistencia total del hilo de la bobina. El solenoide tiene nL espiras y cada una de las espiras de la bobina tiene una longitud l = 2 R, de modo que la longitud total del hilo es nLl = nL 2R. La seccin transversal del hilo es r2 . La resistencia total es entonces 2nLR nL2R = 2 R= r2 r En definitiva B 2nLR 2RBL 2 = V = 0 n r 0 r2 y sustituyendo valores 4 102 m 2 108 m 2 102 T 4 101 m 64 V = 81, 5 V = V = 2 7 2 3 2 0 , 25 (4 10 ) N/A (0, 25 10 ) m

Fuerzas entre corrientes

De la combinacin de (1) y (3), y la definicin de intensidad de corriente, se puede deducir que entre dos conductores rectilneos paralelos, de longitud l y separados una distancia R, por los que circulan corrientes de intensidades I1 y I2 , se ejerce una fuerza ~ = 0 lI1 I2 (4) F 2 R La fuerza es atractiva si ambas corrientes tienen el mismo sentido y respulsiva si tienen sentidos contrarios. (Puede verse la demostracin en la seccin 9.3 de la Unidad Didactica. Hay que advertir, no obstante, que en la frmula (9.6) hay una errata y sobra el factor dl1 .) Ejercicio 7.- Dos cables conductores, cuya masa por unidad de longitud vale = 0, 1 kg/m, se mantienen paralelos y horizontales colgando de hilos de sustentacin aislantes de longitud l = 0, 5 m. La distancia entre los hilos de sustentacin es s = 1 m. Cuando por los cables no pasa corriente los hilos de sustentacin estn en posicin vertical y los cables conductores estn a la misma altura y separados por una distancia d = 1 m. Sin embargo, cuando por los conductores circula una intensidad I, los conductores se atraen y los hilos de sustentacin se inclinan un ngulo respecto a la vertical. Para qu valor de la intensidad el ngulo de inclinacin ser de 30o ? Solucin: Si los hilos de sustentacin se inclinan un ngulo , la distancia entre los cables conductores ser d 2l sin . Cada hilo de sustentacin soporta un peso sg . Si T es la tensin del hilo, el equilibrio de fuerzas verticales sobre los cables conductores es T cos = sg

73

Mara Vir. Martnez Gonzlez

La fuerza atractiva entre una longitud s de los cables es 0 sI 2 F = 2 (d 2l sin ) y el equilibrio de fuerzas horizontales es entonces 0 sI 2 2T sin = 2 (d 2l sin ) Eliminando T entre estas ecuaciones resulta 0 I 2 2 tan = 2 (d 2l sin ) g o o Para = 30 tenemos sin 30 = 1/2, tan 30o = 1/ 3, y as 4 (d l)g 0, 5 m 0, 1 kg m1 9, 8 m s2 2 0 I 2 = I2 = = = 2, 8 106 A2 2 (d l) g 3 30 3 107 N A2 y as I = 1, 7 103 A. (Evidentemente esta es una intensidad muy grande para un cable conductor. La razn de que salga este valor es que en el enunciado se han dado unos valores de d, l, y que permiten simplificar mucho los clculos.)

Movimiento de cargas en un campo magntico


Como ya se ha dicho al inicio, sobre una partcula de carga q que se mueve con velocidad ~ v sometida a ~ acta una fuerza F ~ = q~ ~ Esta fuerza es perpendicular a la trayectoria y no realiza un campo magntico B v B. trabajo sobre la partcula. Por lo tanto, la energa cintica, y por consiguiente la velocidad de la partcula, permanece ~ , la aceleracin de la partcula es una aceleracin normal constante. Si ~ v se encuentra en un plano perpendicular a B de mdulo constante contenida en dicho plano. La partcula describe entonces una trayectoria circular de radio R con una aceleracin de valor aN = v 2 /R y as mv v2 R= (5) m = qvB R qB La velocidad angular es qB v = = R m y el ngulo girado en un tiempo t es = t. Entonces las expresiones para las coordenadas cartesianas de la partcula son qB qB t y (t) = R sin t x(t) = R cos m m donde hemos supuesto que para t = 0 las coordenadas de la partcula son (x0 , y0 ) = (R, 0). ~ = B~ Una demostracin ms rigurosa es la siguiente: supongamos un campo magntico B ez y una partcula ~ La fuerza que acta sobre la partcula es con una velocidad ~ v cualquiera, no necesariamente perpendicular B. ~ = qvy B~ ex qvx B~ ey (6) F de modo que por la segunda ley de Newton dvx dvy dvz m = qBvy = qBvx =0 (7) m m dx dy dz La tercera de estas ecuaciones dice que vz = cte. y as z = vz t. Por otra parte, de la primera y la segunda ecuaciones se obtiene 2 qB dvy qB d2 vx = = vx dx2 m dy m 2 qB dvx d2 vy qB = = vy dy 2 m dx m Y a hemos visto repetidamente este tipo de ecuaciones en donde la derivada segunda de una funcin es proporcional al valor de la funcin con signo negativo. (V er, por ejemplo, el Ejercicio 13 del Captulo 2.) Su solucin es qB qB t + v0y sin t vx (t) = v0x cos m m

74

Mara Vir. Martnez Gonzlez

qB qB t + v0y cos t vy (t) = v0x sin m m siendo v0x , v0y los valores de las componentes X e Y de la velocidad en el instante t = 0. Entonces qB mv0y qB mv0x sin t cos t x(t) = qB m qB m qB mv0y qB mv0x cos t+ sin t y (t) = qB m qB m y as la proyeccin de la trayectoria en el plano XY satisface 2 2 m 2 v0x + v0 x2 + y 2 = y qB que es una circunferencia de radio q mv 2 + v2 con v = v0 R= x 0y qB No obstante, la trayectoria no est, en general, en el plano XY perpendicular al campo porque en general habr una componente vz de la velocidad que se mantiene constante. En definitiva, la trayectoria ms general de la ~ es una trayectoria helicoidal. El eje de la helicoide es paralelo a partcula cargada sometida a un campo magntico B ~ y el radio de la helicoide es R = mv /qB. Adems, el paso de rosca de la helicoide es la direccin de B mvz 2 = 2 h = vz T = vz qB Ejercicio 8.- Tenemos dos partculas de la misma carga q y masas m1 y m2 = 2m1 . Ambas son aceleradas por una diferencia de potencial V , y entran, en el mismo instante t = 0, en una regin donde existe un campo magntico uniforme de intensidad B. a) Calcular la razn entre los radios de las trayectorias que siguen dentro de dicho campo. b) Cuando la velocidad de la partcula ms ligera forma un ngulo de 90 con la velocidad original, qu ngulo formarn entre s las velocidades de ambas partculas? Solucin: a) La velocidad que adquiere una partcula cuando atraviesa una diferencia de potencial V viene dada por el principio de conservacin de la energa r 2qV 1 2 mv = qV v= 2 m El radio de la trayectoria dentro del campo magntico es s 1 2mV mv = R= qB B q Entonces, puesto que las 2 partculas tienen la misma carga pero m2 = 2m1 , la razn entre los radios ser simplemente r R2 m2 = = 2 R1 m1 b) El ngulo girado en un tiempo t es qB t = t = m Por lo tanto, cuando la partcula ms ligera haya girado un ngulo de 90 , la partcula de masa doble habr girado la mitad, 45 , y el ngulo entre las dos direcciones ser tambin de 45 . Ejercicio 9.- Obtener una relacin entre el momento magntico de una espira de corriente circular y el momento angular de las cargas que circulan por dicha espira. Solucin: El razonamiento es similar al del ejercicio de la pgina 209 de la Unidad Didctica donde se calcula el momento magntico de una rbita electrnica, aunque ahora no hay cuantificacin. Supongamos que la espira esta formada por un cable conductor de seccin . Si n (que suponemos constante a lo largo de la espira) es el nmero de cargas libres por unidad de volumen, la intensidad que circula por la espira ser I = nqv

75

Mara Vir. Martnez Gonzlez

siendo v la velocidad de las cargas. El mdulo del momento magntico de la espira de radio R y rea S = R2 es entonces M = IS = nqv R2 El nmero total de cargas libres en la espira es N = n2 R y cada una de ellas tiene un momento angular mvR. Es decir, el momento angular total de las cargas que circulan es L = N mvR = (n2 R) (mvR) = n2 mvR2 Entonces q L M= 2m

CAPTULO 10
Ley de Faraday
El flujo de campo magntico a travs de un circuito cerrado es Z Z ~ dS ~ = BdS cos B = B

siendo el ngulo que forma la normal a la superficie con la direccin del campo. La integral se extiende a la superficie limitada por el circuito. Si el flujo vara en el tiempo, en el circuito se induce una fuerza electromotriz Z d dB ~ dS ~ = B (t) = dt dt El flujo puede variar debido a una variacin del campo, una variacin de la superficie limitada por el circuito o un cambio en la orientacin de la superficie respecto al campo. Casos particularmente simples son: ~ y constantes dB /dt = B (dS/dt) cos a) B b) S y constantes dB /dt = (dB/dt)S cos ~ y S constantes dB /dt = BS (d cos /dt) c) B Ejercicio 10.- Una espira circular de radio r = 10 cm esta hecha de un hilo de resistencia R = 0.5 y colocada perpendicularmente a un campo B = 3 101 T. En un intervalo de tiempo t = 3, 0 s cambia la forma de la espira para hacerse cuadrada. Cunto vale la intensidad media de la corriente durante dicho intervalo? Solucin: En este caso = 0 y B constante, de modo que B = B S. La longitud de la espira es l = 2 r, y el rea que encierra cuando tiene forma circular es Scir = r2 . Cuando la espira se hace cuadrada el rea encerrada es Scua = (2 r/4)2 = 2 r2 /4. Por lo tanto, la variacin del flujo magntico a travs de la espira es 2 r2 4 2 = B r2 B = B S = B r 4 4 La fuerza electromotriz inducida mientras la espira cambia de forma es entonces B = t y la intensidad que circula por la espira es B 4 B r2 = = = I = R Rt 4 Rt 1 2 2 3 (4 ) 4 3 10 T 10 m = 103 A = 4 0, 5 3 s 2 Ejercicio 11.- Una bobina de N = 200 espiras de radio r = 5 cm tiene una resistencia total de 10 . Un campo magntico perpendicular vara con el tiempo. Calcular cunto debe variar en 1 s para tener una corriente de intensidad I = 0, 2 A en la bobina. Solucin:

76

Mara Vir. Martnez Gonzlez

Para que por la bobina circule una corriente de intensidad I debe haber una fuerza electromotriz = IR. El ~ S ~ = B r2 , y el flujo magntico total a travs de la bobina flujo magntico que atraviesa cada espira de la bobina es B ~ S ~ = N B r2 . La variacin de flujo cuando el campo magntico cambia en B es M = B N r2 . es B = N B Entonces (suponiendo que el campo magntico vara a un ritmo constante) M B N r2 dM = = = dt t t y as IRt 0, 2 A 10 1 s 4 t = = = T B = N r2 N r2 200 25 104 m2 Ejercicio 12.- Una bobina de 25 vueltas, cada una de las cuales encierra un rea de 78 mm2 , est colocada en el interior de un solenoide de manera que sus ejes coinciden. Se vara la corriente del solenoide de manera que el campo magntico en su interior cambia liealmente desde +0, 15 T a 0, 15 T en 0,075 segundos. (a) Calcular el flujo B que atraviesa cada espira de la bobina en este intervalo de tiempo. (b) Calcular la fuerza electromotriz inducida en la bobina. Solucin: a) La variacin del campo magntico es B = 0, 15 T (0, 15) T = 0, 3 T , y la variacin del flujo magntico a travs de cada espira de la bobina es B = S B = 78 106 m2 0, 3 T = 2, 34 107 Wb b) La fuerza electromotriz inducida en la bobina es 25 B 25 2, 34 107 Wb dT otal = = = 7, 8 105 V = dt t 0, 075 s

Ejercicio 13.- Tenemos una espira circular de radio R hecha con un alambre conductor de resistividad y seccin a. La espira gira con velocidad angular en torno a un dimetro vertical dentro de un campo magntico ~ = B0~ ex . horizontal uniforme B a) Calcular la intensidad de corriente que circula por la espira en esta situacin. Comprobar, en particular, que la amplitud de la intensidad es proporcional al radio de la espira. b) Calcular el valor numrico para R = 5 cm, = 2, 0 108 m, a = 1 mm2 , B0 = 0, 5 T, = 4 s1 . Solucin: Este problema es prcticamente igual al problema 2 del Captulo 10 de la Unidad Didctica. ~ es uniforme y la espira es circular y de rea S = R2 , el flujo magntico Puesto que el campo magntico B a travs de la misma es ~ S ~ = B0 R2 cos =B siendo el ngulo que forma la normal a la espira con la direccin del campo. Puesto que la espira est rotando alrededor de un eje vertical con velocidad angular constante, tenemos = t (suponiendo que para t = 0 la espira est perpendicular al campo). Entonces = B R2 cos t y la fuerza electromotriz inducida en la espira es d = B0 R2 sin t = dt Por otra parte, puesto que el alambre que constituye la espira tiene longitud l = 2 R y seccin a, su resistencia total es 2 R l R= = a a de modo que la intensidad de la corriente que circula por la espira generada por la fuerza electromotriz inducida es B0 Ra B0 Ra I= = sin t = sin ( t + ) R 2 2 que es una corriente alterna de frecuencia y amplitud I0 = ( B0 a/2) R, proporcional a R. b) Para los valores dados 4 s1 0, 5 T 5 102 m 106 m2 = 2, 5 A = 7, 85 A I0 = 2 (2 108 ) m

77

Mara Vir. Martnez Gonzlez

Ejercicio 14.- Tenemos un circuito conductor cuadrado de lado l. En el mismo plano que contiene al circuito hay un conductor rectilneo infinito paralelo a uno de los lados y a una distancia l del mismo por el que circula una corriente de intensidad I. Calcular el flujo magntico que atraviesa el circuito cuadrado. Calcular la fuerza electromotriz inducida en el mismo si la corriente I que circula por el conductor rectilineo es de la forma I (t) = I0 sin t. Solucin: El campo magntico que produce la corriente que circula por el conductor rectilneo infinito es I B (r) = 0 2r Las lneas de este campo son crculos centrados en el conductor rectilneo y por lo tanto cortan perpendicularmente a la seccin del circuito cuadrado. Entonces Z Z Z 2l Il Il 2l dr ~ dS ~= = 0 ln 2 B (r) ldr = 0 B = B 2 l r 2 S l Si la corriente I es variable, la variacin del campo induce una fuerza electromotriz en el circuito d l ln 2 0 l dI (t) = = ln 2 = 0 I0 cos t dt 2 dt 2 Ejercicio 15.- Sean dos bobinas de 25 vueltas cada una y de radios r1 = 3 cm y r2 = 5 cm. Ambas bobinas son concntricas con un solenoide muy largo de radio R = 4 cm, de modo que la bobina grande rodea al solenoide mientras que la bobina pequea est dentro del mismo. Se vara la corriente del solenoide de manera que el campo magntico en su interior cambia linealmente de +0, 15 T a 0, 15 T en 0,075 segundos. (a) Calcular la variacin de flujo magntico B que atraviesa cada vuelta en cada una de las dos bobinas. (b) Calcular la fuerza electromotriz inducida en cada bobina. Solucin: El campo producido por un solenoide es uniforme dentro del mismo y nulo fuera del solenoide. Por lo tanto, si 2 . B es el valor del campo en el interior del solenoide, el flujo a travs de una espira de la bobina pequea es 1 = B r1 2 Sin embargo, el flujo a travs de una espira de la bobina grande es 2 = B R , porque para r > R el campo es nulo. Para obtener el flujo total en las bobinas hay que multiplicar por el nmero de espiras, es decir, por 25. La fuerza electromotriz inducida en cada bobina es entonces d1B 2 dB 2 B = 25r1 ' 25 r1 = 1 = dt dt t 0, 30 = 25 9 104 Wb = 0, 09 Wb 0, 075 2 R d2T 16 2 dB = 25R = 1 1 = 0, 16 Wb 2 = = dt dt r1 9

Autoinduccin
Un circuito por el que circula corriente crea un campo magntico proporcional a la intensidad de la corriente que circula por el mismo. Por consiguiente, habr un flujo de este campo magntico a travs del circuito que ser tambin proporcional a la intensidad. El coeficiente de proporcionalidad entre el flujo y la intensidad se llama inductancia = LI As, si vara la corriente que circula por el circuito, en el mismo se induce una fuerza electromotriz dI d = L = dt dt Ejercicio 16.- Una bobina de 300 vueltas posee una inductancia de L = 6 104 H. Qu valor tendr el flujo magntico que atraviesa cada vuelta de la bobina en el instante en que la intensidad es I = 1, 5 A? Si la corriente cambia con una rapidez de 0,2 A/s qu fuerza electromotriz se induce en la bobina? Solucin:

78

Mara Vir. Martnez Gonzlez

La inductancia L es la constante de proporcionalida entre el flujo magntico total a travs de la bobina y la intensidad que circula por la misma. As pues, el flujo magntico total para la intensidad dada es B = LI = 6 104 H 1, 5 A = 9 104 Wb y el flujo magntico a travs de cada vuelta de la bobina es B = = 3 106 Wb 300 La fuerza electromotriz inducida cuando vara la intensidad es dI = 6 104 H 0, 2 A s1 = 1, 2 104 V = L dt

Circuitos de corriente alterna


La ecuacin que satisface la intensidad de corriente que circula por un circuito de corriente alterna alimentado por un generador de fuerza electromotriz (t) = 0 cos t es (ver seccin 10.3 de la Unidad Didctica) Z 1 dI I (t)dt = 0 cos t L + RI + (8) dt C o derivando todos los trminos (y teniendo en cuenta que dQ/dt = I ) 1 d2 I dI L 2 + R + I = 0 sin t (9) dt dt C que es la ecuacin (10.9) de la Unidad Didctica. Esta ecuacin tiene la misma forma que la ecuacin para el desplazamiento de un oscilador armnico forzado (ecuacin (4.27) de la Unidad Didctica) dx d2 x + kx = F0 cos t m 2 +b dt dt (La diferencia entre sin t y cos t es simplemente una fase inicial.) Por ello, la intensidad en un circuito de corriente alterna vara de la misma forma que la elongacin en un oscilador armnico. Ejercicio 17.- Un condensador de capacidad C = 12 F se carga conectndolo a una fuente de tensin continua de 1000 V . Una vez cargado se desconecta de la fuente y se conectan sus extremos a los extremos de una bobina de inductancia L = 3 H. Hallar: (a) la frecuencia de las oscilaciones de corriente en el circuito as formado; (b) la intensidad mxima de la corriente en el circuito. Solucin: Este problema es similar al problema 5 del Captulo 10 de la Unidad Didctica. a) La carga que adquieren las placas del condensador al conectarlo a la fuente de tensin es Q = CV = 12 106 F 103 V = 12 103 C Si ahora se desconecta la fuente y se conectan los extremos del condensador a los de una bobina, la ecuacin (CA_2) con R = 0 y 0 = 0 queda d2 I (t) d2 I (t) I (t) 1 = 0 I (t) L + = 2 2 dt C dt LC que es formalmente anloga a la ecuacin de un oscilador armnico de frecuencia natural r r 1 1 1 = = 103 s1 0 = LC 3 H 12 106 F 6 . b) La intensidad de la corriente ser I (t) = I0 sin 0 t y la carga en las placas del condensador Z I0 cos 0 t = Q(0) cos 0 t Q(t) = I (t)dt = 0 La intensidad mxima es entonces Imax = 0 Qmax = 1/6 103 s1 12 103 C = 2 A Ejercicio 18.- Un circuito consta de una resistencia R = 2 y una bobina. En el instante t = 0 el circuito se conecta a una bateria de 12 V . Al cabo de un tiempo t = 0,002 s la corriente que circula por el circuito tiene una intensidad I = 4 A. Calcular la inductancia de la bobina. (Se supone que la resistencia de la bobina es despreciable.)

79

Mara Vir. Martnez Gonzlez

Solucin: La ecuacin (8) en ausencia del trmino correspondiente a la capacidad y para = 0 es R 0 dI dI + I= o L + IR = 0 dt dt L L Al conectar la batera empezar a pasar una corriente cuya intensidad crece lentamente debido al efecto de inercia de la bobina. La constante de tiempo caracterstica es = L/R. El crecimiento seguir hasta llegar a un estado estacionario en el que la intensidad tiene un valor constante I0 = 0 /R = 6 A. Es fcil ver que 0 I (t) = I0 1 et/ = 1 e(R/L)t R es solucin de la ecuacin. Entonces I (t) I0 = et/ I0 que para t = 0, 002 s da 0, 002 = ln 2 2 = e0,002/ y as 0, 004 0, 002R = Hz L = R = ln 2 ln 2

CAPTULO 11 Radiacin
Ondas electromagnticas
En ausencia de cargas libres y corrientes, las ecuaciones de Maxwell (ecuaciones (11.2) y (11.3) de la Unidad Didctica) toman la forma I I ~ ~ ~ dS ~=0 E dS = 0 B (10) I

S Z Z I S d d ~ dS ~ ~ dS ~ ~ d~ ~ d~ B E (11) E l= B l = 0 0 dt dt No hay que confundir las integrales de superficie que aparecen en (10) con las que aparecen en (11). Las primeras se extienden a una superficie cerrada que encierra un volumen y las segundasse extienden a una superficie limitada por una curva cerrada. De estas ecuaciones se pueden deducir ecuaciones para los campos E y B que tienen la forma 2E 2B 2B 2E = 0 0 2 = 0 0 2 (12) 2 2 x t y t Comparando estas expresiones con la frmula (5.13) del Captulo 5 vemos que se tratan de ecuaciones de ondas que se propagan con una velocidad v = 1/ 0 c0 , que es habitual designar por c. En realidad, estas ecuaciones son un caso particular correspondiente a ondas planas que se propagan en la direccin del eje Z. La forma de las soluciones de estas ecuaciones se ha visto tambin en el Captulo 5. y se explican en la seccin 11.3 de la Unidad Didctica. Adems, las ondas son transversales, es decir, tanto E como B oscilan perpendicularmente a la direccin de propagacin. Por ello, pueden estar polarizadas como se explica en la seccin 11.4.

Ejercicio 19.- La potencia de la radiacin solar que llega a la Tierra es WT = 1, 35 103 Wm2 . Suponiendo que la radiacin est constituida por una onda plana calcular la intensidad de los campos elctrico y magntico de dicha onda. Solucin: La densidad de energa transportada por una onda plana por unidad de tiempo es (ver seccin 11.3 de la Unidad Didctica) WT = c0 E 2

80

Mara Vir. Martnez Gonzlez

siendo E la amplitud del campo elctrico de la onda. Por lo tanto 1/2 1/2 1, 35 103 W m2 WT = = E = c0 3 108 m s1 8, 854 1012 C2 N2 m2 1/2 = 0, 5 106 V m1 ' 0, 7 103 V m1 La amplitud del campo magntico es entonces 0, 7 103 V m1 E = B= = 2, 3 106 T c 3 108 m s1

Radiacin de cuerpo negro


La radiacin de cuerpo negro satisface dos leyes fundamentales: Ley de desplazamiento de Wien max T = 0, 29 cm K Ley de Stefan-Boltzmann R(T ) = T 4 Ejercicio 20.- Un cuerpo negro se encuentra inicialmente a una temperatura Ti tal que la energa que irradia tiene un mximo en una longitud de onda i = 6 500 . Se calienta el cuerpo hasta que su intensidad de radiacin total se duplica. Cul es la temperatura final Tf ? A qu corresponde el nuevo mximo de radiacin? Solucin: Segn la ley de Stefan-Boltzmann, la energa radiada por unidad de rea por un cuerpo negro a una temperatura Ti es R(Ti ) = Ti4 (Para obtener la energa total radiada basta multiplicar por el rea total del cuerpo). Si para una temperatura Tf la energa radiada se ha duplicado (mantenindose el rea del cuerpo constante), entonces
4 = 2 Ti4 Tf = 21/4 Ti Tf Entonces, de acuerdo con la ley de desplazamiento de Wien i 6500 Ti i Ti = f Tf f = i = 1/4 ' Tf 1, 2 2

V emos as que para calcular la nueva longitud de onda a la que se emite el mximo de radiacin no es necesario calcular las temperaturas exactas, pues basta con conocer la razn entre ellas. Si queremos calcular Ti y Tf explcitamente necesitamos conocer el valor de la constante de Wien 0, 29 cm K i Ti = 0, 29 cm K Ti = ' 4460 K 6500 Ejercicio 21.- El Sol emite actualmente con un mximo de emisin a una longitud de onda i = 4800 . A medida que se agote el combustible nuclear (hidrgeno) su mximo de emisin se desplazar hasta f = 8000 y se expandir hasta que su radio se multiplique por 100. Cul ser entonces la temperatura del Sol? Cul ser la radiacin total emitida entonces por el Sol comparada con la radiacin actual? Solucin: De acuerdo con la ley de desplazamiento de Wien, la temperatura actual del Sol es 0, 29 cm K ' 6000 K Ti = i Ti = 0, 29 cm K 4800 108 cm y de acuerdo con la ley de Stefan-Boltzmann, la radiacin total emitida por el Sol actualmente es Ri = Ti4 Si siendo Si la superficie del Sol actual. La temperatura final Tf ser entonces i i Ti = f Tf = cte Tf = Ti = 0, 6Ti = 3600 K f

81

Mara Vir. Martnez Gonzlez

y puesto que al multiplicarse el radio por 100, la superficie se multiplica por 1002 , la radiacin total final ser 4 Rf = Tf Sf = (0, 6)4 Ti4 104 Si = 64 Ri Ejercicio 22.- Una estufa esfrica emite radiacin de cuerpo negro de forma istropa. Una persona que inicialmente estaba a una distancia r se aleja hasta una distancia 3r. Cunto debe aumentar la temperatura de la estufa para que le llegue la misma cantidad de radiacin? Cunto habr aumentando la energa radiada por la estufa? Solucin: Si R es la potencia total radiada por la estufa, la potencia recibida a una distancia r es R/(4 r2 ). Por lo tanto, para que llegue a una distancia 3r la misma potencia que llegaba antes a una distancia r, debe cumplirse Rf Ri = Rf = 9Ri 2 4 r 4 (3r)2 Y puesto que la radiacin es proporcional a la cuarta potencia de la temperatura, tenemos 4 Tf = 9Ti4 Tf = 91/4 Ti = 1, 73Ti

82

Mara Vir. Martnez Gonzlez

Bloque temtico 4: Termodinmica


Todos los cuerpos estn compuestos de partculas individuales. En el Bloque Temtico 1 se vio que la descripcin de un sistema de partculas requiere especificar las coordenadas y las velocidades de cada partcula. Esto, para un cuerpo que contiene del orden de 10^23 partculas es evidentemente imposible. Por ello, para describir un sistema semejante se introducen unas magnitudes termodinmicas que son promedios estadsticos de las magnitudes mecnicas. As, la presin que ejerce un gas sobre las paredes del recipiente que lo contiene es un promedio estadstico de la fuerza que ejercen los tomos que chocan con una unidad de rea de la pared en una unidad de tiempo. Este promedio depende del nmero de tomos que chocan y del momento lineal que lleva cada uno. Puede verse entonces que la presin est relacionada con la energa cintica media de las partculas. Si ahora se define una temperatura como promedio de la energa cintica, obtenemos finalmente una ecuacin que liga la presin, la densidad y la temperatura, que se denomina ecuacin de estado. Un modelo sencillo que conduce a una ecuacin de estado muy simple es el modelo del gas perfecto. En este modelo se supone que los tomos no interaccionan y que toda la energa del gas es energa cintica, lo que da lugar a la ecuacin de estado PV=nRT. En general, sin embargo, las partculas interaccionan a travs de una energa potencial de interaccin, y la energa mecnica total del gas debe incluir tambin esta energa. Esta energa total se denomina energa interna U. En un gas ideal la energa interna solo depende de la temperatura. En otros sistemas la energa interna puede depender de otros parmetros, adems de la temperatura. Podemos caracterizar un material viendo que cambios de temperatura provoca un incremento o disminucin de la energa interna. Se define as una capacidad calorfica del medio. No obstante, la capacidad calorfica depende de en qu condiciones se haga la transferencia de energa. Cuando la transferencia se hace manteniendo constante el volumen del sistema, obtenemos una capacidad calorfica a volumen constante. Recprocamente, cuando la transferencia de hace manteniendo constante la presin, obtenemos una capacidad calorfica a presin constante. Una consecuencia de la interaccin entre los tomos a travs de un potencial de interaccin es que para altas temperaturas la distancia media entre los tomos aumenta y los cuerpos se dilatan. Este efecto se puede caracterizar mediante unos coeficientes de dilatacin. Cuando se ponen en contacto varios sistemas caracterizados cada uno de ellos por su presin y su temperatura existe una tendencia hacia un estado de equilibrio en el que las presiones y temperaturas se igualan. Esto se consigue gracias a la transferencia de energa interna entre los sistemas. Una parte de la energa transferida est ligada a la variacin de volumen de los sistemas (con lo que cambian las densidades) y otra parte es lo que se denomina transferencia de energa en forma de calor. Dos modos importantes de transmisin del calor son la conduccin y la radiacin. Las ecuaciones para la conduccin de calor son formalmente idnticas a las ecuaciones para la corriente elctrica. Por su parte, las ecuaciones para la radiacin son similares a las de la radiacin del cuerpo negro. La condicin de igualdad de presiones tiene una expresin especial en presencia de gravedad. En este caso, adems de la energa de interaccin entre los tomos de cada sistema, hay que considerar la energa potencial del campo gravitatorio externo. Por ello, la presin de equilibrio vara con la altura. La forma de esta variacin depende de la compresibilidad del medio, es decir, de cmo soporta cada seccin del medio el peso de las secciones que tiene encima. En un medio gaseoso la presin vara en forma exponencial. Por el contrario, en un medio lquido, poco compresible, la presin vara de forma proporcional a la altura: esto se traduce en el principio de Arqumedes y el principio de Pascal. El primer principio de la termodinmica afirma que la variacin de la energa interna U de un sistema puede tener lugar en forma de trabajo W (cuando hay un cambio de volumen) o en forma de calor Q (no ligada a variaciones de volumen). Es decir U=Q+W Una ecuacin de estado, en general, liga la presin, el volumen y la temperatura de un sistema termodinmico. Dando dos de estas magnitudes podemos obtener la tercera. Por lo tanto, podemos definir
83 Mara Vir. Martnez Gonzlez

el estado de un sistema dando la presin y el volumen, o dando la presin y la temperatura. Una transformacin que lleva un sistema de un estado termodinmico a otro es un proceso termodinmico. Un proceso termodinmico es cuasiesttico si consiste en una secuencia de estados de equilibrio. Por lo tanto, un proceso cuasiesttico puede representarse como una curva en un diagrama (P,V) o en un diagrama (P,T). Los diagramas (P,V) son especialmente interesantes ya que la interpretacin geomtrica de la integral permite interpretar el trabajo termodinmico como el rea encerrada por la curva que representa el proceso termodinmico en el diagrama. Por otra parte, hay una infinidad de procesos cuasiestticos que llevan de un estado de equilibrio dado a otro. Sin embargo, hay cuatro tipos de procesos especialmente importantes: a) Procesos a volumen constante (iscoros): en estos procesos no hay intercambio de energa en forma de trabajo: W=0 b) Procesos a presin constante (isbaros): el trabajo realizado es simplemente la presin por la variacin de volumen: W=PV c) Procesos adiabticos: no hay intercambio de energa en forma de calor: Q=0 d) Procesos isotermos: la temperatura se mantiene constante. Si el proceso isotermo tiene lugar en un gas perfecto, todos los estados de equilibrio por los que pasa el sistema tienen la misma energa interna. Por lo tanto, U=0 y Q=-W. Una mquina trmica es una mquina que trata de extraer energa en forma de calor y convertirla en trabajo. Sin embargo, el segundo principio de la termodinmica afirma que la conversin total de calor en trabajo es imposible. Una mquina trmica siempre debe trabajar entre dos focos trmicos: extrae calor de un foco a alta temperatura, convierte parte de dicho calor en trabajo y cede el resto a un foco a temperatura menor. La razn entre el calor convertido en trabajo y el calor total extrado del foco a alta temperatura es el rendimiento de la mquina que, como hemos dicho, debe ser necesariamente menor que 1. Las mquinas con mximo rendimiento se denominan mquinas de Carnot. Su rendimiento depende nicamente de las temperaturas de los focos trmicos entre los que trabaja. Los modelos sencillos como el del gas perfecto son necesariamente limitados. Las interacciones entre los componentes de una sustancia dependen de la presin, temperatura, etc., lo que hace que una sustancia con unos mismos componentes elementales pueda presentarse en varias fases o estados de agregacin. Por ejemplo, el agua puede presentarse en forma gaseosa (vapor de agua), lquida o slida (hielo, aunque dentro del hielo hay tambin varias fases distintas). Dos fases de una sustancia pura pueden coexistir a lo largo de una curva en un diagrama (P,T): es decir, para cada presin dada, existe un valor de la temperatura para la que pueden coexistir las dos fases. Por ejemplo, a presin atmosfrica, el agua lquida y el hielo pueden coexistir a 0C o 273,15 K. Por encima de dicha temperatura solo puede haber agua lquida, y por debajo solo puede haber hielo. De este modo, si partimos de hielo y vamos suministrando calor al sistema (siempre a presin atmosfrica) llega un momento en que el hielo empieza a fundirse y coexisten hielo y agua lquida. Mientras dura la fusin la temperatura permanece constante a 273,15 K, pese a que seguimos suministrando calor al sistema. Esto se debe a que el calor suministrado no se invierte en aumentar la temperatura sino en cambiar la estructura interna de la sustancia, es decir, en romper enlaces entre las molculas de agua en el hielo. Este calor, que no se manifiesta en un aumento de la temperatura, se denomina calor latente. Una vez que todo el hielo se ha transformado en agua lquida, la temperatura empieza a subir de nuevo. El calor latente est presente en cualquier cambio de fase. Normalmente, el paso de una fase ms ordenada (hielo) a otra menos ordenada (agua) implica una absorcin de calor latente por parte de la sustancia. Por el contrario, si la sustancia pasa de una fase menos ordenada a otra ms ordenada, es la sustancia la que desprende calor latente. Tres fases de una sustancia pura solo pueden coexistir para una presin y temperatura dadas (que, por supuesto, dependen de la sustancia). Estos valores determinan el llamado punto triple de la sustancia.
84 Mara Vir. Martnez Gonzlez

CAPTULO 12
Gases ideales
La ecuacin de estado de un gas ideal es P V = N kB T (1) siendo N el nmero de molculas de gas en el volumen V, y kB es la constante de Boltzmann. Alternativamente podemos escribir P V = nRT (2) donde n = N/NA es el nmero de moles de gas y R = kB NA se denomina constante universal de los gases. Las molculas de un gas ideal solo interaccionan mediante colisiones elsticas. Salvo esto no acta hay ninguna fuerza entre ellas y, por consiguiente, no hay ninguna energa potencial de interaccin. Toda la energa de un gas ideal es energa cintica de las molculas. As, llamando U a la energa interna del gas 1 (3) U = N hK i = N m v 2 2 2 siendo hK i = (1/2)m v la energa cintica media de las molculas del gas. La presin que ejerce un gas ideal sobre las paredes del recipiente que lo contiene est relacionada con la energa cintica media hK i de las molculas del gas mediante P = De las ecuaciones anteriores se deduce y hK i U= 2N hK i 3V (4) (5)

3 3 P V = nRT (6) 2 2 Nota: el valor de R en unidades del Sistema Internacional es R = 8, 314 J/(molK). No obstante, al tratar con gases es habitual utilizar unidades de atmsferas para la presin y litros para el volumen, de modo que tambin se utiliza un valor de R en atmlitro/K. En lugar de hacer la conversin directa de unas unidades a otras podemos utilizar el resultado del problema 2 del Captulo 12 de la Unidad Didctica donde se calcula que un mol de gas a presin atmosfrica y 0o C ocupa un volumen de 22,4 litros. Entonces 1 atm 22, 4 litros PV = = 0, 082 atm litro/K R= T 273, 15 K Ejercicio 1 .- Calentamos una olla a presin de 5 litros de capacidad con un fogn elctrico de 2000 W durante 5 minutos. Suponiendo que la olla est vaca (solo contiene aire en condiciones de temperatura y presin ambientes), calcular la presin en el interior de la olla al cabo de esos 5 minutos. Suponer que el aire se comporta como un gas ideal y que la olla pierde el 90% de la energa del fogn por radiacin. Solucin: En 5 minutos el fogn suministra una energa E = 2000 W 300 s = 6 105 J Sin embargo la olla pierde un 90% de esta energa, y solo un 10% va al aire del interior; es decir, la energa interna del aire aumenta en U = 0, 1E = 6 104 J. Considerando el aire un gas perfecto, se debe satisfacer la ecuacin P V = (2/3)U. Y puesto que el volumen de la olla es constante, la variacin de presin y la variacin de energa interna satisfacen la relacin P V = (2/3)U. As pues (2/3) 6 104 J (2/3)U = = 8 106 Pa P = V 5 103 m3 La presin inicial era la presin atmosfrica Patm = 101325 Pa, de modo que la presin final es Pf inal = Patm + P = 8.101.325 Pa Ejercicio 2.- Una caja cbica de 20 cm. de lado contiene aire a 1 atm y 300 K de temperatura. Se cierra la caja hermticamente y se calienta hasta que alcanza una temperatura de 400 K. Qu fuerza neta acta entonces sobre cada pared de la caja?

1 2 3 m v = kB T 2 2

85

Mara Vir. Martnez Gonzlez

Solucin: Puesto que la caja est cerrada hermticamente el nmero de moles en su interior no vara. Entonces Pf V 4 Pi V Tf = Pf = Pi = atm = 135.100 Pa n= RTi RTf Ti 3 Cada pared de la caja tiene un rea S = 0, 04 m2 , de modo que la fuerza que ejerce el gas sobre cada pared es F = P S = 135.100 Pa 0, 04 m2 = 5404 N Ejercicio 3.- En una vasija cbica de 1 m de lado tenemos oxgeno a 300 K. Comparar la energa cintica media de una molecula del gas con la variacin que experimentara su energa potencial si cayera desde la parte superior al fondo de la vasija. Solucin: De acuerdo con (5) 3 1, 38 1023 J K1 300 K 3 = 621 1023 J h K i = kB T = 2 2 La masa de una molcula es m = M/NA , siendo M la masa de un mol del gas. Entonces, la variacin de la energa potencial de la molcula es M 32 9, 8 m s2 1 m g h = = 52, 08 1023 J Ep = mg h = NA 6, 022 1023 de modo que hK i = 11, 92Ep . De (5) se deduce La cantidad 2 3kB T 3RT v = = m M r p 3RT vcm = hv 2 i = M

(7)

se denomina velocidad cuadrtica media. p En el Captulo 4 se ha visto que la velocidad del sonido en un gas (supuesto ideal) es vson = RT /M . As, pues la velocidad cuadrtica media de una molcula de un gas es del mismo orden que la velocidad del sonido en el gas. Por ejemplo, en el problema 4 del Captulo 12 de la Unidad Didctica se calcula una velocidad cuadrtica media de unos 480 m/s para una molcula de oxgeno a temperatura ambiente. Sin embargo, esto no quiere decir que en un segundo la molcula se haya desplazado a 480 metros de su posicin inicial. Si as fuera, tan solo un segundo despus de abrir un frasco de perfume en un punto se podra notar el olor a muchos metros de distancia. Lo que ocurre en realidad es que cada molcula est chocando continuamente con otras molculas y en cada choque cambia de direccin, de modo que la trayectoria de la molcula es una lnea quebrada y zigzagueante. Ejercicio 4.- Dado el valor ya citado para la velocidad cuadrtica media del oxgeno a temperatura ambiente, cul ser la velocidad cuadrtica media del hidrgeno a la misma temperatura? Solucin: De acuerdo con (7) s r M (O2 ) 32 vcm (H2 ) = = =4 vcm (H2 ) = 4vcm (O2 ) = 1920 m/s vcm (O2 ) M (H2 ) 2

Capacidad calorfica
El calor especfico c de una sustancia es la energa trmica que hay que aadirle para aumentar en un grado la temperatura de una unidad de masa de dicha sustancia. As, la energa trmica que hay que aportar a m gramos de sustancia para aumentar su temperatura en una cantidad T es U = mcT (8) La energa necesaria para aumentar en un grado la temperatura de un mol de sustancia es la capacidad calorfica o calor especfico molar C . As, la energa trmica necesaria para aumentar en T la temperatura de n moles de sustancia es

86

Mara Vir. Martnez Gonzlez

U = nC T (9) Se define la calora como la energa necesaria para aumentar la temperatura de 1 gramo de agua en 1 grado. Una calora equivale a 4,18 J. En otras palabras, el calor especfico del agua es c = 4180 Jkg1 K. En realidad, el calor especfico depende, en general, de la temperatura. Por ejemplo, la energa necesaria para aumentar la temperatura de un gramo de agua de 10o C a 11o C no es exactamente igual a la energa necesaria para aumentar la temperatura de 20o C a 21o C. La calora antes definida es la energa necesaria para calentar el agua de 4o C a 5o C. De la ecuacin (6) se desprende que la energa interna de un gas ideal depende nicamente de su temperatura. Entonces U = (3/2)nRT, que comparada con (9) da 3 C = R ' 12, 5 J/(mol K) 2 Es decir, el calor especfico molar de un gas ideal es constante, independiente de la temperatura. (En realidad ste es el calor especifico cuando la energa trmica se suministra manteniendo constante el volumen que ocupa el gas.) Ejercicio 5.- Una manera de pesar un volumen fijo de gas es viendo cunto aumenta su temperatura al suministrarle cierta cantidad de energa. Sabemos que al calentar cierta masa de O2 suministrndole 10 kJ manteniendo su volumen constate, la temperatura pasa de 10o C a 20o C. Cul es dicha masa? Solucin: De acuerdo con (9) nC T = U y por lo tanto la masa es n= 104 J U = = 80, 23 moles C T (3/2) 8, 31 J/(mol K) 10 K

m = nM = 80, 23 32 g = 2, 567 kg La capacidad calorfica molar de los slidos a temperatura embiente es C ' 3R = 24, 9 J/(molK), es decir, el doble de la capacidad calorfica de un gas ideal. Una explicacin simple para esto es la siguiente. En sus posiciones de equilibrio los tomos de un slido ocupan los nodos de una red cristalina. No obstante, a altas temperaturas los tomos realizan oscilaciones armnicas en torno a los puntos de equilibrio. En definitiva, un slido puede aproximarse por un conjunto de osciladores armnicos de energa 1 1 1 E = Ek + Ep = mv 2 (t) + kx2 (t) = kA2 2 2 2 Ahora bien 1 1 hEk i = m 2 A2 cos2 t y hEp i = m 2 A2 sin2 t 2 2 Pero cos2 t = sin2 t , de modo que la anega de cada oscilador es E = 2 hEk i , y la energa de NA osciladores es U = NA E = 2NA hEk i y admitiendo la relacin Ek = (3/2)kB T, como en (6), resulta 3 U = 2NA kB T = 3NA kB T = 3RT C = 3R 2

Dilatacin
El coeficiente de dilatacin lineal se define de modo que L = LT Asimismo, el coeficiente de dilatacin de volumen se define como V = V T Hay una relacin evidente entre y . Si suponemos un volmen cbico de lado L, y tratando los incrementos como diferenciales, V = L3 V = 3L2 L

87

Mara Vir. Martnez Gonzlez

y as V T = 3L2 LL = 3

Tambien los coeficientes y dependen en general de la temperatura. El caso ms notable es el del agua que tiene un coeficente < 0 entre 0o C y 4o C y un coeficiente > 0 para temperaturas mayores que 4o C. Ejercicio 6.- Cunto vale el coeficiente de dilatacin para un gas ideal? Solucin: De acuerdo con (1), si se calienta un gas ideal manteniendo la presin constante, el aumento de la temperatura y del volumen estn relacionados por P V = N kB T y as T 1 1 V = V = V T = V T T T Ejercicio 7.- Los rales en las vas del tren tienen unas juntas de dilatacin cada ciertos metros para que no se produzcan deformaciones en la va que pudieran dar lugar a descarrilamientos. Estas juntas son las que producen el traquetreo caracterstico del tren. Las juntas suelen tener unos 3 mm de ancho cuando la temperatura es de 20o C. Calcular a que distancia deben colocarse estas juntas para que puedan acomodar las temperaturas de casi 70o C que llegan a tener los rales en verano. (El coeficiente de dilatacin del material de las vas es = 1, 2 105 K1 ) Solucin: Si cada tramo de va entre juntas tiene una longitud L, la junta permite que se dilate una longitud L = 0,003 L = LT L= 3 103 m L = =5m T 1, 2 105 K1 50 K m. As

Ejercicio 8.- Supongamos que se construye un anillo metlico que cie perfectamente la Tierra a lo largo del ecuador cuando la temperatura es de 20o C. A qu distancia del suelo quedar el anillo cuando la temperatura aumente en 1o C? (Suponer de nuevo = 1, 2 105 K1 ) Solucin: Si el anillo cie exactamente la Tierra su longitud es la longitud del ecuador, es decir, Li = 40.000 km. Si la temperatura aumenta en T , la longitud final del anillo ser Lf = Li + Li = Li (1 + T ) de modo que la diferencia entre los radios final e inicial del anillo ser Li T Lf Li Rf Ri = = 2 2 Entonces para T = 1o C 4 107 m 1, 2 105 = 76, 4 m Rf Ri = 2

Ejercicio 9.- Un reloj de pndulo tiene un pndulo metlico con un coeficiente de dilatacin lineal = 1, 2 105 K1 Cmo vara el periodo del pndulo cuando vara la temperatura? De acuerdo con esto, cunto atrasara el reloj en un da si la temperatura aumenta en 10o C? Solucin Llamaremos al periodo del pndulo para distinguirlo de la temperatura T. Entonces el periodo vale 1/2 l = 2 g 1 l 1 = = T 2 l 2 = 6 105

y as

que para T = 10o C da

88

Mara Vir. Martnez Gonzlez

En un da, de duracin td = 86400 s el pndulo de periodo hace td / oscilaciones. Para hacer esas mismas oscilaciones, el pndulo de periodo + necesita un tiempo td ( + ) = td + td de modo que el retraso en un da es = 86400 s 6 105 = 5, 18 s td

Mezcla de gases ideales


Ley de Dalton: la presin total de una mezcla de gases ideales esla suma de las presiones que ejercera cada cada gas por separado.
Ejercicio 10.- Una mezcla de hidrgeno y nitrgeno a 500 K en un recipiente de 200 litros ejerce sobre las paredes del recipiente una presin de 5 atmsferas. La masa del nitrgeno contenido en el recipiente es de 300 gr. Cul es la masa del hidrgeno? Solucin: De acuerdo con la ley de Dalton (nN i + nHi ) RT P = PN i + PHi = V siendo nN i y nHi el nmero de moles de nitrgeno y de hidrgeno, respectivamente. Las masas molculares del nitrgeno y el hidrgeno son MN i = 34 g y MHi = 2 g, de modo que mHi mN i + nN i + nHi = 34 2 que sustituido en la ecuacin anterior da mHi PV mNi + = MN i MHi RT de la que podemos despejar fcilmente el valor que se nos pide de mHi mN i 300 g 5 atm 200 litros PV = 31, 14 g =2 mHi = 2 RT MN i 0, 082 atm litro/K 500 K 34 g

89

Mara Vir. Martnez Gonzlez

CAPTULO 13
Conduccin trmica
Existe un claro paralelismo entre la conduccin trmica y la conduccin elctrica que se ha estudiado en el Captulo 8. El papel que desempeaba la diferencia de potencial en la conduccin de carga elctrica lo desempea la diferencia de temperatura en la conduccin de calor. Tenemos as

V Rel

Conduccin elctrica = Rel Iel 1 l = S

Conduccin trmica T = Rter Iter 1 x Rter = k A

La resistencia elctrica Rel y la resistencia trmica Rter tienen la misma estructura: ahora el papel de la conductividad elctrica lo desempea la conductividad trmica k. Por lo dems, la combinacin de resistencias obedece a las mismas relas: las resistencias en serie se suman, mientras que en las resistencias en paralelo es la inversa de la resistencia total la que es suma de las inversas de las resistencias individuales. Ejercicio 1.- Una pared de hormign de una casa de 3 metros de alta, 6 metros de larga y 15 cm de grosor tiene una ventana de 3 m2 de cristal de 8 mm de grueso. En invierno, cuando la temperatura media exterior es de 0 o C, queremos mantener una temperatura media interior de 20 o C a base de calefactores elctricos. Al mismo tiempo, queremos gastar un mximo de 300 euros en nuestra factura mensual por lo que sospechamos que tenemos que aislar la pared. Es suficiente cubrir la pared con placas de 2 cm de ancho de poliestireno expandido o tenemos que poner tambin doble cristal en la ventana (que reduce las prdidas a la mitad? (Suponer que el tejado y el resto de las paredes de la casa estn perfectamente aislados.) Datos: conductividad trmica del homign 1 W/mK; conductividad del poliestireno expandido 0,033 W/mK; conductividad del cristal 0,8 W/mK, precio del kilovatio-hora (=3600 kJ) de energa elctrica 0,11 euros. Solucin: Evaluemos primero las prdidas de calor en ausencia de aislamiento. La pared de hormign y la ventana estn en paralelo, de modo que su resistencia trmica combinada es 1 1 khor Ahor kven Aven + = + = Rhor Rven xhor xven 1 W/(m K) 15 m2 0, 8 W/(m K) 3 m2 + = 400 W K1 = 0, 15 m 0, 008 m donde hemos tenido en cuenta que la superficie de hormign es la superficie total de la pared (18 m2 ) menos los 3 m2 de la ventana. Por lo tanto, la energa trmica perdida es T = 20 K 400 W K1 = 8000 W I= R Es decir, perdemos 8 kJ cada segundo. Un mes de 30 das tiene 30 86400 = 2 592 000 s, de manera que en un mes habremos perdido 20736000 kJ = 5760 kWhora. Esto nos cuesta 5760 0, 11 = 633, 6 euros. Si ahora recubrimos el hormign con 2 cm poliestireno, hormign y poliestireno estn en serie y as, la resistencia trmica de la pared es ahora xhor xpol + = Rpared = Rhor + Rpol = khor Ahor kpol Apol 0, 02 m 0, 15 m + = 0, 05 W1 K = 1 W/(m K) 15 m2 0, 033 W/(m K) 15 m2 y la resistencia trmica con la ventana en paralelo es ahora 1 1 1 = + = 20 W K1 + 300 W K1 = 320 W K1 R Rpared Rven 1 R =

90

Mara Vir. Martnez Gonzlez

de modo que la energa perdida ahora es T = 20 K 320 W K1 = 6400 W R As pues, con el revestimiento de poliestireno reducimos las prdidas en 1600 W, que en un mes son 1152 kWh, lo que supone un ahorro de 126,72 euros. I = Si, adems, ponemos doble cristal en la ventana, la resistencia trmica de la ventana se duplica y entonces 1 1 1 = + = 20 W K1 + 150 W K1 = 170 W K1 R Rpared 2Rven y T = 20 K 170 W K1 = 3400 W I = R con un ahorro adicional de 3000 W o 237,6 euros. En definitiva, recubriendo la pared de poliestireno y poniendo doble cristal en la ventana, la factura se reduce a 633, 6 126, 72 237, 6 = 269, 28 euros. Ejercicio 2.- En un modelo sencillo del interior de la Tierra, ste est dividido entre el manto slido terrestre, que es una bola de hierro fundido a 1500o C y de 3.400 km de radio. Suponiendo que la conductividad trmica del manto es 0,150 Wm1 K1 y que la temperatura media de la superficie de la Tierra es de 20o C, calcular la cantidad de calor que emana del interior de la Tierra al da y por metro cuadrado. El radio de la Tierra es de 6.300 km. Solucin: Tambin aqu hay un paralelismo con la conduccin elctrica en un conductor esfrico. Podemos considerar el interior de la Tierra dividido en cortezas esfricas de radio r y grosor dr. Si la diferencia de temperatura entre la superficie interior y exterior de la corteza es dT, la ecuacin (13.7) de la Unidad Didctica debe transformarse en dT dT = k 4r2 I = kA dr dr pues 4 r2 es el rea de la superficie interior de la corteza de radio r. Por otra parte, la intensidad de la energa trmica que atraviesa las diferentes cortezas es la misma, pues no hay acumulacin de calor en ninguna zona. Entonces I dr dT = 4 k r2 que se integra fcilmente I 1 1 Tb Ta = 4 k rb ra Es decir, la resistencia trmica de la corteza 1 1 1 Rter = 4k rb ra que tiene la misma forma que la resistencia elctrica de un conductor esfrico que se estudi en el Ejercicio 3 del material complementario del Captulo 8. Haciendo ra = 3400 km y rb = 6300 km Tb Ta 4 k (Tb Ta ) 4k (Tb Ta ) ra rb = I = = = 1 1 Rter ra rb
rb ra

K1 1480 K 3, 4 6, 3 1012 m2 = 2, 06 1010 W 2, 9 106 m 2 Esta intensidad repartida por los 4 rb = 4 6, 32 1012 m2 de la superficie de la Tierra da 75, 4 106 W/m2 . = 4 0, 15 W m

Radiacin
Ejercicio 3.- Tenemos dos paredes negras planas y paralelas, separadas por un espacio vaco. Una de las paredes est a una temperatura alta Tc y la otra a una temperatura ms fra Tf . Al cabo de un tiempo se establece un flujo de calor estacionario debido a la radiacin trmica. (a) Calcular este flujo. (b) Se coloca una lmina delgada de material negro en el espacio vaco entre las paredes y paralela a ellas. Calcular el nuevo flujo de calor. (c) Se colocan ahora dos de estas lminas en el espacio entre las paredes. Calcular el nuevo flujo de calor.

91

Mara Vir. Martnez Gonzlez

Solucin: 4 , donde a es el coeficiente de (a) La pared caliente irradia una cantidad de energa dada por Ie = a ATc absorcin del cuerpo que, al ser negro, es a = 1; es la constante de Stefan y A es el rea de la pared. Al mismo 4 . De modo que el flujo de tiempo, la pared caliente absorbe la radiacin que emite la pared fra, que es Ia = a ATf calor (energa por unidad de rea y tiempo) que irradia en total la pared caliente es Ie Ia 4 4 = (Tc Tf ). J0 = A (b) Al colocar una lmina parelela, al cabo de un cierto tiempo sta se pondr a una temperatura T1 intermedia entre Tc y Tf . En el estado estacionario, el flujo de calor J que sale de la pared caliente y va a la lmina, y el flujo que sale de la lmina y va a la pared fra sern iguales, de manera que tenemos 4 4 J0 T1 ) J = (Tc 4 4 2J = (Ta Tb ) = J0 J= 4 4 Tf ) J = (T1 2 Por lo tanto, al poner una lmina entre las paredes se reduce el flujo de calor a la mitad. (c) Si ahora son dos lminas las que estn entre las paredes, repitiendo al argumento, tenemos que una estar a temperatura T1 y la otra a temperatura T2 . El flujo de calor entre la pared caliente y la lmina 1, el flujo entre la lmina 1 y la lmina 2, y el flujo entre la lmina 2 y la pared fra sern iguales. Entonces 4 4 T1 ) J = (Tc J0 4 4 4 4 T2 ) J = (T1 3J = (Tc Ta ) = J0 J= 3 4 4 Tf ) J = (T2 y el flujo original se ha dividido por 3. En general, si tenemos n lminas intermedias, el flujo se reduce en un factor n + 1. Ejercicio 4.- El agua del mar absorbe la radiacin solar que penetra en su interior. La cantidad de radiacin por unidad de tiempo y rea (el flujo de energa) que llega a una profundidad h y que no ha sido absorbida todava est dada por I = I0 exp(h), donde I0 es el flujo incidente en la superficie del agua y es una constante que depende de la turbiedad del agua. Para aguas claras este cociente vale = 0, 05 m1 . Para que se realice la fotosntesis es necesario que el flujo de energa sea superior a 0,02 W/m2 . En un da normal, cuando la rediacin incidente es de 160 W/m2 ,a qu profundidad es de esperar que dejen de realizarse los procesos fotosintticos? (b) Si el flujo de radiacin solar incidente en cierta costa vara aproximadamente como I = 160 + 80 cos(2t/12) donde t es el nmero del mes e I est dada en W/m2 , estimar la profundidad hasta la que puede haber fotosntesis a lo largo del ao. Solucin: (a) La profundidad a la cual deja de producirse la fotosntesis es aquella para la que la intensidad vale I = 0, 02 W/m2 cuando I0 = 160 W/m2 , es decir, la que cumple 0, 02 = 160 exp(h) o bien, despejando h 1 0, 02 1 0, 02 = = 180 m ln h = ln 160 0, 05 m1 160 (b) Ahora tenemos una intensidad que depende del tiempo, pero la solucin es anloga a la anterior 0, 02 1 ln h(t) = 0, 05 m1 160 + 80 cos(2 t/12) V emos que esta profundidad lmite oscila a lo largo del ao. Ser mxima en el mes en que la radiacin solar es mxima, es decir, cos(2 t/12) = 1 y por lo tanto I = 160 + 80 = 1240 W/m2 , y ser mnima cuando cos(2 t/12) = 1 y por lo tanto I = 160 80 = 80 W/m2 . En el mes ms clido tenemos fotosntesis hasta la profundidad 0, 02 1 = 188 m ln h= 0, 05 m1 240 y en el mes ms fri la profundidad por debajo de la cual no hay fotosntesis es 0, 02 1 = 166 m ln h= 0, 05 m1 80

92

Mara Vir. Martnez Gonzlez

CAPTULO 14
Presin en gases y lquidos
Ejercicio 5.- Consideremos un recipiente cilndrico lleno de oxgeno a una temperatura uniforme de 300 K y sometido a la accin de la gravedad. A qu altura sobre la base del recipiente la presin se reduce a la mitad? Solucin: De acuerdo con la ecuacin (14.4) de la Unidad Didctica, la presin vara con la altura de la forma m0 g Mmol g h = P0 exp h P (h) = P0 exp kB T RT Por lo tanto, la altura h1/2 a la que la presin se reduce a la mitad satisface P (h1/2 ) 1 Mmol g h1/2 = = exp P0 RT 2 y as 8, 31 J K1 300 K RT h1/2 = ln 2 = ln 2 = 7950 ln 2 = 5510 m Mmol g 0, 032 kg 9, 8 m s2 Ejercicio 6.- Supongamos que en el ejercicio anterior la temperatura del gas no es uniforme sino que disminuye con la altura de forma T (h) = T0 h. a) Cmo vara entonces la presin con la altura?; b) A qu altura la presin se habr reducido a la mitad si T0 = 300 K y = 6 K km1 Solucin: a) La ecuacin (14.3) de la Unidad Didctica se transforma ahora en m0 g Mmol g dP = P = P dh kB (T0 h) R (T0 h) que es fcil de integrar Mmol g dh Mmol g dP (Mg/R) = ln P = ln (T0 ah) P (h) = (T0 ah) P R T0 h R b) Ahora ah1/2 (Mg/R) 1 P (h1/2 ) T0 ah1/2 (M g/R) = = 1 = P (0) T0 T0 2 de donde # " (R/Mg) T0 1 h1/2 = 1 2 Sustituyendo valores 8, 314 J K1 0, 006 K m1 R = = 0, 159 Mmol g 0, 032 kg 9, 8 m s2 y 300 300 K 1 0, 50.159 = (1 0, 9) = 5000 m h1/2 = 1 0, 006 K m 0, 006 Es importante comparar estos resultados con lo que sucede en un problema similar en lquidos, como el problema 2 del Captulo 14 de la Unidad Didctica.

Ejercicio 7.- Se llena de mercurio un largo tubo abierto por arriba. A continuacin se coloca el tubo invertido sobre un recipiente que tambin contiene mercurio. Qu altura h alcanzar sobre el nivel del mercurio en el recipiente alcanzar el mercurio en el tubo? (La densidad del mercurio es = 13, 6 103 kg m3 ) Solucin: Al volcar el tubo sobre el recipiente quedar un vaco en la parte superior del tubo, donde la presin es nula. Entonces, la presin que ejerce la columna de mercurio sobre la base debe ser igual a la presin atmosfrica que acta sobre la superficie del mercurio en el recipiente. Si la seccin del tubo es A, el volumen de la columna de mercurio de altura h es V ol = Ah. As pues, su peso es W = Ahg, y la presin sobre la base es P = gh. Entonces 101325 Pa = 0, 76 cm gh = 1atm = 101325 Pa h= 13, 6 103 kg m3 9, 8 m s2

93

Mara Vir. Martnez Gonzlez

Principio de Arqumedes
Ejercicio 8.- Un globo se llena de helio a presin atmosfrica. El volumen del globo es 16 m3 y la masa de la tela del globo es de 2.8 kg. Cul es el peso mximo que puede levantar este globo? (Densidad del He: 0,179 kg/m3 ; densidad del aire: 1,293 kg/m3 .) Solucin: El empuje que acta sobre el globo es E = aire gV El peso del helio dentro del globo es Phelio = helio V g y el peso total del globo es P = Phelio + mg de modo que la fuerza neta que acta sobre el globo cuando no lleva ningn peso aadido es E P = (aire aire ) gV mg = = 1, 114 kg m3 9, 8 m s2 16 m3 2, 8 kg 9, 8 m s2 = 147, 24 N Este ser, por lo tanto, el peso mximo que podr levatar el globo, que corresponde a una masa de 147,24/9,8 =15 kg Ejercicio 9.- Una esfera hueca de radio interior R y radio exterior 2R est formada de un material de densidad 0 y flota en la superficie de un lquido de densidad 20 . El interior se llena ahora de material de densidad de tal modo que la esfera glota justamente totalmente sumergida. Determinar . Solucin: Si la esfera est totalmente sumergida desplaza un volumen de fluido V = (4/3) (2R)3 y, por lo tanto, recibe un empuje 4 E = l V g = 20 (2R)3 g 3 Por otra parte, el peso total de la esfera sumergida es el peso de la corteza esfrica de densidad 0 ms el peso del espacio interior relleno de fluido de densidad 4 4 .P = 0 (2R)3 R3 g + R3 g 3 3 Puesto que la esfera no flota ni se hunde, el empuje es igual al peso: entonces = 90 E=P 20 23 = 0 23 1 +

Ejercicio 10.- Un globo de aire caliente tiene un volumen de 15 m3 y est abierto por la parte inferior. Si el aire del interior del globo est a una temperatura de 75o C mientras que la temperatura del aire exterior es de 25o C y a una presin de 1 atm. Cul es la fuerza neta sobre el globo y el contenido? (Despreciar el peso de la tela del globo y la barquilla) Solucin: Este problema plantea una sencilla explicacin del funcionamiento de un globo aerosttico. El aire dentro del globo est mantenido a una temperatura ms alta que la del aire exterior gracias a un hornillo de gas que lo calienta continuamente, compensando las prdidas por radiacin.. Por otra parte, el globo est abierto por abajo para que la presin interior sea la misma que la presin exterior. Si hubiera la ms mnima diferencia aparecera una corriente de aire, hacia dentro o hacia fuera, que las igualara. El peso del globo (despreciendo la tela y la barquilla) es W = nc Mmol g siendo nc el nmero de moles de aire caliente dentro del globo (es decir, en un volumen de 1,5 m3 ) y Mmol la masa molecular del aire. Por otra parte, de acuerdo con el principio de Arqumedes, sobre el globo se ejerce un empuje vertical hacia arriba igual al peso de un volumen equivalente de aire fro exterior al globo E = nf Mmol g y la fuerza neta vertical hacia arriba ser F = E W = (nf nc )Mmol g

94

Mara Vir. Martnez Gonzlez

Puesto que la presin es la misma dentro que fuera, tenemos (aplicando la ecuacin de los gases perfectos y llamando Tc y Tf a las temperaturas del aire caliente y el aire fro, respectivamente) Tf P V = nc RTc = nf RTf nc = nf Tc y as Tf Mmol g F = (nf nc )Mmol g = nf 1 Tc o tambin Tf P V Tc Tf PV 1 Mmol g = Mmol g F = RTf Tc R Tf Tc Sustituyendo valores 50 K 1 atm 15000 litros 0, 032 kg 9, 8 m s2 ' 28 N F = 0, 082 atm l/(mol K) 298 K 348 K Ejercicio 11.- Se puede pesar a una persona utilizando el principio de Arqumedes por medio de una plataforma flotante de rea A, espesor h = 20 cm y masa M = 600 kg sobre un estanque tranquilo. Cuando no hay nadie sobre la plataforma, sta se hunde d1 = 7 cm en el agua. Cuando la persona se coloca sobre la plataforma, sta se hunde d2 = 8, 4 cm. (a) Cul es la masa de la persona? (b) Cul es la masa mxima que puede pesar esta balanza. Solucin: (a) El volumen sumergido de la plataforma cuando est vaca y sumergida d1 es Ad1 . En este caso experimenta un empuje E = a Ad1 g . Este empuje compensa el peso M g de la plataform, es decir M g = a Ad1 g Si ahora se coloca una persona de masa m sobre la plataforma, sta se hundir una profundidad d2 tal que (M + m) g = a Ad2 g Dividiendo la segunda ecuacin por la primera obtenemos m M +m d2 =1+ = d1 M M y as 8, 4 d2 1 600 kg = 120 kg 1 M = m= d1 7 (b) La mxima masa que puede pesarse es la que hace que la plataforma se hunda justo toda su altura. Una masa mayor hara que la plataforma se hundiera hasta el fondo del estanque. Por tanto dicha masa es h 20 1 600 kg = 1 114, 3 kg 1 M = mmax = d1 7 Ejercicio 12.- Hemos comprado un anillo muy caro y nos encontramos con el mismo problema que tuvo Arqumedes: queremos saber si es de oro puro. Para ello pesamos el anillo y observamos que pesa 0,158 N. Lo sumergimos en el agua y lo volvemos a pesar, y ahora su peso es 0,145 N. Es de oro el anillo? (La densidad del oro es oro = 19, 3 103 kg/m3 ) Solucin: Si el anillo fuera realmente de oro, su peso (en aire) sera Paire = oro V g siendo V el volumen del anillo. Al introducirlo en el agua experimentara un empuje E = agua V g, de modo que su peso en agua sera Pagua = oro agua V g de modo que oro agua agua Pagua 1 = 0, 948 = = 1 =1 Paire oro oro 19, 3 Sin embargo, segn los datos que nos dan este cociente es 145/158 = 0,918 , de modo que el anillo no es de

95

Mara Vir. Martnez Gonzlez

oro. De hecho, es de un material de densidad agua = 0, 918 1

agua = 12, 20 103 kg m3 0, 082

96

Mara Vir. Martnez Gonzlez

CAPTULO 15
Primer principio de la termodinmica
La energa interna de un sistema vara o bien por aporte de calor Q o bien por trabajo W realizado por o sobre el sistema U = Q W (1) El calor es la energa transferida que no depende de la variacin de volumen. Si adems hay una variacin de volumen, hay una energa transferida en forma de trabajo termodinmico dado por Z W = P dV (2)

Con este convenio de signos el trabajo es positivo si el volumen aumenta. Entonces el trabajo positivo es trabajo realizado por el sistema sobre el medio externo, y esto hace disminuir la energa interna del sistema. R (Alternativamente se puede definir el trabajo como W = P dV , lo que equivale a decir que el trabajo postivo es el realizado por el medio externo sobre el sistema, y entonces el primer principio queda U = Q + W ).

Procesos cuasiestticos en un gas ideal


Un proceso termodinmico cuasiesttico es una sucesin de estados de equilibrio termodinmico que lleva un sistema termodinmico de un estado de energa interna Ui a un estado final de energa Uf . La relacin entre la energa interna U , la presin P, el volumen V y la temperatura T dependen del sistema. En el captulo 12 se ha visto que para un gas ideal monoatmico esta relacin es 3 3 (3) U = P V = nRT 2 2 A partir de esta relacin es fcil calcular W y Q para algunos procesos particularmente simples: i) Proceso a volumen constante (isocoro) V = cte. W =0 y Q = U = nCV (Tf Ti ) (4) siendo CV la capacidad calorfica molar a volumen constante. Comparando est expresin con (3) se tiene que para un gas perfecto monoatmico CV = (3/2)R. ii) Proceso a presin constante (isobaro) P = cte. Z f dV = Pi (Vf Vi ) W = Pi
i

Q = nCP (Tf Ti )

(5)

siendo CP la capacidad calorfica molar a presin constante. Para un gas ideal a presin constante Pi Pi (Vf Vi ) = nR(Tf Ti ) de modo que tambin podemos escribir Pi (Vf Vi ) W = nR(Tf Ti ) y Q = CP R Entonces CP 1 (Vf Vi ) = n (CP R) (Tf Ti ) U = Q W = Pi R iii) Proceso a temperatura constante (isotermo) T = cte. En este caso U = 0. Adems, teniendo en cuenta que Pi Vi = P V = cte Z f Z f Vf Vf dV = Pi Vi ln P dV = Pi Vi = nRTi ln y W = V V Vi i i i

(6)

Q=W

(7)

iv) Proceso adiabtico Q = 0. Puede demostrarse que en un proceso adiabtico P V = cte. donde = CP /CV es el llamado coeficiente adiabtico del gas. Entonces ! Z f Z f 1 dV 1 1 P dV = Pi Vi = Pi Vi = W = 1 Vf 1 Vi 1 i i V

97

Mara Vir. Martnez Gonzlez

1 1

Pf Vf Vf 1

Pi Vi Vi 1

nR 1 (Pf Vf Pi Vi ) = (Tf Ti ) 1 1

(8)

Ejercicio 1.- Consideremos un gas ideal que ocupa inicialmente un volumen V y su temperatura es T. El gas se somete entonces a dos procesos consecutivos: primero se deja enfriar a volumen constante hasta que su temperatura se reduce a T /2, y luego se calienta a presin constante hasta que vuelve de nuevo a la temperatura T. A partir de esto, demostrar que la diferencia entre las capacidades calorficas del gas a presin constante y a volumen constante es CP CV = R, siendo R la constante de los gases ideales. Solucin: Inicialmente el gas est a una temperatura T tal que P V = nRT. El primer proceso se realiza a volumen constante, por lo que no se realiza trabajo. Puesto que la temperatura se ha reducido a la mitad, la presin al final de este proceso tambin se habr reducido a la mitad, es decir, a P/2. Y el calor cedido por el gas es Qced = nCV (T /2 T ) = nCV T /2. El segundo proceso se realiza a una presin constante P/2. Para alcanzar la misma temperatura de antes el volumen tendr que duplicarse. Por lo tanto, el gas realizar un trabajo W = (P/2)(2V V ) = P V /2 = nRT /2. Adems absorber un calor Qabs = nCP (T T /2) = nCP T /2. Puesto que la temperatura final es la misma que la inicial, tambin las energas internas inicial y final son iguales. Es decir CP CV = R U = Qabs Qced W = 0 De hecho, podamos haber obtenido este resultado directamente de (6) As pues, para un gas ideal monoatmico CV = (3/2) R y CP = CV + R = (5/2)R, con lo que su coeficiente adiabtico es = 5/3. Para gases ideales no monoatmicos las capacidades calorficas son diferentes, pero la relacin CP CV = R sigue siendo vlida. Ejercicio 2.- El coeficiente adiabtico de un gas diatmico es = 7/5 = 1, 4. Cunto valen sus calores especficos a volumen y a presin constantes? Solucin: CP 7 y CP CV = R = CV 5 de donde 7 5 7 y CP = R CV = R CP = CV + R = CV 5 2 2 Ejercicio 3.- Un mol de gas monoatmico ideal se calienta desde 300 K hasta 600 K. (a) Hallar el calor suministrado al gas, el trabajo realizado por el gas y la variacin de la energa interna si el calentamiento se produce a volumen constante. (b) Hallar estas mismas magnitudes si el gas se calienta a presin constante. Solucin: Antes de hacer ningn clculo, sabemos que la variacin de energa interna ser la misma en ambos casos, ya que la energa imterna de un gas ideal est completamente determinada por su temperatura. Lo que vara en cada caso es cmo se reparte esta variacin de energa entre trabajo realizado y calor transferido. a) Si el volumen se mantiene constante, el trabajo realizado por el gas es nulo: W = 0. Entonces la variacin de la energa interna es igual al calor suministrado al gas 3 U = Q = nCV T = RT = 2 3 1 = 8, 31 J K 300 K = 3741 J 2 b) Si la presin se mantiene constante, Pf = Pi , el trabajo realizado por el gas es W = Pi (Vf Vi ) = nR(Tf Ti ) = nRT = 8, 31 J K1 300 K = 2494 J El calor suministrado al gas es 5 5 Q = nCP T = RT = 8, 31 J K1 300 K = 6235 J 2 2

98

Mara Vir. Martnez Gonzlez

y la variacin de energa interna es U = Q W = n(CP R)T = 3741 J igual que en a), como ya habamos anticipado.

Ejercicio 4.- Dentro de un cilindro provisto de un pistn mvil hay 2 moles de un gas ideal monoatmico a una temperatura inicial de 20o C. Se suministra calor por un valor de 500 J. Hallar: a) el incremento de temperatura si se mantiene la presin constante; b) el trabajo realizado por el gas; c) el cociente entre los volmenes inicial y final ocupados por el gas. Solucin: a) Si se suministra calor a presin constante, el incremento de temperatura es 2Q 1000 J 5 T = = = 12 K Q = nCP T = n RT 2 n5R 10 8, 31 J K1 b) El trabajo realizado es W = P (Vf Vi ) = nR(Tf Ti ) = nRT = 2 8, 31 J K1 12 K = 200 J c) El cociente entre los volmenes es 293 K nRTf Vi Ti Ti nRTi = = 0, 93 = = = Vi Vf Vf Tf Ti + T 305 K Ejercicio 5.- Un mol de gas ocupa inicialmente un volumen de 1 litro y su presin es de 3 atm. El gas se expansiona isotrmicamente hasta que su volumen es de 3 litros y su presin de 1 atm. Luego se calienta a volumen constante hasta que alcanza una presin de 2 atm. (a) Calcular la variacin de la energa interna del gas. (b) Calcular el trabajo realizado por el gas. (c) Calcular el calor absorbido por el gas durante este proceso. (Datos: constante de los gases R = 0, 082 atm.l/(mol.K) = 8,31 J/mol.K); coeficiente adiabtico del gas = CP /CV = 5/3) Solucin: Dado que = 5/3 sabemos que el gas es monoatmico. El proceso total se compone de un proceso isotermo desde el punto (P1 , V1 ) al punto (P2 , V2 ) = (P1 /3, 3V1 ), seguido de un calentamiento a volumen constante desde (P2 , V2 ) = (P1 /3, 3V1 ) hasta (P3 , V3 ) = (2P2 , V2 ). Entonces la variacin total de la energa interna es 9 8, 31 3 3 J = 456 J U = (P3 V3 P1 V1 ) = (2 3 3 1) atm l = 2 2 2 0, 082 Durante la expansin isoterma no vara la temperatura y, por lo tanto, tampoco lo hace la energa interna del gas. Teniendo en cuenta que P V = P1 V1 , el trabajo realizado por el gas durante la expansin isoterma es Z V2 Z V2 V2 dV = P1 V1 ln P dV = P1 V1 = P1 V1 ln 3 = 3 ln 3 atm l = 304 ln 3 J = 334 J W = V V1 V1 V1 Esta misma cantidad ha tenido que suministrarse al gas en forma de calor para mantener la temperatura (es decir, la energa interna) constante; es decir Q1 = W. A continuacin el gas se calienta a volumen constante, de modo que no se realiza trabajo. Adems, cuando la presin se haya duplicado tambin lo habr hecho la temperatura. El calor suministrado al gas es entonces 3 3 3 Q2 = CV T = R(2T1 T1 ) = RT1 = P1 V1 = 456 J 2 2 2 En definitiva, el calor total suministrado al gas durante el proceso completo es entonces QT = Q1 + Q2 = 790 J. El trabajo total realizado es W = 334 J. Ejercicio 6.- Dos moles de helio se encuentran inicialmente a la temperatura de 27o C y ocupan un volumen de 20 litros. El gas se expande primero a presin constante hasta duplicar su volumen, y luego se expande adiabticamente hasta que la temperatura vuelve a su valor inicial. (a) Calcular la variacin total de la energa interna del helio; (b) calcular el calor total suministrado al gas; (c) calcular el trabajo total realizado por el gas; (d) calcular el volumen final; (e) calcular los trabajos realizados en la expansin isobara y en la expansin adiabtica por separado (Datos: constante de los gases R = 0,082 atm.l/(mol.K) = 8,31 J/(mol.K); coeficiente adiabtico del helio = CP /CV = 5/3) Solucin:

99

Mara Vir. Martnez Gonzlez

Nuevamente se trata de un gas monoatmico. a) Puesto que la temperatura final es igual a la inicial, la energa interna final tambin ser igual a la inicial, de modo que la variacin de energa interna en el proceso total es nula: UT = 0. b) Puesto que en la expansin adiabtica no hay intercambio de calor, el nico calor aportado al sistema es en la expansin a presin constante. Como en esta expansin el volumen se ha duplicado, la temperatura tambin lo habr hecho. Entonces 5 Q = nCP (T2 T1 ) = 2 R (2T1 T1 ) = 5RT1 = 2 = 5 8, 31 J K1 300 K = 12465 J c) Para el proceso total se debe satisfacer UT = QT WT = 0 WT = QT = 12465 J d) Puesto que el paso de (P2 , V2 ) a (P3 , V3 ) es adiabtico debe cumplirse P2 V2 = P3 V3 . Como adems P2 V2 = nRT2 y P3 V3 = nRT3 , tenemos 1 3/2 1 T2 2T1 1 1 nRT2 V2 = nRT3 V3 V3 = V2 = 2V1 = 4 2V1 = 80 2 litros T3 T1 e) El trabajo realizado en la expansin isobara es Z V2 P dV = P1 (V2 V1 ) = P1 V1 = 2RT1 = 4986 J W12 =
V1

El trabajo realizado en la expansin adiabtica es nR 2R (T3 T2 ) = (T1 2T1 ) = 3RT1 = 7479 W23 = 1 2/3 y asi WT = W12 + W13 = 12465 J, como ya se ha visto en c). Ejercicio 7.- Una cierta cantidad de aire, cuyo coeficiente adiabtico es = 7/5, se expansiona adiabtica y cuasiestticamente desde una presin inicial de 2 atm y volumen 2 litros a temperatura ambiente de 20o C hasta doblar su volumen original. Calcular: a) la presin final; b) la temperatura final; c) el trabajo realizado por el gas en la expansin. Solucin: Puesto que = 7/5 es fcil deducir que CV = (5/2) R y CP = (7/2) R. En un proceso adiabtico cuasiesttico se cumple que P V se mantiene constante. Sean Pi , Vi la presin y el volumen en el estado inicial y Pf , Vf la presin y el volumen en el estado final. Se cumplir Pi Vi = Pf Vf es decir Pf = Pi Vi Vf

Para los valores dados en el enunciado tenemos 7/5 2 = 0, 758 atm Pf = 2 atm 4 Para calcular la temperatura final podemos utilizar la relacin Ti Vi 1 = Tf Vf 1 . Entonces 1 2/5 Vi 2 Tf = Ti = 293 K = 222 K Vf 4 El trabajo realizado en un proceso adiabtico est dado por nR Pi Vi Pi Vi Tf (Tf Ti ) = W = (Tf Ti ) = 1 = 1 ( 1) Ti ( 1) Ti 2 atm 2 litros 222 K = 1 = 2, 41 atm litro ' 244 J 2/5 293 K Alternativamente, podramos haber calculado el trabajo a partir de la variacin de la energa interna, que en

100

Mara Vir. Martnez Gonzlez

este caso es U = nCV (Tf Ti ) = = 5 5 nR(Tf Ti ) = (Pf Vf Pi Vi ) = 2 2 5 (0, 758 4 2 2) atm litro = 2, 41 atm litro 2

Ejercicio 8.- Un gas diatomico ideal tiene un coeficiente adiabtico = 1, 4. Supongamos que inicialmente tenemos en un recipiente 4 litros de este gas a una presin de1 atm y una temperatura de 300 K. Entonces se realiza un ciclo cerrado compuesto de los siguientes procesos: i) se triplica su presin a volumen constante; ii) a continuacin se expande adiabticamente hasta volver a su presin inicial; iii) finalmente se comprime a presin constante hasta llegar a su volumen original, con lo que se cierra el ciclo. a) Calcular la temperatura final del ciclo; b) calcular la temperatura del gas al principio de la expansin adiabtica; c) determinar el volumen del gas al final de la expansin adiabtica; d) calcular el trabajo neto realizado por el gas en el ciclo. Solucin: a) Puesto que al finalizar el ciclo el sistema ha vuelto a su estado inicial, la temperatura fnal es la temperatura de partida, 300 K, y la variacin total de energa interna es cero UT = 0. b) La primera fase del ciclo es un proceso a volumen constante que lleva al sistema desde (P1 , V1 ) a (P2 , V2 ) = (3P1 , V1 ). Por lo tanto, 3P1 V1 P2 V2 = = 3T1 = 900 K T2 = nR nR que es la temperatura cuando empieza la expansin adiabtica. c) La expansin adiabtica lleva al sistema desde (P2 , V2 ) = (3P1 , V1 ) hasta (P3 , V3 ) = (P1 , V3 ). Entonces P2 V2 = P3 V3 3P1 V1 = P1 V3 V3 = 31/ V1 = 35/7 V1 ' 8, 76 litros d) El trabajo en el proceso a volumen constante es W12 = 0. El trabajo en la expansin adiabtica es 1 1 1/ (P3 V3 P2 V2 ) = 3 P1 V1 3P1 V1 = 1 1 5 P1 V1 35/7 3 = 8, 1 atm litro ' 820, 9 J = 2 Finalmente, el trabajo en la compresin que lleva de (P3 , V3 ) = (P1 , V3 ) hasta (P1 , V1 ) es W31 = P1 (V1 V3 ) = P1 V1 1 35/7 ' 4, 76 atm litro ' 482, 4 J W23 =

Por lo tanto, el trabajo neto realizado en el ciclo es 5 P1 V1 35/7 3 + P1 V1 1 35/7 = WT = W12 + W23 + W31 = 2 17 7 5/7 3 ' 338, 5 J = P1 V1 2 2

El calor absorbido en el proceso a volumen constante es 5R 5 (T2 T1 ) = (P2 V2 P1 V1 ) = 5P1 V1 = 2 2 = 20 atm litro = 2026, 8 J y el calor absorbido en el proceso a presin constante 7R 7 7 Q31 = nCP (T1 T3 ) = n (T1 T3 ) = (P1 V1 P3 V3 ) = P1 V1 1 35/7 2 2 2 = 16, 7 atm litro ' 1688, 3 J y as 7 7 QT = Q12 + Q31 = P1 V1 5 + 35/7 = 2 2 Q12 = nCV (T2 T1 ) = n

101

Mara Vir. Martnez Gonzlez

= P1 V1

17 7 5/7 3 2 2

' 338, 5 J

Ejercicio 9.- Un mol de gas ideal monoatmico ocupa inicialmente un volumen Vi = 25 litros y est a una presin Pi = 1 atm. El gas se calienta lentamente, de modo que su estado sigue una lnea recta en un diagrama PV hasta que su volumen final es Vf = 75 litros y su presin Pf = 3 atm. Calcular: (a) el trabajo realizado por el gas en el proceso; (b) la variacin de la energa interna del gas; (c) el calor suministrado al gas. (Datos: constante de los gases R = 0,082 atm l/(mol K) = 8,31 J/(mol K ); coeficiente adiabtico de un gas monoatmico = CP /CV = 5/3.) Solucin: a) El proceso de calentamiento no es en este caso ninguno de los procesos simples ya estudiados. Ahora el estado termodinmico pasa de (Pi , Vi ) a (Pf , Vf ) = (3Pi , 3Vi ) siguiendo una lnea recta. La pendiente de la recta es (Pf Pi )/(Vf Vi ) = 2Pi /2Vi , de modo que la ecuacin de la recta en el diagrama PV es Pf Pi Pi Pi P = Pi + (V Vi ) = Pi + (V V1 ) = V Vf Vi Vi Vi de modo que el trabajo realizado por el gas al expandirse es Z Z Vf Pi 3Vi Pi 8V12 = 4Pi Vi = P dV = V dV = W = Vi Vi Vi 2 Vi 100 8, 31 J ' 10134 J = 100 atm litro = 0, 082 b) 3 3 U = (Pf Vf Pi Vi ) = 8Pi Vi = 300 atm litro = 30402 J 2 2 c) El calor suministrado al gas es entonces Q = U + W = 12Pi Vi + 4Pi Vi = 16Pi Vi ' 40536 J

Mquinas trmicas
Ejercicio 10.- Una amiga que pesa 60 kg nos pide que le digamos hasta qu altura debe subir por una escalera para quemar las 500 caloras ingeridas al comerse un helado, y de esta forma no acumular peso. (Una calora de valor alimentario corresponde a 1 kilocalora = 4.180 J.) Qu le contestamos? Solucin: La energa requerida para subir una altura h esta dada por Ep = mgh Esta energa debe coincidir con la que corresponde al helado Q = 500 4.180 J = 2, 09 106 J. Entonces 2, 09 106 J Q = mgh = Q h= = 3554 m mg 60 kg 9, 8 m/s2 Es decir, que para quemar el contenido energtico de este helado tendramos que escalar ms de tres kilmetros. Sin embargo, tngase en cuenta que en el propio proceso de metabolizacin de la comida se requiere hasta un 40% del aporte energtico ya que digerir alimentos consume energa. Ejercicio 11.- Un camin funciona con un motor de gasolina que en cada ciclo usa 10 kJ del calor resultante de quemar gasolina y produce 2 kJ de trabajo mecnico. (a) Calcular el rendimiento del motor. (b) Cunto calor se vierte a la atmsfera por ciclo? (c) Cunta gasolina se quema por ciclo? (El calor de combustin de la gasolina es Lc = 50 kJ/g, es decir, quemando un gramo de gasolina se obtienen 50 kJ de eerga.(d) Si el motor ejecuta 25 ciclos por segundo, calcular la potencia del motor. (e) Qu volumen de gasolina se quema en una hora? (Densidad de la gasolina 0,7 g/cm3 ) Solucin:

102

Mara Vir. Martnez Gonzlez

(a) El rendimiento es = W/Qa , donde en este caso W = 2 kJ y Qa = 10 kJ, de manera que = 0, 2 , es decir, un rendimiento del 20% que es tpico para coches y camiones. (b) El calor Qa que no se invierte en trabajo W se vierte directamente a la atmsfera. Por tanto, Qb = Qa W = 8 kJ en cada ciclo. (c) Tenemos que Qa = mLc , donde m es la masa de gasolina que se quema por ciclo. Es decir m = Qa /Lc = 0, 2 g. (d) En cada ciclo obtenemos 2 kJ y en cada segundo hay 25 ciclos. Por lo tanto, el trabajo que se hace cada segundo es W = 25 s1 2 kJ = 50 kJ/s = 50 kW. (e) Sabemos, por el apartado (c), que en cada ciclo se queman 0,2 g de gasolina. Por lo tanto, en los 25 ciclos que se producen cada segundo se habrn quemado 5 g de gasolina; es decir, en una hora se queman 18 kg de gasolina o, lo que es lo mismo, 25,7 litros. Ejercicio 12.- Una planta de electricidad tiene una capacidad generadora de 0,4 gigavatios (0, 4 109 W) de potencia elctrica. La planta quema 10.500 toneladas de carbn al da. Sabiendo que el calor de combustin del carbn es 6, 2 106 J/kg, calcule el rendimiento trmico global de la planta. Solucin: En un da la planta consume 10, 5 106 kg de carbn, lo que da lugar a 10, 5 106 kg 6, 2 106 J kg1 = 65, 1 1012 J de energa calorfica. Por otra parte, puesto que cada da tiene 86.400 segundos, la energa elctrica que produce la central en un da es 0, 4 109 W 86.400 s = 34, 56 1012 J . Por lo tanto, el rendimiento ser 34, 56 1012 J = 0, 53 = 65, 1 1012 J

Mquinas reversibles y rendimiento de Carnot


Como se describe en la seccin 15.3 de la Unidad Didctica una mquina reversible realiza un proceso cclico que es una sucesin de estados de equilibrio. En particular, las transferencias de calor entre la mquina y los focos trmicos son procesos isotermos. La sustancia de la mquina est a la misma temperatura que el foco trmico del que absorbe o al que cede calor. Por ello se puede hacer funcionar al revs, describiendo el ciclo en sentido contrario. Si en un sentido la mquina absorbe un calor Qa de un foco caliente, realiza un trabajo W = Qa y cede un calor Qb = Qa W = Qa (1 ) a un foco fro, trabajando en sentido contrario la mquina absorbera un calor Qb del foco fro, absorbera tambin un trabajo W (que se realizara sobre la mquina), y finalmente cedera un calor Qa = Qb + W al foco caliente. Por consiguiente, todas las mquinas reversibles deben tener el mismo rendimiento. Supongamos que una segunda mquina tuviese un rendimiento mayor que la anterior, es decir, que para realizar el mismo trabajo W tomara un calor Qa < Qa del foco caliente y cedera un calor Qb < Qb al foco fro, con Qa Qa = Qb Qb . Y supongamos ahora que acoplamos la primera mquina trabajando en sentido inverso con la segunda trabajando en sentido inverso. El resultado de esta mquina compuesta sera absorber un calor Qb Qb del foco fro y ceder la misma cantidad de calor al foco caliente sin realizar trabajo neto. Esto es contrario al segundo principio de la termodinmica. Ejercicio 13.- Una mquina de Carnot cuyo foco caliente se encuentra a 400 K toma 500 J a esta temperatura y cede 350 J al foco fro. (a) Cul es la temperatura de este ltimo?; (b) cul es el rendimiento de la mquina? Solucin: Para una mquina de Carnot T Q = Q+ T+ siendo Q+ al calor tomado del foco caliente y Q el calor cedido al foco fro. En nuestro caso Q+ = 500 J y Q = 350 J. Entonces 350 Q K = 280 K T = T + + = 400 Q 500 y el rendimiento es T T+ T 120 K r =1 + = = 0, 3 = T T+ 400 K

103

Mara Vir. Martnez Gonzlez

Ejercicio 14.- Una mquina trmica absorbe en cada ciclo 200 J de calor de una fuente trmica a 100o C y cede 146,4 J a una fuente trmica de menor temperatura a 0o C. Calcular el rendimiento suponiendo que la mquina es reversible. Calcular el trabajo neto realizado por la mquina en cada ciclo. Solucin: Una mquina reversible tiene el rendimiento de Carnot 273 Tf = 0, 27 =1 r =1 Tc 373 El trabajo realizado en un ciclo es W = rQabs = 0, 27 200 J = 54 J

104

Mara Vir. Martnez Gonzlez

CAPTULO 16
Cambios de fase
Una sustancia simple es un agregado de molculas de un mismo tipo. Sin embargo, dependiendo de las condiciones de presin y temperatura, estas molculas pueden estar en distintos estados de agregacin. Estos diferentes estados de agregacin se denominan fases. La Figura 16.10 de la Unidad Didctica muestra en forma cualitativa el diagrama de fases del agua. Para presiones y temperaturas por debajo de la lnea abc, el agua se presenta en forma de vapor. Para presiones y temperaturas a la izquierda de la lnea abd, el agua se presenta en forma slida (hielo). Y para presiones y temperaturas entre las lneas bd y bc, el agua se presenta en forma lquida. Las curvas ab, bc y bd tienen en comn el punto b. Este punto se llama punto triple, y para estos valores de presin y temperatura pueden coexistir las fases slida, lquida y vapor. A lo largo de la curva bd coexisten la fase slida y la fase lquida. Si partimos de un punto a la izquierda de la curva bd, donde tenemos agua slida (hielo), y suministramos calor al sistema, manteniendo constante la presin, la temperatura aumenta hasta que, al llegar a la curva bd, el hielo empieza a fundirse y transformarse en agua lquida. Mientras ocurre este cambio o transicin de fase, la temperatura permanece constante: el calor suministrado al sistema no se traduce en un aumento de temperatura sino que se utiliza en romper los enlaces moleculares. Este calor se denomina calor latente de fusin. Una vez que todo el hielo se ha transformado en agua lquida, la temperatura vuelve a aumentar hasta que, al llegar a la curva bc, el agua empieza a hervir y transformarse en vapor. Mientras esto ocurre la temperatura permanece constante: ahora existe un calor latente de vaporizacin. Una vez convertida toda el agua en vapor, la temperatura vuelve a aumentar. Lo contrario sucede si partimos de un punto a la derecha de la curva bc y, manteniendo constante la presin, robamos calor al sistema. La temperatura desciende hasta que, al llegar a la curva bc, el vapor empieza a condensarse. En este proceso el vapor cede calor, por lo que la temperatura se mantiene constante: este calor latente de condensacin tiene el mismo valor que el calor latente de vaporizacin. Cuando todo el vapor se ha condensado la temperatura vuelve a descender hasta que, al llegar a la curva bd, el agua lquida empieza a congelarse. La temperatura se mantiene entonces constante hasta que toda el agua se ha congelado, y luego vuelve a descender. Si partimos de un punto a la izquierda de la curva ab, donde tenemos agua slida, y suministramos calor, manteniendo la presin constante, la temperatura del hielo aumentar hasta que, al llegar a la curva ab, el hielo emperar a evaporarse. Mientras se evapora la temperatura permanece constante: ahora hay un calor latente de sublimacin. Procesos similares ocurren si cambiamos la presin manteniendo la temperatura constante. Por ejemplo, si partiendo de agua lquida por encima de la curba bc, disminuimos la presin manteniendo constante la temperatura, el agua empezar a hervir al llegar a la curva bc. La curva bc tiene siempre pendiente positiva: esto significa que si aumenta la presin tambin aumenta la temperatura de ebullicin. As, a una presin de 1 atm el agua hierve a 100 o C, pero a una presin mayor el agua se mantiene lquida a esta temparatura y es necesaria una mayor temperatura para que hierva: esto es lo que ocurre en una olla a presin. Recprocamente, a presiones menores que 1 atm el agua hierve a una temperatura inferior a los 100o C. Esto es lo que sucede, por ejemplo, a gran altura, donde es ms difcil cocinar los alimentos. Sin embargo, la curva bc termina en el punto c, llamado punto crtico. A una presin por encima de la del punto crtico no hay transicin de fase lquido-vapor aunque aumentemos la temperatura. Esto quiere decir que podemos pasar de agua a vapor sin que en ningn momento haya una discontinuidad en la temperatura ni, por consiguiente, calor latente: simplemente se produce una disminucin continua de la densidad. Y lo mismo sucede si disminuimos la presin a una temperatura por encima de la temperatura del punto crtico. Muy diferente de la curva bc de coexistencia lquido-vapor es la curva bd de coexistencia slido-lquido. En primer lugar, no parece que esta curva termine en ningun punto. Adems, en el caso del agua, esta curva tiene pendiente negativa: si aumenta la presin, disminuye la temperatura. Por ejemplo, a una presin de 1 atm el hielo funde a 0o C, pero a una presin mayor el hielo funde a una temperatura menor. No obstante, la pendiente de la curva bd es muy grande (en valor absoluto) por lo que se necesita un aumento de presin muy grande para tener un descenso apreciable de la temperatura de fusin. Otra consecuencia del hecho de que la pendiente de la curva bd sea negativa es que a una temperatura constante ligeramente por debajo de la del punto triple es posible pasar de hielo a agua lquida aumentando la presin.

105

Mara Vir. Martnez Gonzlez

No obstante, esta pendiente negativa de la curva de coexistencia slido-lquido solo se da para el agua y alguna otra sustancia muy especial. Para todas las dems sustancias la pendiente es positiva, aunque siempre muy grande.

Ejercicio 1.- Una nube tpica de tormenta tiene un volumen de 2 2 10 kilmetros cbicos. Suponiendo que un 10% de la masa del aire hmedo que forma la nube es vapor de agua, estimar cunto calor se libera si todo este vapor se condensa en pequeas gotas de agua. Solucin: La nube tiene un volumen de 40 km3 . Un volumen semejante de aire seco tiene una masa aproximada de m = 40 103 m3 1, 3 kg = 52000 kg Si suponemos que en aire hmedo un 10% de la masa es vapor de agua, esto da una masa de vapor de 5200 kg. Si todo este vapor se condensa, libera un calor Q = 5200 kg 2257 103 J/(kg K) = 11, 7 109 J Ejercicio 2.- Un gramo de agua se convierte en 1.671 cm3 de vapor de agua cuando hierve a presin constante de 1 atm (es decir, 1, 013 105 Pa). Sabiendo que el calor de vaporizacin a esta presin es Lv = 2, 256 106 J/kg, calcular (a) el trabajo realizado por el agua al vaporizarse, y (b) el aumento de energa interna del agua. Solucin: (a) Como el proceso tiene lugar a presin constante, el trabajo realizado por el agua ser W = P (V2 V1 ), del gramo de agua donde P es la presin atmosfrica, V1 es el volumen y V2 es el volumen final del vapor. El inicial volumen de 1 gramo de agua es V1 = m/agua = 103 kg / 103 kg/m3 = 106 m3 . Por lo tanto (b) El calor que se suministra al agua es el necesario para evaporar 1 g de agua, es decir, Q = mLv = 103 kg 2, 256 106 J/kg = 2256 J. Por la primera ley de la termodinmica, la variacin de la energa interna es U = Q W = 2087 J. V emos que de todo el calor suministrado al agua (Q = 2256 J) casi todo se invierte en aumentar la energa interna mientras que una parte muy pequea (W = 169 J) se destina a hacer trabajo en forma de expansin del vapor de agua. Ejercicio 3.- Algunos animales de sangre caliente, como los humanos, pueden sudar para mantener su temperatura corporal cuando la del entorno sube demasiado. (a) Qu masa de agua debe evaporarse de la piel de una mujer de 50 kg para enfriar su cuerpo en un grado centgrado? (b) Qu volumen de agua debe beber la mujer para reponer la que evapor? (El calor latente de vaporizacin del agua a 37o C es Lv = 2, 42 106 J/kg y la capacidad calorfica del cuerpo humano es C = 3.480 J/kg.K.) Solucin: Para bajar 1 grado la temperatura corporal de la mujer hay que extraerle una cantidad de calor Q = mC T = 50 kg 3.480 J/(kg K) 1 K = 174 103 J Esta energa se extrae del cuerpo de la mujer y se invierte en vaporizar el sudor (que es prcticamente agua). Con ello se puede llegar a vaporizar una cantidad Q 174 103 J = 0, 072 kg = ma = Lv 2, 42 106 J/kg que equivale a un volumen de 0,072 litros, es decir, casi un dcimo de litro. Ejercicio 4. - En un sistema de calefaccin por agua el agua llega a 60o C y sale a 38o C. Se sustituye por otro de vapor en el que el vapor a presin atmosfrica se condensa y sale a 82o C. Cuntos kilogramos de vapor suministan el mismo calor que suministraba un kilo de agua? Solucin: El calor que cede una masa ma = 1 kilogramo de agua al pasar de 60o C a 38o C es Qagua = ma ca T = 1 kg 4180 J/(kg K) 22 K = 91960 J W = P (V2 V1 ) = 1, 013 105 Pa (1.671 1) 106 m3 = 169 J

106

Mara Vir. Martnez Gonzlez

El mismo calor lo cedera una masa mv de vapor de agua a 100o C que se condensa en agua lquida (liberando calor latente) y luego se enfra hasta 82o C Qvapor = mv Lv + mv ca T = mv (Lv + ca T ) = = mv 2257 103 J/kg + 4180 J/(kg K) 18 K = mv 2332240 J/kg As 91960 J = 0, 039 kg Qagua = Qvapor mv = 2332240 J/kg Ejercicio 5.- Queremos enfriar una bebida que est a 33o C. En un vaso aislante vertimos 0,24 kg de bebida y dos cubitos de hielo (cada uno de 0,025 kg y a 0o C de temperatura). (a) Cul ser la temperatura final de la bebida. (b) Si en lugar de dos cubitos ponemos seis, cul ser la temperatura final de la bebida? (c) Cuntos cubitos como mnimo son necesarios para que la bebida est a 0o C? (Suponer que la bebida tiene la misma capacidad calorfica del agua. Datos: calor especfifo del agua c = 4, 18 kJ/kg.K; calor de fusin del hielo Lf = 333, 5 kJ/kg.) Solucin: (a) El calor Qb que pierde la bebida al enfriarse es igual al calor Qh que ganan los cubitos de hielo. El calor que pierde la bebida es Qb = mb c (Ti Tf ) donde mb = 0, 24 kg es la masa de la bebida, c es la capacidad calorfica de la bebida (la del agua), Ti = 273, 15 K+33 K = 306, 15 K es la temperatura inicial de la bebida y Tf es la temperatura final, todava desconocida. Por otra parte el calor Qh que gana el hielo se invierte en fundir el hielo y subir la temperatura del agua resultante. Si suponemos que todo el hielo se funde tenemos Qh = mh Lf + mh c(Tf Th ) donde mh = 2 0, 025 kg es la masa de los dos cubitos de hielo, Lf = 333, 5 kJ/kg es el calor de fusin del hielo, Th = 273, 15 K es la temperatura inicial del agua que procede de la fusin del hielo y Tf es la temperatura final del agua. Igualando ambos calores obtenemos mb c (Ti Tf ) = mh Lf + mh c(Tf Th ) y despejando Tf obtenemos mb cTi + mh c(Ti 33) mh Lf (mb + mh ) cTi mh (33c + Lf ) mb cTi + mh cTh mh Lf = = = Tf = (mb + mh )c (mb + mh )c (mb + mh )c 0, 05 Kg (33 K 4, 18 kJ/kg.K + 333, 5 kJ/kg) mh (33c + Lf ) = 306, 15 = 286, 75 K = 13, 6o C = Ti (mb + mh )c (0, 24 + 0, 05) kg 4, 18 kJ/kg.K

(b) Si en lugar de dos cubitos de hielo ponemos seis, ahora mh = 0, 15 kg. Sustituyendo este valor en la expresin anterior obtenemos Tf = 10, 4 o C. Pero esto es imposible porque querra decir que despus de poner los cubitos que estn a 0o C toda la bebida est por debajo de cero grados! Lo que est fallando aqu es la hiptesis de que todo el hielo se ha fundido. Lo que ocurre en realidad es que solo se ha fundido una parte del hielo, y que el estado final es tal que la bebida y el hielo que queda estn a la misma temperatura de 0 o C. (c) Para que la bebida se enfre de 33o C a 0o C debe ceder un calor igual a Qb = (0, 24 kg) (4, 18 kJ/kg.K) 33 K= 33, 1 kJ. Este calor puede fundir una masa de hielo igual a mh = Qb /Lf = (33, 1 kJ)/(333, 5 kJ/kg) = 0,10 kg, que es la masa aproximada de cuatro cubitos de hielo. Por lo tanto, debemos poner cuatro cubitos para enfriar la bebida a cero grados. Si ponemos ms hielo, lo nico que conseguiremos es que haya ms hielo en la bebida, pero no conseguiremos bajar ms la temperatura. Ejercicio 6.- Un calormetro contiene 500 g de agua y 300 g de hielo, todo ello a 0o C. Se introduce dentro del calormetro un bloque metlico de 1000 g a 240o C, y el resultado es la fusin exacta de todo el hielo. Cul hubiese sido la temperatura final si la masa del bloque hubiese sido el doble? Solucin: Si al introducir una masa mm de metal a una temperatura Tm el nico efecto es fundir todo el hielo, debe cumplirse mm cm (Tm 273, 15) = mh Lf Si se introduce una masa de metal doble, habr calor suficiente para fundir el hielo y elevar la temperatura de los 500+300 = 800 g de agua lquida resultantes desde 0o C hasta una temperatura final Tf . Esta ser tambin la

107

Mara Vir. Martnez Gonzlez

temperatura final del metal. As pues, ahora debe cumplirse 2mm cm (Tm Tf ) = mh Lf + (mh + ma ) ca (Tf 273, 15) Pero ya hemos visto que mm cm Tm = mh Lf + 273, 15mm cm , lo que sustituido en la ecuacin anterior da 2mh Lf + 2 273, 15mm cm 2mm cm Tf = mh Lf + (mh + ma ) ca (Tf 273, 15) de donde se deduce 273, 15 [2mm cm + (mh + ma ) ca ] + mh Lf mh Lf Tf = = 273, 15 K + = 2mm cm + (mh + ma ) ca 2mm cm + (mh + ma ) ca 0, 3 kg 333, 5 103 J/kg = 273 K + ' 273 K + 24 K = 297 K 2 1 kg 417 J/(kg K) + 0, 8 kg 4180 J/(kg K) Ejercicio 7.- Un vaso contiene 500 g de agua a 80o C Cuntos gramos de hielo a -20o C hay que introducir para que la temperatura final sea 50o C? (Calor latente de fusin Lv = 333, 5 kJ/kg; calor especfico del hielo ch = 2106 J/(kgK) ) Solucin: Al pasar de una temperatura Ta = 273, 15 + 80 K a una temperatura final T = 273, 15 + 50 K el agua habr cedido un calor Qa = ma ca (Ta T ) Este calor habr sido absorbido por el hielo que pasa de Th = 273, 15 20 K a 273 K, se funde para dar agua lquida que luego pasa de 273,15 K a T = 273, 15 + 50 K. Qh = mh ch (273, 15 Th ) + mh Lf + mh ca (T 273, 15) = = mh [ch (273, 15 Th ) + Lf + ca (T 273, 15)] Entonces, Qh = Qa implica mh = = ma ca (Ta T ) = ch (273, 15 Th ) + Lf + ca (T 273, 15) 0, 5 kg 4180 J/(kg K) 30 K = 0, 107 kg 2106 J/(kg K) 20 K + 333500 J/kg + 4180 J/(kg K) 50 K

Tensin superficial
Hay que sealar una errata en la Fig.16.9 de la Unidad Didctica. El texto dice correctamente que un lquido que no moja, como es el caso del mercurio, no tiene tendencia a subir por un capilar, como hace el agua, sino todo lo contrario. Por ello, en la figura el nivel del lquido en el capilar debera estar por debajo del nivel en el recipiente exterior, y no por encima. Ejercicio 8.- En el ejercicio 7 del material complementario del Captulo 14 se afirmaba que si se vuelca un tubo lleno de mercurio sobre un recipiente con mercurio, el lquido dentro del tubo alcanza una altura de 0,76 cm sobre el nivel del lquido en el recipiente. No contradice esto lo que se acaba de afirmar: que si se introduce un capilar en un recipiente con mercurio, el nivel del mercurio en el capilar desciende? Solucin: Las situaciones son distintas. En el primer caso el tubo est cerrado en un extremo y al volcarlo queda un vaco en la parte superior. Por lo tanto, no hay presin sobre el mercurio en el tubo y el peso de la columna de mercurio es compensado por la presin atmosfrica sobre el mercurio en el recipiente. Por el contrario, en el segunco caso el tubo capilar est abierto y existe la misma presin atmosfrica sobre el mercurio dentro y fuera del tubo; ahora es la tensin superficial dentro del capilar la que tira del mercurio hacia abajo. Ejercicio 9.- Deducir una expresin que de la altura a la que se eleva por capilaridad un lquido en el espacio comprendido entre dos lminas paralelas introducidas en el mismo. (b) Se sumerjen en agua dos lminas planas de vidrio, paralelas y separadas 0,5 mm. Qu altura alcanzar el agua entre ellas?(Suponer que el ngulo de contacto es cero).

108

Mara Vir. Martnez Gonzlez

Solucin: Supongamos que la separacin entre las lminas es d y que los lados sumergidos de las lminas tienen una longitud l. Si el agua asciende entre ellas una altura h, el volumen del agua que se ha elevado respecto al nivel de la superficie del agua fuera de las lminas es vol = hld, y su peso sera W = (hdl)g. Este peso lo soporta la fuerza debida a la tensin superficial. Puesto que la lnea de contacto del agua con cada lmina es l, la fuerza total es F = 2l . As pues 2 0, 073 N m1 2 = 3 = 0, 030 m 2l = hdlg h= dg 10 kg m 3 5 104 m 9, 8 m s2 Es conveniente en este punto recordar lo que se estudi en el Captulo 1 sobre las escalas propias de los fenmenos capilares. Sin entrar en detalle en los mecanismos de formacin de gotas, hay que sealar que el tamao de una gota (en presencia de gravedad) depende de la raz cuadrada de la tensin superficial. Por lo tanto, si disminuye la tensin superficial de un lquido, disminuye el tamao de las gotas que se pueden formar. En esto se basa, por ejemplo, la accin de un detergente. Un detergente rebaja la tensin superficial de un lquido. Si en el lquido hay gotas de grasa, rebajar la tensin superficial hace que las gotas se reduzcan y puedan disolverse con ms facilidad. La tensin superficial determina tambin el tamao de una burbuja de gas en un lquido. La tensin superficial tiene dimensiones de fuerza/longitud, o tambin de energa/superficie. As, una burbuja de superficie S dentro de un lquido tiene una energa extra ES = S, debida a la tensin superficial. Para que esta energa sea mnima, la superficie de la burbuja, y por lo tanto el volumen, debera reducirse. Si no se reduce es porque la presin interior supera a la presin exterior e impide que la burbuja se comprima. En la situacin de equilibrio, la reduccin de energa E = S que supondra una reduccin de superficie debe compensarse con el trabajo que supondra la correspondiente reduccin de volumen. Si la presin interior de la burbuja es Pi y la presin exterior es Pe , el trabajo realizado es W = (Pi Pe )V. Entonces S = (Pi Pe )V Teniendo en cuenta que para una superficie esfrica de radio R S = 4 R2 S = 8 RR 4 3 R V = 4R2 R V = 3 la ecuacin de equilibrio implica 2 Pi Pe = R Es decir, para que exista una burbuja de radio R la presin interior debe exceder a la exterior en una cantidad 2 /R.

109

Mara Vir. Martnez Gonzlez

Bloque temtico 5: Fsica de Fluidos


Cuando un sistema no est en equilibrio termodinmico, es decir, cuando hay variaciones de presin, densidad, temperatura, etc., a lo largo del sistema, aparecen fuerzas internas. En el caso de un slido estas fuerzas pueden dar lugar a deformaciones, pero en el caso de un lquido dan lugar a flujos continuos de unas partes respecto a otras. El flujo puede caracterizarse por las lneas de corriente, que indican la velocidad en cada regin del fluido. Cuando las lneas de corriente son suaves y unas capas de fluido deslizan sobre otras, decimos que el rgimen es laminar. Sin embargo, a velocidades muy grandes el flujo es ms complicado y empiezan a producirse vrtices: decimos que el flujo es turbulento. Existe un parmetro adimensional denominado nmero de Reynolds que caracteriza el flujo: por debajo de un valor crtico del nmero de Reynolds el flujo es laminar, y por encima del mismo el flujo es turbulento. Normalmente, el movimiento de unas capas de fluido es frenado por las capas vecinas; es decir, hay una transferencia de cantidad de movimiento entre capas vecinas que caracterizamos por un parmetro denominado viscosidad. Llamamos fluido ideal a un fluido sin viscosidad. En un fluido ideal hay dos leyes importantes a considerar. La primera se expresa en la ecuacin de continuidad, vlida para fluidos en que la densidad se mantiene constante. Puesto que la cantidad de fluido se debe conservar, la cantidad de fluido que entra en una regin por unidad de tiempo debe ser igual a la que sale de dicha regin en el mismo tiempo. Por lo tanto, si la entrada en la regin tiene una seccin grande y la salida tiene una seccin menor, la velocidad del fluido en la salida debe ser mayor que en la entrada. La otra ley no es otra que la ley de conservacin de la energa aplicada a un medio continuo. La gravedad y las diferencias de presin entre los extremos de una regin del fluido realizan un trabajo sobre la regin del fluido que modifica su energa cintica. Esto se traduce en la ecuacin de Bernouilli. La presencia de la viscosidad altera las cosas. Ahora aparecen fuerzas viscosas que frenan al fluido y es necesaria una sobrepresin para mantener el flujo, incluso si el flujo sigue siendo laminar. Otro efecto importante de la viscosidad es la resistencia que ofrece un fluido al movimiento de un cuerpo en su seno. Para cuerpos esfricos pequeos esto da lugar a la ley de Stokes, que ya se estudio en el Bloque Temtico I.

110

Mara Vir. Martnez Gonzlez

CAPTULO 17
Viscosidad
En la seccin 17.2 de la Unidad Didctica se describe la forma de determinar la viscosidad dinmica de un fluido a partir de la fuerza tangencial necesaria para mover una placa respecto a otra vA (1) F = h As pues, las dimensiones de la viscosidad son MLT2 L [F ] [h] = [ ] = = ML1 T1 [v ] [A] LT1 L2 Si el rea de las placas es A = 1 cm2 = 104 m2 , su separacin es h = 1 cm, y es necesaria una fuerza F = 1 dina = 105 N para mover la placa superior a una velocidad v = 1 cm/s = 102 m/s, entonces la viscosidad es 1 dina 1 cm = = 1 g cm1 s1 = 1 poise 1 cm/s 1 cm2 es decir 105 N 102 m = 101 N s m2 = 101 Pa s 1 poise = 2 10 m/s 104 m2 Se define la viscosidad cinemtica como = con dimensiones ML1 T1 [ ] = = L2 T1 [ ] = [] ML3 Una unidad habitualmente utilizada para la viscosidad cinemtica es el stokes 1 poise 1 g cm1 s1 1 stokes = = = 1 cm2 s1 = 104 m2 s1 3 1 g/cm 1 g/cm3 Ecuacin de continuidad En un fluido incompresible (es decir, cuya densidad no vara con la presin) la conservacin de la masa se traduce en la conservacin del volumen ocupado por una masa de fluido dada A1 v1 = A2 v2

Fluidos idales
Teorema de Bernouilli Sean 1, 2 dos puntos en una lnea de corriente en un fluido ideal (sin viscosidad). Entonces 1 2 1 2 P1 + gh1 + v1 = P2 + gh2 + v2 2 2 Ejercicio1.- Un oleoducto tiene una capacidad de 240.000 m3 al da. En su mayor parte del recorrido el radio es de 60 cm, pero en algunos tramos el radio se reduce a la mitad. Si la presin en los tramos anchos es de 180 kPa, qu presin soportan los tramos estrechos? (Suponer que el crudo tiene una densidad de 800 kg/m3 y se pueden despreciar los efectos viscosos.) Solucin: La capacidad deloleoducto es 240.000 m3 = 2, 78 m3 /s C = 240.000 m3 /da = 86.400 s y la ecuacin de continuidad da va Aa = ve Ae = C donde el subndice a se refiere a los tramos anchos y el subndice e a los tramos estrechos. O en funcin de los radios Ra y Re = Ra /2 1 2 2 2 C = va Ra = ve Re = ve (Ra /2)2 = ve Ra 4

111

Mara Vir. Martnez Gonzlez

Introduciendo esta relacin en la ecuacin de Bernouilli, y teniendo en cuenta que la altura es constante a lo largo del oleoducto 2 2 1 2 1 2 1 1 4C C Pe + = Pa + Pe + ve = Pa + va 2 2 2 2 2 Ra 2 Ra y as 15C 2 15 800 kg m3 2, 782 m6 s2 = Pe = Pa 2 4 = 18 104 Pa 2 Ra 2 2 0, 64 m4 = (18 3, 62) 104 Pa = 14, 36 104 Pa Ejercicio 2.- En un punto de un tubo horizontal por el que circula agua la presin es de 1,25 atm y la seccin es de 18 cm2 . En otro, la presin es 1,2 atm y la seccin 9 cm2 . Calcular el nmero de litros por minuto que fluyen por el tubo. Solucin: De un modo similar al ejercicio anterior se llega a 2 2 1 1 C C = Pa + Pe + 2 Ae 2 Aa y teniendo en cuenta que Aa = 2Ae 1 1 1 2 1 4 3C 2 1 2 C = = Pe Pa = C 2 A2 A2 2 4A2 4A2 8A2 a e e e e Por lo tanto 8 0, 05 101325 Pa 9 104 m2 8 (Pa Pe ) A2 e C2 = = = 0, 0122 m6 /s2 3 3 103 kg/m3 y as C = 0, 11 m3 /s = 6600 litros/minuto

Ejercicio 3.- Una tubera horizontal de seccin circular est compuesta por tres tramos, el primero de seccin con rea 10 cm2 , un segundo con rea 20 cm2 y un tercero de nuevo con rea 10 cm2 . Un extremo de la tubera est conectado a un depsito abierto y muy grande de agua, mientras que el otro extremo est abierto al aire. Sabiendo que la diferencia de presin entre un punto del eje en el primer tramo y otro en el segundo tramo es de 500 Pa, calcular las presiones y velocidades en los puntos centrales de cada tramo de la tubera. (Despreciar los efectos viscosos) Solucin: Sea A el punto central del primer tramo de la tubera, B el punto central del segundo tramo y C el punto central del tercer tramo, en los que queremos calcular la velocidad y presin. Puesto que AB = 2AA = 2AC , la ecuacin de continuidad nos dice que vA = vC = 2vB . Entonces, la ecuacin de Bernouilli para los puntos A y B toma la forma 1 2 1 2 1 1 2 3 2 PB PA = vA vB = (2vB )2 vB = vB 2 2 2 2 2 de donde s s 1 2 2 (PB PA ) 2 500 Pa = = m/s vB = y vA = vC = m/s 3 3 3 3 10 kg/m 3 3 Puesto que vA = vC y las alturas son las mismas, tambin PA = PC = 1 atm = 101325 Pa y PB = PA + 500 = 101825 Pa Ejercicio 4.- Un tubo horizontal a ras de suelo conduce un fluido incompresible cuya densidad es 1, 30 103 kg/m . Para evitar un obstculo, el tubo se debe doblar hacia arriba, hasta alcanzar una altura de un metro. El tubo tiene rea transversal constante. Si la presin en la seccin inferior es 1,50 atm, calcule la presin en la parte superior del tubo. Solucin: Segn la ecuacin de continuidad, al ser la seccin del tubo constante, la velocidad de la tubera tambin es constante en todos sus puntos. Aplicando la ecuacin de Bernouilli en la parte superior e inferior del tubo tendremos Psup + ghsup = Pinf + ghinf
3

112

Mara Vir. Martnez Gonzlez

es decir Psup = Pinf + g (hinf hsup ) = 1, 5 101325 Pa + 1, 3 103 kg/m3 9, 8 m/s2 (0 1) m = = 139247 Pa = 1, 37 atm

Ejercicio 5.- El ala de un avin tiene una masa de 300 kg y una superficie de 20 m2 . El aire que pasa bajo el ala tiene una velocidad de 90 m/s mientras que el aire que pasa sobre el ala tiene una velocidad de 100 m/s. Qu fuerza neta acta sobre el ala? Solucin: Llamando 1 a un punto sobre el ala y 2 a un punto por debajo del ala, tenemos h1 ' h2 y as 1 2 2 = v1 v2 P2 P1 = 2 1 = 1, 3 kg m3 (1002 902 ) m2 s2 = 1235 Pa 2 Por lo tanto, la fuerza de sustentacin sobre el ala es Fsus = (P2 P1 )S = 1235 Pa 20 m2 = 24700 N Por otra parte el peso del ala es P = 300 kg 9, 8 m s2 = 2940 N de modo que la fuerza total sobre el ala es FT = Fsus P = 21760 N

Ejercicio 6.- Un depsito cilindrico cerrado est lleno de agua hasta la mitad de su capacidad. Por encima del agua hay aire comprimido que ejerce sobre el agua una presin de 3 atmsferas. En la pared del depsito, a 5 m por debajo del nivel del agua, hay un orificio. Calcular la velocidad del agua cuando escapa por el orificio. Solucin: De acuerdo con el teorema de Bernouilli 1 2 1 2 = P2 + gh2 + v2 P1 + gh1 + v1 2 2 donde el subndice 1 se refiere a un punto en la superficie del agua en el depsito y el subndice 2 se refiere a un punto en el orificio. Si suponemos que la seccin del depsito es grande y el descenso del nivel del agua es muy lento, podemos considerar prcticamente cero la velocidad del agua en la superficie. Entonces P1 = 3 atm, h1 = 0 y v1 = 0. Por otra parte, en el orificio tenemos P2 = 1 atm, h2 = 5 m y una velocidad de salida v2 , que es la que hay que calcular. As s 2 (P1 P2 ) 1 2 v = P1 P2 gh2 2gh2 v2 = 2 2 Sustituyendo valores y teniendo en cuenta que 2 101325 Pa P1 P2 = = 202, 65 m2 s2 1000 kg/m3 resulta finalmente p v2 = 2 202, 65 + 98 m s1 = 22, 4 m s1 Ejercicio 7.- Un depsito cilndrico de agua est cerrado por encima con una placa de 12 m2 y 1200 kg de peso que descansa sobre el agua y desciende a medida que lo hace el nivel del agua. El nivel del agua en el depsito es de 3,5 m de altura, que es tambin la posicin de la placa (es decir, no hay aire en el depsito). a) Calcular la presin en el fondo. b) Si se abre un orificio circular de 5 cm de radio a medio metro por encima del fondo, calclese el volumen de agua que sale por segundo por este orificio. (Se considera que el rea del orificio es muy pequea frente al rea del depsito). Solucin:

113

Mara Vir. Martnez Gonzlez

a) La presin sobre el fondo del deposito es la suma de las presiones debidas al peso del agua, Wa , el peso de la placa, Wp , y el peso del aire sobre la placa. Por lo tanto, llamando S a la superficie del fondo del depsito, tenemos que Wa + Wp ( Sh + mp ) g mp + Patmosfera = a + Patm = a h + Pfondo = g + Patm = S S S 1200 9, 8 Pa + 101.325 Pa = 136.605 Pa = 103 3, 5 + 12 b) Para calcular la velocidad de salida del agua por el orificio, usamos la ecuacin de Bernouilli, eligiendo como punto inicial A uno de la superficie en contacto con la placa y punto final B un punto del orificio. As, tendremos 1 2 1 2 PA + ghA + vA = PB + ghB + vB 2 2 de modo que 2 (PA PB ) 2 2 + 2g (hA hB ) = vA + vB Si S es grande y el orificio es pequeo, el nivel del agua desciende muy lentamente y podemos aproximar vA ' 0. Adems, PA = Patm + mp g/S, y PB = Patm , de donde PA PB = mp g/S. Finalmente hA hB = 3 m. Entonces 2mp g 2 1200 9, 8 2 2 2 + 2g (hA hB ) = m /s + 2 9, 8 3 m2 /s2 = 60, 76 m2 /s2 = vB S 103 12 As pues, vB = 7, 79 m/s y el volumen de agua que sale por el orificio es 2 vB AB = vB rB = 0, 061 m3 /s Ejercicio 8.- Un recipiente cilndrico de base muy amplia y abierto por arriba est lleno de agua hasta una altura de 30 cm. Se abre un orificio de 4 cm2 en la base y empieza a caer el agua. A que altura, por debajo de la base del recipiente, la seccin del chorro se habr reducido a la mitad? Solucin: Consideremos tres puntos en el flujo del agua: un punto 1 en la superficie del agua en el recipiente, un punto 2 en el orificio de salida y un punto 3 en donde la seccin del chorro se ha reducido a la mitad. El teorema de Bernouilli dice que 1 2 1 2 1 2 = p2 + gh2 + v2 = p3 + gh3 + v3 p1 + gh1 + v1 2 2 2 Si fijamos el origen de alturas en el orificio de salida, tenemos h1 = 30 cm, h2 = 0 y h3 = d. Por otra parte, en los tres puntos la presin es la presin atmosfrica. Finalmente, puesto que la seccin del recipiente es muy grande, podemos aproximar v1 = 0. Con esto, la expresin anterior se reduce a 1 2 1 2 gh1 = v2 = gd + v3 2 2 de donde p p y v3 = 2g (h1 + d) v2 = 2gh1 Pero la ecuacin de continuidad, llamando A2 a la seccin del orificio y A3 la seccin del chorro reducida, establece que v3 A2 = =2 v2 A2 = v3 A3 v2 A3 de modo que s 2g (h1 + d) =2 d = 3h1 = 0, 90 cm 2gh1 es decir, la seccin se habr contrado a la mitad a 90 cm por debajo del orificio.

En los ejercicios anteriores se calcula la velocidad de salida del agua para una altura dada del nivel del agua en el depsito. Sin embargo, a medida que el nivel del agua desciende tambin lo hace la velocidad de salida, de modo que esta velocidad vara con el tiempo. El siguiente ejercicio trata esta variacin.

114

Mara Vir. Martnez Gonzlez

Ejercicio 9.- Un depsito cilndrico con una base de rea S muy grande est abierto por arriba y lleno inicialmente de agua hasta una altura h0 . En la base hay un orificio de rea a (mucho menor que S ) que se abre en el instante t = 0, y el depsito empieza a vaciarse. Calcular cmo vara con el tiempo la altura del agua en el depsito. Cunto tiempo tarda el depsito en vaciarse? Solucin: Sea h(t) la altura del agua en un instante t > 0. La velocidad del agua en la superficie ser entonces dh(t)/dt. Si la velocidad de salida por el orificio es v (t), la ecuacin de continuidad dice que av (t) = S dh(t)/dt. Puesto que el depsito est abierto por arriba, la presin tanto en la superficie del agua como en el exterior del orificio es la presin atmsfrica. Entonces, el teorema de Bernouilli se reduce a 2 dh(t) 1 1 = v 2 (t) gh(t) + 2 dt 2 y teniendo en cuenta la relacin derivada de la ley de continuidad 2 2 2 1 1 dh(t) S dh(t) = gh(t) + 2 dt 2 a dt es decir s p 2g dh(t) = 2 h(t) S dt 1 a donde el signo menos indica que la altura disminuye. Esta es una ecuacin diferencial fcil de integrar por separacin de variables. En efecto s s p p 2g 2g dh(t) p h(t) h0 = 2 dt 2 t = 2 S S h(t) 1 1 a a Es decir, la altura del agua depende del tiempo en la forma 2 s p g h 2 i t h(t) = h0 2 S 1 a

El depsito se habr vaciado por completo en un instante t = T tal que h(T ) = 0 o v h i u 2 u 2 S 1 h0 s p t a g h 2 iT T = h0 = S g 2 1


a

Flujo de Poiseuille
Ejercicio 10.- La sangre tarda aproximadamente un segundo en fluir por un capilar sanguneo de 1 mm de longitud. Si el dimetro del capilar es de 7 micras y la cada de presin de 2.6 kPa, calcular la viscosidad de la sangre, suponiendo que el flujo es laminar. Solucin: Reordenando la frmula de Poiseuille, tenemos que P R4 = L 8I donde es la viscosidad, P = 2600 Pa es la diferencia de presiones en los extremos del capilar de longitud L = 103 m, R = 3, 5 106 m es el radio del capilar e I es el caudal. En un segundo, toda la sangre que est en el interior del capilar habr salido del capilar. Por tanto, el volumen de sangre que sale por segundo es precisamente el volumen del capilar. Es, decir, el caudal es I = R2 L m3 /s y as 2, 6 103 Pa 3, 5 106 m2 P R2 P R4 = = 0, 004 Pa s = = L 8I 8L2 8 106 m2 /s

Fuerza de arrastre

115

Mara Vir. Martnez Gonzlez

Ejercicio 11.- Una esfera de cobre de 0,4 g cae con una velocidad lmite de 5 cm/s en cierto lquido. Si la densidad del cobre es de 8900 kg/m3 y la del lquido es 2800 kg/m3 , qu viscosidad tiene el lquido? Solucin: Cuando la esfera de cobre empieza a hundirse en el lquido est sometida a tres fuerzas: su peso W , el empuje hidrosttico E y, finalmente, la fuerza de friccin. Si V es el volumen de la esfera, el peso es W = mg = Cu V g y el empuje es E = l V g. La fricin viene dada por la frmula de Stokes. En definitiva, llamando z a la coordenada vertical, la segunda ley de Newton se traduce en z d2 ~ ~ +E ~ +F ~R = (Cu l )V g + 6 Rv m 2 =W dt Cuando se llega a la velocidad lmite, las tres fuerzas cancelan exactamente, de manera que la velocidad de la bola es constante. En esta situacin, tenemos (Cu l )V g = 6 Rvlim o, teniendo en cuenta que V = (4/3) R3 4 ( l )2R2 g (Cu l ) R2 g = 6 vlim = Cu 3 9vlim El radio R se puede calcular a partir de la masa de la esfera de cobre 1/3 1/3 3mCu 3 0, 4 103 kg 4 3 R= = = 2, 2 103 mCu = Cu R 3 4Cu 4 8, 9 103 kg/m3 de modo que, finalmente 2/3 2(Cu l )g 3mCu = = 9vlim 4Cu 2 2 6100 kg/m3 9, 8 m/s2 2, 2 103 m2 = 1, 29 Pa s = 9 0, 05 m/s Ejercicio 12.- Con qu velocidad lmite se eleva una burbuja de aire de 1 mm de dimetro en un lquido de viscosidad 150 centipoises y densidad 0,90 g/cm3 ? Cul es la velocidad de la misma burbuja en el agua? (La viscosidad del agua es 1 centipoise) Solucin: En este caso, la fuerza de empuje hidrosttico supera al peso, de modo que la burbuja asciende. Entonces la fuerza de friccin acta hacia abajo. Es decir z d2 ~ ~ +E ~ +F ~R = (aire l )V g 6 Rv m 2 =W dt y la velocidad lmite es ahora 2 (l aire ) R2 g (aire l )V g = vlim = 6 R 9 El valor de la viscosidad es = 150 centipoises = 1, 50 poises = 0, 15 Pas y as 2 900 kg m3 106 m2 9, 8 m s2 = 0, 013 m/s vlim ' 9 0, 15 Pa s En agua, la velocidad lmite sera 4 F = (l aire ) r3 g 6 rv 3 2 agua aire r2 g 2 1000 kg m3 106 m3 9, 8 m s2 ' = 0, 022 m/s vlim = 9 9 0, 1 Pa s

116

Mara Vir. Martnez Gonzlez

You might also like